You are on page 1of 154

INSURANCE G01 CASE DIGESTS

At the trial, petitioner presented communications of the insurance


POLICY adjuster to Asian Surety revealing undeclared co-insurances with
the following: P30,000 with Wellington Insurance; P25,000 with
67. RCBC V. CA (FULL CASE POSTED IN THE FB GROUP - ADDITIONAL Empire Surety and P250,000 with Asian Surety undertaken by
CASE - PLEASE READ ON YOUR OWN- FIRST CASE TO BE DISCUSSED) insured Paramount on the same property covered by its policy with
Oriental whereas the only co-insurances declared in the subject
68. PACIFIC BANKING V . CA, 168 SCRA 1 (1988)
policy are those of P30,000.00 with Malayan P50,000.00 with South
Sea and P25.000.00 with Victory.
Facts: The defense of fraud, in the form of non-declaration of co-
An open fire insurance policy, was issued to Paramount Shirt insurances which was not pleaded in the answer, was also not
Manufacturing by Oriental Assurance Corporation to indemnify pleaded in the Motion to Dismiss.
P61,000.00, caused by fire to the factory’s stocks, materials and The trial court denied the respondent’s motion. Oriental filed
supplies. another motion to include additional evidence of the co-insurance
The insured was a debtor of Pacific Banking in the amount of which could amount to fraud.
(P800,000.00) and the goods described in the policy were held in The trial court still made Oriental liable for P 61,000. The CA
trust by the insured for Pacific Banking under trust receipts. reversed the trial court decision. Pacific Banking filed a motion for
The policy was endorsed to Pacific Banking as mortgagee/ trustor of reconsideration of the said decision of the respondent Court of
the properties insured, with the knowledge and consent of private Appeals, but this was denied for lack of merit.
respondent to the effect that "loss if any under this policy is payable
to the Pacific Banking Corporation". Issues:
A fire broke out on the premises destroying the goods contained in 1. WON unrevealed co-insurances Violated policy conditions No. 3
the building. 2. WON the insured failed to file the required proof of loss prior to
The bank sent a letter of demand to Oriental for indemnity. court action.
The company wasn’t ready to give since it was awaiting the
adjuster’s report. Held: Yes. Petition dismissed.
The company then made an excuse that the insured had not filed
any claim with it, nor submitted proof of loss which is a clear Ratio:
violation of Policy Condition No.11, as a result, determination of the 1. Policy Condition No. 3 explicitly provides:
liability of private respondent could not be made. 3. The Insured shall give notice to the Company of any insurance
Pacific Banking filed in the trial court an action for a sum of money already effected, or which may subsequently be effected, covering
for P61,000.00 against Oriental Assurance. any of the property hereby insured, and unless such notice be given
Page 1 of 154
INSURANCE G01 CASE DIGESTS

and the particulars of such insurance or insurances be stated in or shall not be invalidated by any act or neglect—except fraud or
endorsed on this Policy by or on behalf of the Company before the misrepresentation, or arson—of the mortgagor or owner/trustee of
occurrence of any loss or damage, all benefit under this policy shall the property insured; provided, that in case the mortgagor or
be forfeited. owner/ trustee neglects or refuses to pay any premium, the
The insured failed to reveal before the loss three other insurances. mortgagee/ trustor shall, on demand pay the same.”
Had the insurer known that there were many co-insurances, it could The paragraph clearly states the exceptions to the general rule that
have hesitated or plainly desisted from entering into such contract. insurance as to the interest of the mortgagee, cannot be
Hence, the insured was guilty of clear fraud. invalidated; namely: fraud, or misrepresentation or arson.
Concrete evidence of fraud or false declaration by the insured was Concealment of the aforecited co-insurances can easily be fraud, or
furnished by the petitioner itself when the facts alleged in the policy in the very least, misrepresentation.
under clauses "Co-Insurances Declared" and "Other Insurance Undoubtedly, it is but fair and just that where the insured who is
Clause" are materially different from the actual number of co- primarily entitled to receive the proceeds of the policy has by its
insurances taken over the subject property. fraud and/or misrepresentation, forfeited said right.
As the insurance policy against fire expressly required that notice Petitioner further stressed that fraud which was not pleaded as a
should be given by the insured of other insurance upon the same defense in private respondent's answer or motion to dismiss, should
property, the total absence of such notice nullifies the policy. be deemed to have been waived. It will be noted that the fact of
Petitioner points out that Condition No. 3 in the policy in relation to fraud was tried by express or at least implied consent of the parties.
the "other insurance clause" supposedly to have been violated, Petitioner did not only object to the introduction of evidence but on
cannot certainly defeat the right of the petitioner to recover the the contrary, presented the very evidence that proved its existence.
insurance as mortgagee/assignee. Hence, they claimed that the 2. Generally, the cause of action on the policy accrues when the loss
purpose for which the endorsement or assignment was made was occurs, But when the policy provides that no action shall be brought
to protect the mortgagee/assignee against any untoward act or unless the claim is first presented extrajudicially in the manner
omission of the insured. It would be absurd to hold that petitioner is provided in the policy, the cause of action will accrue from the time
barred from recovering the insurance on account of the alleged the insurer finally rejects the claim for payment
violation committed by the insured. In the case at bar, policy condition No. 11 specifically provides that
It is obvious that petitioner has missed all together the import of the insured shall on the happening of any loss or damage give
subject mortgage clause which specifically provides: notice to the company and shall within fifteen (15) days after such
“Loss, if any, under this policy, shall be payable to the PACIFIC loss or damage deliver to the private respondent (a) a claim in
BANKING CORPORATION Manila mortgagee/trustor as its interest writing giving particular account as to the articles or goods
may appear, it being hereby understood and agreed that this destroyed and the amount of the loss or damage and (b) particulars
insurance as to the interest of the mortgagee/trustor only herein, of all other insurances, if any.
Page 2 of 154
INSURANCE G01 CASE DIGESTS

Twenty-four days after the fire did petitioner merely wrote letters 69. ORIENTAL ASSURANCE V . COURT OF APPEALS, 200 SCRA 459
to private respondent to serve as a notice of loss. It didn’t even (1991)
furnish other documents. Instead, petitioner shifted upon private
respondent the burden of fishing out the necessary information to
ascertain the particular account of the articles destroyed by fire as ORIENTAL ASSURANCE v. CA (PANAMA SAW MILL)
well as the amount of loss. Since the required claim by insured, 200 SCRA 459
together with the preliminary submittal of relevant documents had
not been complied with, it follows that private respondent could MELENCIO-HERRERA; August 9, 1991
not be deemed to have finally rejected petitioner's claim and
NATURE
therefore there was no cause of action.
It appearing that insured has violated or failed to perform the Petition for review on certiorari
conditions under No. 3 and 11 of the contract, and such violation or
want of performance has not been waived by the insurer, the FACTS
insured cannot recover, much less the herein petitioner.
- Sometime in January 1986, private respondent Panama Sawmill
Co., Inc. (Panama) bought, in Palawan, 1,208 pieces of apitong logs,
with a total volume of 2,000 cubic meters. It hired Transpacific
Towage, Inc., to transport the logs by sea to Manila and insured it
against loss for P1-M with petitioner Oriental Assurance
Corporation (Oriental Assurance).

- While the logs were being transported, rough seas and strong
winds caused damage to one of the two barges resulting in the loss
of 497 pieces of logs out of the 598 pieces loaded thereon.

- Panama demanded payment for the loss but Oriental Assurance


refuse on the ground that its contracted liability was for "TOTAL
LOSS ONLY."

- Unable to convince Oriental Assurance to pay its claim, Panama


filed a Complaint for Damages against Oriental Assurance before
the Regional Trial Court.

Page 3 of 154
INSURANCE G01 CASE DIGESTS

- RTC ordered Oriental Assurance to pay Panama with the view that - More importantly, the insurer's liability was for "total loss only." A
the insurance contract should be liberally construed in order to total loss may be either actual or constructive (Sec. 129, Insurance
avoid a denial of substantial justice; and that the logs loaded in the Code). An actual total loss is caused by:
two barges should be treated separately such that the loss
sustained by the shipment in one of them may be considered as (a) A total destruction of the thing insured;
"constructive total loss" and correspondingly compensable. CA (b) The irretrievable loss of the thing by sinking, or by being
affirmed in toto. broken up;
ISSUE (c) Any damage to the thing which renders it valueless to the
WON Oriental Assurance can be held liable under its marine owner for the purpose for which he held it; or
insurance policy based on the theory of a divisible contract of (d) Any other event which effectively deprives the owner of the
insurance and, consequently, a constructive total loss possession, at the port of destination, of the thing insured.
HELD (Section 130, Insurance Code).

NO - A constructive total loss is one which gives to a person insured a


right to abandon, under Section 139 of the Insurance Code. This
- The terms of the contract constitute the measure of the insurer provision reads:
liability and compliance therewith is a condition precedent to the
insured's right to recovery from the insurer. Whether a contract is SECTION 139. A person insured by a contract of marine insurance
entire or severable is a question of intention to be determined by may abandon the thing insured, or any particular portion thereof
the language employed by the parties. The policy in question shows separately valued by the policy, or otherwise separately insured,
that the subject matter insured was the entire shipment of 2,000 and recover for a total loss thereof, when the cause of the loss is a
cubic meters of apitong logs. The fact that the logs were loaded on peril injured against,
two different barges did not make the contract several and divisible (a) If more than three-fourths thereof in value is actually lost, or
as to the items insured. The logs on the two barges were not would have to be expended to recover it from the peril;
separately valued or separately insured. Only one premium was
paid for the entire shipment, making for only one cause or (b) If it is injured to such an extent as to reduce its value more
consideration. The insurance contract must, therefore, be than three-fourths;
considered indivisible.
xxx xxx xxx

Page 4 of 154
INSURANCE G01 CASE DIGESTS

- The requirements for the application of Section 139 of the 70. FORTUNE ASSURANCE V . COURT OF APPEALS, 244 SCRA 308
Insurance Code, quoted above, have not been met. The logs (1995)
involved, although placed in two barges, were not separately valued
by the policy, nor separately insured. Resultantly, the logs lost in the G.R. No. 115278 May 23, 1995
damaged barge in relation to the total number of logs loaded on the
same barge cannot be made the basis for determining constructive Petitioner: FORTUNE INSURANCE AND SURETY CO., INC. (Fortune)
total loss. The logs having been insured as one inseparable unit, the
Respondent: PRODUCERS BANK OF THE PHILIPPINES (PBP)
correct basis for determining the existence of constructive total loss
is the totality of the shipment of logs. Of the entirety of 1,208, FACTS:
pieces of logs, only 497 pieces thereof were lost or 41.45% of the
entire shipment. Since the cost of those 497 pieces does not exceed > PBP filed against Fortune a complaint for recovery of the sum of
75% of the value of all 1,208 pieces of logs, the shipment cannot be P725,000.00 under the policy issued by Fortune. The money was
said to have sustained a constructive total loss under Section 139(a) allegedly lost during a robbery of Producer's armored vehicle while
it was in transit to transfer the money from its Pasay City Branch to
of the Insurance Code.
its head office in Makati along Taft Avenue.
Disposition judgment under review is SET ASIDE
>The armored car was driven by Benjamin Magalong escorted by
Security Guard Saturnino Atig.

>Driver Magalong was assigned by PRC Management Systems with


the PBP by virtue of an Agreement and Atiga was assigned by
Unicorn Security Services, Inc. by virtue of a contract of Security
Service.

>After an investigation conducted by the Pasay police authorities,


the driver Magalong and guard Atiga were charged, together with
Edelmer Bantigue, Reynaldo Aquino and John Doe, with violation of
P.D. 532 (Anti-Highway Robbery Law) before the Fiscal of Pasay City.

>Demands were made by PBP but Fortune refused to pay as the loss
is excluded from the coverage of the insurance policy which is
stipulated under "General Exceptions" Section (b) which reads as
follows:

Page 5 of 154
INSURANCE G01 CASE DIGESTS

GENERAL EXCEPTIONS Fortune is exempt from liability under the general exceptions clause
of the insurance policy.
The company shall not be liable under this policy in
report of >It should be noted that the insurance policy entered into by the
parties is a theft or robbery insurance policy which is a form of
xxx xxx xxx casualty insurance (Section 174 of the Insurance Code). Other than
what is mentioned in the provision, the rights and obligations of the
(b) any loss caused by any dishonest, fraudulent or parties must be determined by the terms of their contract, taking
criminal act of the insured or any officer, employee, into consideration its purpose and always in accordance with the
partner, director, trustee or authorized general principles of insurance law.
representative of the Insured whether acting alone
or in conjunction with others. . . . >The purpose of the exception is to guard against liability should the
theft be committed by one having unrestricted access to the
8. The plaintiff opposes the contention of the property. In such cases, the terms specifying the excluded classes
defendant and contends that Atiga and Magalong are to be given their meaning as understood in common
are not its "officer, employee, . . . trustee or speech. The terms "service" and "employment" are generally
authorized representative . . . at the time of the associated with the idea of selection, control, and compensation.
robbery.
>A contract of insurance is a contract of adhesion, thus any
>RTC & CA: held that there should be recovery. The trial court ruled ambiguity therein should be resolved against the insurer, or it
that Magalong and Atiga were not employees or representatives of should be construed liberally in favor of the insured and strictly
Producers. The wages and salaries of both Magalong and Atiga are against the insurer. Limitations of liability should be regarded with
presumably paid by their respective firms, which alone wields the extreme jealousy and must be construed
power to dismiss them. Neither is the Court prepared to accept the in such a way, as to preclude the insurer from non-compliance with
proposition that driver Magalong and guard Atiga were the its obligation.
"authorized representatives" of plaintiff.
>If the terms of the contract are clear and unambiguous, there is no
ISSUE: room for construction and such terms cannot be enlarged or
diminished by judicial construction.
W/N the recovery in the policy is precluded under the general
exceptions clause? >An insurance contract is a contract of indemnity. It is settled that
the terms of the policy constitute the measure of the insurer's
HELD: liability. In the absence of statutory prohibition to the contrary,
insurance companies have the same rights as individuals to limit
YES.
Page 6 of 154
INSURANCE G01 CASE DIGESTS

their liability and to impose whatever conditions they deem best others or another in a special capacity, as an agent, and is
upon their obligations not inconsistent with public policy. interchangeable with "agent."

>It was clear that Fortune’s intention is to exclude and exempt from
protection and coverage losses arising from dishonest, fraudulent,
or criminal acts of persons granted or having unrestricted access to 71. GREAT PACIFIC LIFE V . COURT OF APPEALS, 89 SCRA 543 (1979)
Producers' money or payroll. When it used then the term
"employee," it must have had in mind any person who qualifies as GREAT PACIFIC LIFE v. CA (NGO HING)
such as generally and universally understood, or jurisprudentially 89 SCRA 543
established in the light of the four standards in the determination of DE CASTRO, J; April 30, 1979
the employer-employee relationship, or as statutorily declared even !
in a limited sense as in the case of Article 106 of the Labor Code NATURE
which considers the employees under a "labor-only" contract as Petition for certiorari
employees of the party employing them and not of the party who !
supplied them to the employer. FACTS
- On March 14, 1957, private respondent Ngo Hing filed an
>Fortune claims that Producers' contracts with PRC Management application with the Great Pacific Life Assurance Co. (Pacific Life)
Systems and Unicorn Security Services are "labor-only" contracts. for a 20 year endowment policy of P50k on the life of his 1
But even granting for the sake of argument that these contracts year old daughter, Helen. Ngo Hing supplied the essetntial data
were not "labor-only" contracts, and PRC Management Systems and which petitioner Mondragon, branch manager of the Pacific Life
Unicorn Security Services were truly independent contractors, we in Cebu, wrote on the corresponding form in his own
are satisfied that Magalong and Atiga were, in respect of the handwriting, later typing the data on an application form signed by
transfer of Producer's money from its Pasay City branch to its head Ngo Hing. The latter paid the P1077.75 annual premium but
office in Makati, its "authorized representatives" who served as retained P1,317 as commission as he was also a duly
such with its teller Maribeth Alampay. authorized agent of Pacific Life. The binding deposit receipt
was then issued to Ngo Hing;
>Producers entrusted the three with the specific duty to safely
transfer the money to its head office, with Alampay to be Mondragon handwrote his strong recommendation for the
responsible for its custody in transit; Magalong to drive the armored approval of the application on the
vehicle which would carry the money; and Atiga to provide the back of the form.
needed security for the money, the vehicle, and his two other
companions. In short, for these particular tasks, the three acted as - On April 30, Mondragon received a letter from Pacific Life
agents of Producers. A "representative" is defined as one who which stated that the 20 year
represents or stands in the place of another; one who represents

Page 7 of 154
INSURANCE G01 CASE DIGESTS

endowment plan was not available for minors below 7, but that -This implies the receipt is merely an acknowledgement, on
Pacific Life could consider the same under the Juvenile Triple behalf of the company, that the Cebu branch of Pacific Life had
Action Plan, advising that if the offer was acceptable, the received the premium and had accepted the application subject to
Juvenile Non-Medical Declaration be sent to the company. processing by the insurance company, which will approve or reject it
depending on whether the applicant is insurable on standard rates.
-Mondragon allegedly failed to inform Ngo Hing of the non- As such, the receipt was never in force-it does not insure outright.
acceptance of the insurance plan, instead writing Pacific Life No liability attaches until the principal approves the risk and a
again, recommending the approval of the endowment plan to receipt is given by the agent; because private respondent failed
children since customers had been asking for such coverage since to accept Pacific Life's offer for the Juvenile Triple Action plan,
1954. there was no meeting of the minds and thus no contract. Also,
being an authorized agent of Pacific Life, Ngo Hing must have
-On May 28, 1957, Helen died of influenza. Ngo Hing sought the known the company did not offer the insurance applied for and
payment of the proceeds of the insurance, but having failed to do merely took a chance on Mondragon's recommendation.
so, filed an action for recovery with the CFI of Cebu. The Court Disposition the decision appealed from is set aside, absolving
ordered Pacific Life to pay P50k with 6% interest, hence this Pacific Life from their civil liabilities
petition.

!
ISSUE
WON the binding deposit receipt constituted a temporary
contract of the life insurance in question
!

HELD
NO
- The binding deposit receipt is merely a provisional contract
and only upon compliance with the ff conditions: (1) that the
company be satisfied that the applicant was insurable on
standard rates (2) that if the company does not accept the
application and offers a different policy, the insurance contract
shall not be binding until the applicant accepts the new policy
(3) that if the applicant is not found to be insurable on standard
rates and the application is disapproved, the insurance shall not be
in force at any time and the premium be returned to the applicant.

Page 8 of 154
INSURANCE G01 CASE DIGESTS

72. BONIFACIO BROTHERS V . MORA, 20 SCRA 261 (1967)  The insurance company filed its answer with a
counterclaim for interpleader, requiring the Bonifacio
FACTS: Bros. Inc. and the H.S. Reyes, Inc. to interplead in order to
determine who has better right to the insurance proceeds
 Enrique Mora, owner of Oldsmobile sedan model 1956 in question.
mortgaged the same to the H.S. Reyes, Inc., with the  Municipal Court rendered a decision declaring the H.S.
condition that the former would insure the automobile Reyes, Inc. as having a better right to the disputed amount
with the latter as beneficiary. and ordering State Bonding & Insurance Co. Inc. to pay to
 The automobile was thereafter insured with the State the H. S. Reyes, Inc. the said sum of P2,002.73.
Bonding & Insurance Co., Inc., and motor car insurance  From this decision, the appellants elevated the case to the
policy was issued to Enrique Mora. CFI of Manila which rendered a decision, affirming the
 During the effectivity of the insurance contract, the car met decision of the Municipal Court. The Bonifacio Bros. Inc.
with an accident. Enrique Mora, without the knowledge and the Ayala Auto Parts Co. moved for reconsideration of
and consent of the H.S. Reyes, Inc., authorized the the decision, but the trial court denied the motion. Hence,
Bonifacio Bros. Inc. to furnish the labor and materials, this appeal.
some of which were supplied by the Ayala Auto Parts Co.
For the cost of labor and materials, Enrique Mora was ISSUE: WON there is privity of contract between the Bonifacio Bros.
billed at P2,102.73 through the H.H. Bayne Adjustment Co. Inc. and the Ayala Auto Parts Co. on the one hand and the insurance
 The insurance company after claiming a franchise in the company on the other.
amount of P100, drew a check in the amount of P2,002.73,
as proceeds of the insurance policy, payable to the order HELD: NONE
of Enrique Mora or H.S. Reyes,. Inc., and entrusted the
check to the H.H. Bayne Adjustment Co. for disposition and Appellant = Bonifacio Bro., Inc.
delivery to the proper party.
 In the meantime, the car was delivered to Enrique Mora From the undisputed facts and from the pleadings it will be seen
without the consent of the H.S. Reyes, Inc., and without that the appellants' alleged cause of action rests exclusively upon
payment to the Bonifacio Bros. Inc. and the Ayala Auto the terms of the insurance contract. The appellants seek to recover
Parts Co. of the cost of repairs and materials. the insurance proceeds, and for this purpose, they rely upon
 Upon the theory that the insurance proceeds should be paragraph 4 of the insurance contract document executed by and
paid directly to them, the Bonifacio Bros. Inc. and the between the State Bonding & Insurance Company, Inc. and Enrique
Ayala Auto Parts Co. filed a complaint with the MTC of Mora. The appellants are not mentioned in the contract as parties
Manila against Enrique Mora and the State Bonding & thereto nor is there any clause or provision thereof from which we
Insurance Co., Inc. for the collection of the sum of can infer that there is an obligation on the part of the insurance
P2,002.73. company to pay the cost of repairs directly to them.

Page 9 of 154
INSURANCE G01 CASE DIGESTS

It is fundamental that contracts take effect only between the expressed or implied exists. We, therefore, agree with the trial court
parties thereto, except in some specific instances provided by law that no cause of action exists in favor of the appellants in so far as
where the contract contains some stipulation in favor of a third the proceeds of insurance are concerned. The appellants' claim, if
person.1Such stipulation is known as stipulation pour autrui or a at all, is merely equitable in nature and must be made effective
provision in favor of a third person not a pay to the contract. Under through Enrique Mora who entered into a contract with the
this doctrine, a third person is allowed to avail himself of a benefit Bonifacio Bros. Inc. This conclusion is deducible not only from the
granted to him by the terms of the contract, provided that the principle governing the operation and effect of insurance contracts
contracting parties have clearly and deliberately conferred a favor in general, but is clearly covered by the express provisions of
upon such person. section 50 of the Insurance Act which read:

In this connection, this Court has laid down the rule that the fairest The insurance shall be applied exclusively to the proper
test to determine whether the interest of a third person in a interests of the person in whose name it is made unless
contract is a stipulation pour autrui or merely an incidental otherwise specified in the policy.
interest, is to rely upon the intention of the parties as disclosed by
their contract.4 In the instant case the insurance contract does not The policy in question has been so framed that "Loss, if any, is
contain any words or clauses to disclose an intent to give any payable to H.S. Reyes, Inc.," which unmistakably shows the
benefit to any repairmen or materialmen in case of repair of the intention of the parties.
car in question. The parties to the insurance contract omitted such
stipulation, which is a circumstance that supports the said The final contention of the appellants is that the right of the H.S.
conclusion. On the other hand, the "loss payable" clause of the Reyes, Inc. to the insurance proceeds arises only if there was loss
insurance policy stipulates that "Loss, if any, is payable to H.S. and not where there is mere damage as in the instant case. Suffice it
Reyes, Inc." indicating that it was only the H.S. Reyes, Inc. which to say that any attempt to draw a distinction between "loss" and
they intended to benefit. "damage" is uncalled for, because the word "loss" in insurance law
embraces injury or damage. Indeed, according to sec. 120 of the
If it were the intention of the insurance company to make itself Insurance Act, a loss may be either total or partial.
liable to the repair shop or materialmen, it could have easily
inserted in the contract a stipulation to that effect.

Another cogent reason for not recognizing a right of action by the


appellants against the insurance company is that "a policy of
insurance is a distinct and independent contract between the
insured and insurer, and third persons have no right either in a
court of equity, or in a court of law, to the proceeds of it, unless
there be some contract of trust, expressed or implied between the
insured and third person."5 In this case, no contract of trust,

Page 10 of 154
INSURANCE G01 CASE DIGESTS

73. HEIRS OF L.G. MARAMAG V . MARAMAG, 588 SCRA 774 (2009) THE COMPLAINT OR PETITION FAILED TO STATE A CAUSE OF
ACTION AS TO
DECLARE AS VOID THE DESIGNATION OF EVA AS BENEFICIARY for
1. PETITIONERS were legitimate wife and children of Loreto Loreto revoked her
Maramag. designation and already disqualified her.

2. PETITIONERS' CONTENTION: 6. GREPALIFR CONTENTION: Eva was not designated as an insurance


a. RESPONDENTS were Loreto''s illegitimate family. policy beneficiary, that r claims of the illegitimate children were
denied because Loreto was ineligible for the insurance due to the
b.Eva Maramag was concubine of Loreto and a suspect in the killing misrepresentation in his application form that he was not more
of Loreto. Thus, she is disqualified to receive any proceeds from his than 65 years old.
insurance policies from Insular Life Assurance (INSULAR) and Great
Pacific Life 7.Both Insular and Grepalife countered that the insurance proceeds
Assurance (GREPALIFE) belong exclusively to the
designated beneficiaries in the policies, not to the estate or to the
c. The illegitimate children were entitled only to 1/2 of the legitime heirs of the insured.
of the legitimate children, Grepalife also reiterated that it had disqualified Eva as a beneficiary
thus the proceeds released to the illegitimate children were when it ascertained that
inofficious and should be Loreto was legally married to Vicenta Pangilinan Maramag.
reduced.
d. x x x 8.RTC: In favor of the respondents. Neither could the plaintiffs
invoked (sic) the law on donations or the rules on testamentary
3. TRO and writ of preliminary injunction were filed by the succession in order to defeat the right of herein defendants to
petitioner because, some part of collect the insurance indemnity. The beneficiary in a contract of
the insurance proceeds had already been released in favor of one of insuranceis not the donee spoken in the law of donation. The rules
the illegitimate and the on testamentary succession cannot apply here, for the insurance
rest are to be releases in favor of the other illegitimate. indemnity does not partake of a donation.

4.Insular admitted that Loreto misrepresented Eva as his legitimate THE PROCEEDS BELONG EXCLUSIVELY THE BENEFICIARY AND NOT
wife and Odessa, Karl TO THE ESTATE OF THE PERSON. NO SUFFICIENT CAUSE OF ACTION
Brian, and Trisha Angelie as his legitimate children, and that they AGAINST THE ILLEGITIMATE FLR THE REDUCTION AND/OR
filed their claims for the DECLARATION OF INOFFICIOUS OF
insurance proceeds of the insurance policies. DONATION AS PRIMARY BENEFICIARY. EVA AS THE CONCUBINE
CANNOT BE A
5. INSULAR ALLEGATION: BENEFICIARY

Page 11 of 154
INSURANCE G01 CASE DIGESTS

stipulations or indemnity. In such a case, third parties may directly


9. CA: Affirmed the decision of the RTC. The distribution of the sue and claim from the insurer.
insurance proceeds is governed primarily by the Insurance Code,
and the provisions of the Civil Code are irrelevant and inapplicable. Petitioners are third parties to the insurance contracts with Insular
and Grepalife and, thus, are not entitled to the proceeds thereof.
ISSUE: Accordingly, respondents Insular and Grepalife have no legal
Whether or not the legitimate children of the insured decease shall obligation to turn over the insurance proceeds to petitioners. The
receive the proceeds of revocation of Eva as a beneficiary in one policy and her
the insurance that originally designated to Eva? disqualification as such in another are of no moment considering
that the designation of the illegitimate children as beneficiaries in
HELD: Loreto's insurance policies remains valid. Because no legal
No. Even assuming Insular disqualified Eva as a beneficiary, her proscription exists in naming as beneficiaries the children of illicit
share should not have been distributed to the legitimate heirs of relationships by the insured,22 the shares of Eva in the insurance
the insured deceased. proceeds, whether forfeited by the court in view of the prohibition
on donations under Article 739 of the Civil Code or by the insurers
RATIONALE: themselves for reasons based on the insurance contracts, must be
It is evident from the face of the complaint that petitioners are not awarded to the said illegitimate children, the designated
entitled to a favorable judgment in light of Article 2011 of the Civil beneficiaries.
Code which expressly provides that insurance contracts shall be
governed by special laws, i.e., the Insurance Code. Section 53 of the
Insurance Code states—

SECTION 53. The insurance proceeds shall be applied exclusively to


the proper interest of
the person in whose name or for whose benefit it is made unless
otherwise specified in the
policy.

Pursuant thereto, it is obvious that the only persons entitled to


claim the insurance proceeds are either the insured, if still alive; or
the beneficiary, if the insured is already deceased, upon the
maturation of the policy. The exception to this rule is a situation
where the insurance contract was intended to benefit third persons
who are not parties to the same in the form of favorable

Page 12 of 154
INSURANCE G01 CASE DIGESTS

74. COQUIA V. FIELDMEN’ S INSURANCE , 26 SCRA 178 (1968) Held:


1. YES
2. While the general rule is that only parties to a contract may
Coquia vs Fieldmen’s Insurance bring an action based thereon, one exception is found
under Article 1311 of the Civil Code.
Facts: 3. It provides that If a contract should contain some stipulation
1. Fieldmen's Insurance Company, Inc. issued, in favor of the in favor of a third person, he may demand its fulfillment
Manila Yellow Taxicab Co., Inc. a common carrier accident provided he communicated his acceptance to the obligor
insurance policy. before its revocation.
2. It was stipulated in said policy that the company will 4. These are contracts pour autrui wherein enforcement of a
indemnify the Insured in the event of accident caused by or contract may be demanded by a third party for whose
arising out of the use of Motor Vehicle against all sums benefit it was made, although not a party to the contract.
which the Insured will become legally liable to pay. 5. In this case, the policy contained a stipulation which states
3. While the policy was in force, or on February 10, 1962, a the following: Section I — Liability to Passengers. 1. The
taxicab of the Insured, driven by Carlito Coquia, met a Company will, subject to the Limits of Liability and under
vehicular accident at Mangaldan, Pangasinan, in the Terms of this Policy, indemnify the Insured in the event
consequence of which Carlito died. of accident caused by or arising out of the use of Motor
4. The Insured filed therefor a claim for P5,000.00 to which the Vehicle against all sums which the Insured will become
Company replied with an offer to pay P2,000.00, by way of legally liable to pay in respect of: Death or bodily injury to
compromise. any fare-paying passenger including the Driver ... who is
5. The Insured rejected the same and made a counter-offer for riding in the Motor Vehicle insured at the time of accident
P4,000.00, but the Company did not accept it. or injury.
6. The insured and Carlito’s parents or the Coquias filed a
complaint against the company to collect the proceeds of 6. Another stipulation provides that “In the event of death of
the insurance policy. any person entitled to indemnity under this Policy, the
7. As a defence, the company argued lack of cause of action on Company will, in respect of the liability incurred by such
the part of the Coquias. person, indemnify his personal representatives in terms of
8. RTC ruled for the plaintiffs sentencing the company to pay and subject to the limitations of this Policy, provided, that
4k. such representatives shall, as though they were the Insured,
observe, fulfill and be subject to the Terms of this Policy
Issue: insofar as they can apply.”
1. Whether or not the Coquias can claim under the policy even
if they are alleged to not have a cause of action against the 7. Pursuant to these stipulations, the Company "will
company as they are not parties to the insurance policy? indemnify any authorized Driver who is driving the Motor
Vehicle" of the Insured and, in the event of death of said

Page 13 of 154
INSURANCE G01 CASE DIGESTS

driver, the Company shall, likewise, "indemnify his personal 75. LOPEZ V . DEL ROSARIO , 44 PHIL 98 (1922)
representatives."
Lopez vs. Del Rosario (1922)
8. Thus, the policy under consideration is typical of
contracts pour autrui, this character being made more Facts:
manifest by the fact that the deceased driver paid fifty 1. Mrs. del Rosario owned a bonded warehouse in Manila.
percent (50%) of the corresponding premiums, which were Engaged in the business of a warehouse keeper, she stored
deducted from his weekly commissions. copra and other merchandise in building.
2. Among the persons who had copra deposited in the Del
9. Under these conditions, it is clear that the Coquias — who, Rosario warehouse was Froilan Lopez, the holder of
admittedly, are the sole heirs of the deceased — have a fourteen warehouse receipts in his own name, and the
direct cause of action against the Company,3 and, since they name of Elias T. Zamora.
could have maintained this action by themselves, without 3. Lopez named a declared value of P107,990.40. The
the assistance of the Insured, it goes without saying that warehouse receipts provided: (1) For insurance at the rate
they could and did properly join the latter in filing the of 1 per cent per month on the declared value; (2) the
complaint herein. company reserves to itself the right to raise and/or lower
the rates of storage and/or of insurance on giving one
calendar month's notice in writing; (3) this warrant carries
no insurance unless so noted on the face hereof, cost of
which is in addition to storage; (4) the time for which
storage and/or insurance is charged is thirty (30) days; (5)
payment for storage and/or insurance, etc., shall be made
in advance, and/or within five (5) days after presentation of
bill.
4. It is admitted that insurance was paid by Lopez to May 18,
1920, but not thereafter.
5. Mrs. Del Rosario secured insurance on the warehouse and
its contents with the National Insurance Co., Inc., the
Commercial Union Insurance Company, the Alliance
Insurance Company, the South British Insurance Co., Ltd.,
and the British Traders Insurance Co., Ltd.
6. The warehouse caught fire. Everything was destroyed. Only
copra worth P49,985 was salvaged.
7. Mrs. del Rosario was able to settle everything except the
account of Lopez.

Page 14 of 154
INSURANCE G01 CASE DIGESTS

the Del Rosario the sum of P81,093.65, with interest at 6 per cent
Issue: Whether or not Mrs. Del Rosario acted as an agent of Lopez per annum from May 13, 1921, until paid.
in taking out an insurance on the contents of the warehouse

Held: YES. The agency can be deduced from the warehouse receipts,
the insurance policies, and the circumstances surrounding the 76. DEVELOPMENT BANK V, INTERMEDIATE APPELLATE COURT, 143
transaction. The law is that a policy effected by bailee and covering SCRA 62 (1986)
by its terms his own property and property held in trust; inures, in
the event of a loss, equally and proportionately to the benefit of all
the owners of the property insured. Even if one secured insurance Development Insurance vs IAC
covering his own goods and goods stored with him, and even if the GR No. 71360 July 16, 1986
owner of the stored goods did not request or know of the
insurance, and did not ratify it before the payment of the loss, yet it Facts:
has been held by a reputable court that the warehouseman is liable  A fire occurred in the building of Philippine Union Realty
to the owner of such stored goods for his share. Development Corporation (PURDC) and it sued for damages
from Devt Insurance based on an insurance contract. Devt
Insurance failed to answer on time and was declared in
default by the trial court. A judgment of default was
Moreover, the Court found in two documents of Mrs. Del Rosario subsequently rendered on the strength of the evidence
against the insurance companies (agreement for arbitration and the submitted ex parte by PURDC, which was allowed full
statement of claim)— she acknowledged her responsibility to the recovery of its claim.
owners of the stored merchandise, against risk of loss by fire. The  Devt Insuracne moved to lift the order of default, invoking
award of the arbitrators covered not alone Mrs. Del Rosario's excusable neglect, and to vacate the judgment by default;
warehouse but the products stored in the warehouse by Lopez and which was denied by the court. The IAC affirmed the
others. decision of the trial court.
 The face value of the policy is P2,500,000. Devt Insurance is
claiming that since at the time of the fire, the building
insured was worth P5.8M, they can only be liable to the
Lopez' rights to the insurance money have not been forfeited by extent of the proportion between the difference between
failure to pay the insurance provided for in the warehouse receipts. that amount and the face value, as against the total loss
A preponderance of the proof does not demonstrate that he ever sustained, which is P508,867; making them only liable for
ordered the cancellation of his insurance with the Del Rosario. Nor only P67,629.31.
is it shown that the Lopez ever refused to pay the insurance when
the bills were presented to him, and that notice of an intention to
cancel the insurance was ever given to him. Lopez can recover from Issue:
Page 15 of 154
INSURANCE G01 CASE DIGESTS

Whether the insurer should be liable for the whole amount of the 77. TEAL MOTOR V . ORIENT INSURANCE , 59 PHIL. 809
loss Facts:

Ruling: -These seven cases related to insurance policies covering the goods,
The insurer is liable for the whole amount.
wares, and merchandise contained in the building in the Port Area
 The policy issued to PURDC is an open policy and is subject
to the express condition that in the event of loss, whether in the City of Manila which was damaged by a fire of unknown origin
total or partial, it is understood that the amount of the loss the afternoon of Sunday, January 6, 1929.
shall be subject to appraisal and the liability of the
company, if established, shall be limited to the actual loss, -At the request of the insured, the companies gave additional time
subject to the applicable terms, conditions, warranties and for the filing of the claims of loss. These claims were definitely
clauses of the policy, and in no case shall exceed the rejected in writing by the insurance companies through their agents
amount of the policy. on April 15, 1929.
o An open policy is one in which the value of the thing
insured is not agreed upon but is left to be
- Among the special defenses of the insurance companies is one
ascertained in case of loss
 There is no evidence on record that the building was worth based upon a clause in the policies which, with the exception of
P5.8M at the time of the loss; only Devt Insurance says so, those of the Atlas Assurance Company, Ltd., among other things
and it does not back up its self-serving estimate with any provides:
independent corroboration.
 Since the building was insured at P2.5M, this must be “if the claim be made and rejected, and action or suit be not
considered the value of the building on the day the fire commenced within three months after such rejection, ...
occurred.
all under this Policy shall be forfeited.”
 The actual loss has been ascertained in this case and the
Court will respect such factual determination in the absence
- While those cases were under advisement here, the Supreme
of proof that it was arrived at arbitrarily. There is no such
showing. Court noticed that the provision relating to the Atlas policy reads:
 Applying the open policy clause as expressly agreed upon by
the parties in their contract, the Court holds that PURDC is “if the claim be made and rejected and arbitration
entitled to the payment of indemnity in the total amount of proceedings be not commenced in pursuance of the 18th
P508,867. Condition of this Policy within three months after such
rejection; all benefit under this Policy shall be forfeited.”

- No such arbitration proceedings were instituted within the three


months' period.
Page 16 of 154
INSURANCE G01 CASE DIGESTS

- The seven suits were filed between the 3rd and the 15th day of 78. ANG V. FULTON FIRE INSURANCE , 2 SCRA 945 (1961)
August, 1929, or more than three months after the rejection by the
defendant companies of plaintiff's claim.
G.R. No. L-15862 July 31, 1961
Issue:
PAULO ANG and SALLY C. ANG, plaintiffs-appellees,
Whether or NOT the claims were filed on time? vs.
FULTON FIRE INSURANCE CO., ET AL., defendants.
FULTON FIRE INSURANCE CO., defendant-appellant.
Ruling:
LABRADOR, J.:
The Supreme Court held that the case was filed out of time. Plaintiff
was given such time as it deemed necessary to formulate and FACTS
present its claim of loss. That claim was investigated by the
adjusters for several months, and under the contract of insurance,  September 9, 1953 - Fulton Fire Insurance Company issued
the insured had three months after rejection in which to bring suit. a fire insurance policy in favor of P. & S Department Store
The issues were virtually joined on the presentation of the claims (owned by the Spouses Paulo Ang and Sally C. Ang) over
and their rejection by the companies in writing, and three months stocks of general merchandise, consisting principally of dry
goods, contained in a building occupied by the Angs at
thereafter is not an unreasonably short time to draft and file in
Laoag, Ilocos Norte. The premium is P500.00 annually.
court an appropriate complaint on a contract of fire insurance.
 September 31, 1954 – policy was renewed for another year.
Ratio:  December 17, 1954 - the store was destroyed by fire.
 December 30, 1954 - the Angs executed the first claim form
A provision requiring presentation of claim within three months together with all the necessary papers (books of accounts of
after the fire, and the bringing of action within three months after the insured for the year 1953-1954 and a clearance from
refusal of claim is valid. the Philippine Constabulary and the police), and they were
all forwarded to the Manila Adjustment Company, Fulton's
adjusters.
 January 13, 1955 - Paulo Ang and 10 others were charged
for arson in a Criminal Case but Paulo Ang was eventually
acquitted.
 April 6, 1956 - Fulton denied the claim.

Page 17 of 154
INSURANCE G01 CASE DIGESTS

 April 19, 1956 - denial received by Angs. additional sum of P2,000.00 as attorney's fees, and costs).
The CFI held that the bringing of the action May 11, 1956,
 st
May 11, 1956 – Angs filed 1 case to assert the claim but
tolled the running of the 12 month period.Fulton appealed
against Paramount Surety and Insurance Company (Fulton’s
directly to the Supreme Court.
agent).
 September 3, 1957 - 1st case was dismissed without ISSUE
prejudice.
Whether the Angs may validly claim on the policy even with the
 May 5, 1958 – Angs filed 2nd case against Fulton and
prohibition on Paragraph 13 of the policy.
Paramount Surety to recover from them the face value of
the policy, but Paramount was eventually dropped from the
complaint. SC HELD

 May 26, 1958 – ANSWER OF FULTON: (1) denied that the NO.
loss by the fire was accidental, alleging that it was
occasioned by the willful act of Paulo Ang himself. The basic error committed by the trial court is its view that the filing
of the action against the agent of the defendant company was
(2) claimed that under paragraph "merely a procedural mistake of no significance or consequence,
13 of the policy, if the loss or damage is which may be overlooked." The condition contained in the
occasioned by the willful act of the insurance policy that claims must be presented within one year
insured, or if the claim is made and rejected after rejection is not merely a procedural requirement. The
but no action is commenced within 12 condition is an important matter, essential to a prompt settlement
months after such rejection, all benefits of claims against insurance companies, as it demands that insurance
under the policy would be forfeited. Since suits be brought by the insured while the evidence as to the origin
Angs received notice of denial on April 18, and cause of destruction have not yet disappeared. It is in the
1956, and they filed action only on May 5, nature of a condition precedent to the liability of the insurer, or in
1958, all the benefits under the policy have other terms, a resolutory cause, the purpose of which is to
been forfeited. terminate all liabilities in case the action is not filed by the insured
within the period stipulated.
 February 12, 1959 – REPLY OF ANGS: The 1st case was filed
May 11, 1956 but was dismissed without prejudice on The bringing of the action against the Paramount Surety &
September 3, 1957. That period between May 11, 1956 to Insurance Company, the agent of the defendant Company cannot
September 3, 1957 must be deducted from the prescriptive have any legal effect except that of notifying the agent of the claim.
period of 12 months. Beyond such notification, the filing of the action can serve no other
 CFI – 2nd case in favor of Angs (ordering Fulton to pay the purpose. There is no law giving any effect to such action upon the
Angs the sum of P10,000.00, with interest, and an principal. Besides, there is no condition in the policy that the action

Page 18 of 154
INSURANCE G01 CASE DIGESTS

must be filed against the agent, and this Court can not by 79. SUN I NSURANCE OFFICE V . CA, 195 SCRA 193 (1991)
interpretation, extend the clear scope of the agreement beyond
what is agreed upon by the parties. FACTS:

The case of E. Macias & Co. vs. China Fire Insurance Co. has settled Emilio Tan took from Sun Insurance Office a P300,000.00 property
the issue presented by the appellees in the case at bar definitely
insurance policy to cover his interest in the electrical supply store of
against their claim. In that case, We declared that the contractual
station in an insurance policy prevails over the statutory his brother. Four days after the issuance of the policy, the building
limitation, as well as over the exceptions to the statutory limitations was burned including the insured store. On August 20, 1983, Tan
that the contract necessarily supersedes the statute (of limitations) filed his claim for fire loss with Sun Insurance Office, but on
and the limitation is in all phases governed by the former. (E. Macias February 29, 1984, Sun Insurance Office wrote Tan denying the
& Co. vs. China Fire Insurance & Co., 46 Phil. pp. 345-353). As stated latter’s claim. On April 3, 1984, Tan wrote Sun Insurance Office,
in said case and in accordance with the decision of the Supreme seeking reconsideration of the denial of his claim. Sun Insurance
Court of the United States in Riddlesbarger vs. Hartford Fire
Office answered the letter, advising Tan’s counsel that the Insurer’s
Insurance Co. (7 Wall., 386), the rights of the parties flow from the
contract of insurance, hence they are not bound by the statute of denial of Tan’s claim remained unchanged.
limitations nor by exemptions thereto. In the words of our own
law, their contract is the law between the parties, and their ISSUES:
agreement that an action on a claim denied by the insurer must be
brought within one year from the denial, governs, not the rules on (1)WON the filing of a motion for reconsideration interrupts the 12
the prescription of actions. months prescriptive period to contest the denial of the insurance
claim; and (2)WON the rejection of the claim shall be deemed final
The judgment appealed from is hereby set aside and the case
only of it contains words to the effect that the denial is final;
dismissed, with costs against the plaintiffs-appellees.

APPEAL GRANTED. HELD:

(1) No. In this case, Condition 27 of the Insurance Policy of the


parties reads:

27. Action or suit clause - If a claim be made and rejected and an


action or suit be not commenced either in the Insurance
Commission or in any court of competent jurisdiction within twelve
(12) months from receipt of notice of such rejection, or in case of
arbitration taking place as provided herein, within twelve (12)
Page 19 of 154
INSURANCE G01 CASE DIGESTS

months after due notice of the award made by the arbitrator or It also begs to ask, when does the cause of action accrue? The
arbitrators or umpire, then the claim shall for all purposes be insured’s cause of action or his right to file a claim either in the
deemed to have been abandoned and shall not thereafter be Insurance Commission or in a court of competent jurisdiction
recoverable hereunder. commences from the time of the denial of his claim by the Insurer,
either expressly or impliedly. But the rejection referred to should be
As the terms are very clear and free from any doubt or ambiguity construed as the rejection in the first instance (i.e. at the first
whatsoever, it must be taken and understood in its plain, ordinary occasion or for the first time), not rejection conveyed in a resolution
and popular sense. of a petition for reconsideration. Thus, to allow the filing of a
motion for reconsideration to suspend the running of the
Tan, in his letter addressed to Sun Insurance Office dated April 3, prescriptive period of twelve months, a whole new body of rules on
1984, admitted that he received a copy of the letter of rejection on the matter should be promulgated so as to avoid any conflict that
April 2, 1984. Thus, the 12-month prescriptive period started to run may be brought by it, such as:
from the said date of April 2, 1984, for such is the plain meaning and
intention of Section 27 of the insurance policy. a.whether the mere filing of a plea for reconsideration of a denial is
sufficient or must it be supported by arguments/affidavits/material
The condition contained in an insurance policy that claims must be evidence;
presented within one year after rejection is not merely a procedural
requirement but an important matter essential to a prompt b.how many petitions for reconsideration should be permitted?
settlement of claims against insurance companies as it demands
that insurance suits be brought by the insured while the evidence as (2) No. The Eagle Star case cited by Tan to defend his theory that
to the origin and cause of destruction have not yet disappeared. the rejection of the claim shall be deemed final only of it contains
words to the effect that the denial is final is inapplicable in the
It is apparent that Section 27 of the insurance policy was stipulated instant case. Final rejection or denial cannot be taken to mean the
pursuant to Section 63 of the Insurance Code, which states that: rejection of a petition for reconsideration. The Insurance policy in
the Eagle Star case provides that the insured should file his claim,
Sec. 63. A condition, stipulation or agreement in any policy of first, with the carrier and then with the insurer. The final rejection
insurance, limiting the time for commencing an action thereunder being referred to in said case is the rejection by the insurance
to a period of less than one year from the time when the cause of company.
action accrues, is void.

Page 20 of 154
INSURANCE G01 CASE DIGESTS

80. PACIFIC BANKING CORP. V CA, 168 SCRA 1 (1988) then made an excuse that the insured had not filed any claim
with it, nor submitted proof of loss which is a clear violation of
Policy Condition No.11, as a result, determination of the liability
PACIFIC BANKING CORPORATION vs. CA of private respondent could not be made.

No. L-41014; November 29, 1988


 Pacific Banking filed in the trial court an action for a sum of
money for P61,000.00 against Oriental Assurance. At the trial,
Pacific Bank presented communications of the insurance
Facts: adjuster to Asian Surety revealing undeclared co-insurances with
the following: P30,000 with Wellington Insurance; P25,000 with
 On October 21, 1963, an Open Fire Policy was issued to the Empire Surety and P250,000 with Asian Surety undertaken by
Paramount Shirt Manufacturing Co. (insured) by which private insured Paramount on the same property covered by its policy
respondent Oriental Assurance bound itself to indemnify the with Oriental whereas the only co-insurances declared in the
insured for any los or damage, not exceeding P61,000.00, caused subject policy are those of P30,000.00 with Malayan P50,000.00
by fire to its property consisting of stocks, materials and supplies with South Sea and P25.000.00 with Victory.
usual to a shirt factory while contained in the first to third floors
of the building where they are located for a period of one year
starting October 21, 1964.  The defense of fraud, in the form of non-declaration of co-
insurances which was not pleaded in the answer, was also not
pleaded in the Motion to Dismiss. The trial court denied the
 At the time the policy was issued, Paramount Shirt was a debtor respondent’s motion. Oriental filed another motion to include
of Pacific Bank amounting to P800,000.00. Goods in the additional evidence of the co-insurance which could amount to
said policy were held in trust by Paramount for Pacific Bank fraud. The trial court rendered judgment making Oriental
under trust receipts. Said policy was duly endorsed to the Assurance liable for P61,000.00, but the Court of Appeals
petitioner bank as mortgagee/trustor of the properties insured, reversed the RTC decision.
with the knowledge and consent of private respondent to the
effect that “loss if any under this policy is payable to the (Pacific
Bank)". Issues:

1. Whether or not the unrevealed co-insurances violated policy


 While the aforesaid policy was in full fore and effect, a fire broke conditions no. 3?
out on the subject premises destroying the goods contained in 2. Whether or not the insured failed to file the required proof of
its ground and second floors. Pacific Bank sent a letter of loss prior to court action?
demand to Oriental Assurance for indemnity, but the latter
wasn’t ready to give since it was awaiting the adjuster’s report. It
Page 21 of 154
INSURANCE G01 CASE DIGESTS

Held and Ratio: The evidence adduced shows that twenty-four (24) days after
the fire, petitioner merely wrote letters to private respondent to
1. YES. Policy Condition 3 provides that the insured must give serve as a notice of loss, thereafter, the former did not furnish
notice of any insurance already in effect or subsequently be in
the latter whatever pertinent documents were necessary to
effect covering same property being insured. Failure to do so,
the policy shall be forfeited. Failure to reveal before the loss of prove and estimate its loss. Instead, petitioner shifted upon
the 3 other insurances is a clear misrepresentation or a false private respondent the burden of fishing out the necessary
declaration. The material fact was asked for but was not information to ascertain the particular account of the articles
revealed. Representations of facts are the foundations of the destroyed by fire as well as the amount of loss. It is noteworthy
contract. Pacific itself provided for the evidences in trial court that private respondent and its adjuster notified petitioner that
that proved existence of misrepresentation.
insured had not yet filed a written claim nor submitted the
supporting documents in compliance with the requirements set
2. YES. Generally, the cause of action on the policy accrues when forth in the policy. Despite the notice, the latter remained
the loss occurs, But when the policy provides that no action shall unheedful.
be brought unless the claim is first presented extrajudicially in
the manner provided in the policy, the cause of action will accrue
from the time the insurer finally rejects the claim for payment.
Since the required claim by insured, together with the
preliminary submittal of relevant documents had not been
In the case at bar, policy condition No. 11 specifically provides complied with, it follows that private respondent could not be
that the insured shall on the happening of any loss or damage deemed to have finally rejected petitioner's claim and therefore
give notice to the company and shall within fifteen (15) days the latter's cause of action had not yet arisen. Compliance with
after such loss or damage deliver to the private respondent (a) a condition No. 11 is a requirement sine qua non to the right to
claim in writing giving particular account as to the articles or maintain an action as prior thereto no violation of petitioner's
goods destroyed and the amount of the loss or damage and (b) right can be attributable to private respondent. This is so, as
particulars of all other insurances, if any. Likewise, insured was before such final rejection, there was no real necessity for
required "at his own expense to produce, procure and give to bringing suit. Petitioner should have endeavored to file the
the company all such further particulars, plans, specifications, formal claim and procure all the documents, papers, inventory
books, vouchers, invoices, duplicates or copies thereof, needed by private respondent or its adjuster to ascertain the
documents, proofs and information with respect to the claim". amount of loss and after compliance await the final rejection of
its claim. Indeed, the law does not encourage unnecessary
litigation.

Page 22 of 154
INSURANCE G01 CASE DIGESTS

81. TRAVELLERS INSURANCE V . CA, 272 SCRA 536 (1997) can sue the insurer. Where the contract is for indemnity against
TRAVELLERS INSURANCE & SURETY CORP. v. CA (MENDOZA) actual loss or payment, then third persons cannot proceed against
the insurer, the contract being solely to reimburse the insured for
272 SCRA 536 liability actually discharged by him thru payment to third persons,
HERMOSISIMA, JR; May 22, 1997 said third persons' recourse being thus limited to the insured alone.
But in the case at bar, there was no contract shown. What then was
NATURE the basis of the RTC and the CA to say that the insurance contract
was a third-party liability insurance policy? Consequently, the trial
The petition herein seeks the review and reversal of the decision of
court was confused as it did not distinguish between the private
respondent Court of Appeals affirming in toto the judgment of the
respondent's cause of action against the owner and the driver of
Regional Trial Court in an action for damages filed by private
the Lady Love taxicab and his cause of action against petitioner. The
respondent Vicente Mendoza, Jr. as heir of his mother who was
former is based on torts and quasi-delicts while the latter is based
killed in a vehicular accident.
on contract.
FACTS

An old lady was hit by a taxicab. The taxicab was later identified and
Even assuming arguendo that there was such a contract,
a case was filed against the driver and owner. Later, an amendment
private respondent's cause of action can not prevail because he
was filed to include the insurance company. RTC and CA ordered
failed to file the written claim mandated by the Insurance Code
that the owner, driver as well as the insurance company be held
(before it was amended-action must be brought within six months
solidarily liable.
from date of the accident (this is what’s applicable here) ; after
amendment- "action or suit for recovery of damage due to loss or
injury must be brought in proper cases, with the Commissioner or
ISSUE the Courts within one year from denial of the claim, otherwise the
claimant's right of action shall prescribe" ). He is deemed, under this
WON RTC and CA erred
legal provision, to have waived his rights as against petitioner-
HELD insurer.

YES

Where the contract provides for indemnity against liability


to third persons, then third persons to whom the insured is liable

Page 23 of 154
INSURANCE G01 CASE DIGESTS

82. LOPEZ V . COMPANIA DE SEGUROS, 16 SCRA 855 (1966) property previously been declined insurance by another
Lopez vs. Filipinas Compañia de Seguros company.
 May 27, 1960, the plaintiff filed with the Office of the
G.R. No. L-19613 April 30, 1966 Insurance Commissioner a complaint against the said
company.
 As suggested, the plaintiff was willing to submit his claim to
arbitration but was contested by the defendant since "the
FACTS: claim of the plaintiff cannot be resolved by arbitration, as
recourse to arbitration referred to in the policy contract,
 Plaintiff applied with the defendant company for the envisioned only differences or disputes, 'with respect to the
insurance of his properties: Biederman truck tractor and a amount of the company's liability,' and not to cases where
Winter Weils trailer from loss or damage in the amount of the company does not admit its liability to the insured.
P20,000.00 and P10,000.00, respectively.  With this rejection, the plaintiff filed his complaint with the
 During the application, the defendant company inquired of CFI of Manila on September 19,1961.
the plaintiff the ff:  Against the above complaint, the defendant-appellee filed
 Has any company in respect of the insurance of any on September 29, 1961 a motion to dismiss on the ground
car or vehicle of prescription. The latter argued that the plaintiff's claim
 (A) declined, cancelled or refused to renew your had already prescribed since it was not filed within twelve
insurance? months from its rejection by the insurance company as
 (B) increased your premium renewal? stipulated under paragraph 9 of the General Conditions of
 Plaintiff answered in negative but the truth was that the Commercial Vehicle Comprehensive Policy Nos. 5598 and
American International Underwriters of the Philippines 5599, to wit:
(AIU) had already declined similar application for insurance
by the plaintiff with respect of the above-mentioned
vehicles. If a claim be made and rejected and an action or suit be not
 The defendant issued to the plaintiff two Commercial commenced within twelve months after such rejection or (in case of
Vehicle Comprehensive Policies covering the said an arbitration taking place as provided herein) within twelve
properties.
months after the arbitrator, arbitrators, or umpire shall have made
 The vehicles mentioned figured in an accident resulting in
the total loss of the tractor and partial damage to the their award then the claim shall for all purposes be deemed to have
trailer. Plaintiff demand upon the defendant for the been abandoned and shall not thereafter be recovered hereunder.
payment to him the total amt. of damages resulting from
the accident.
 On April 28, 1960, defendant rejected the claim on the
ground of concealment of a material fact: that the insured

Page 24 of 154
INSURANCE G01 CASE DIGESTS

ISSUE: Whether the complaint filed by the plaintiff-appellant with justice, the claim may not properly be categorized under either
the Office of the Insurance Comm. on May 27,1960 a term.
commencement of an "action or suit" within the meaning and
intent of general condition? No.
 An "action or suit" is essentially "for the enforcement or
protection of a right, or the prevention or redress of a
RATIO: wrong." (Rule 2, Sec. 1, Rules of Court). There is nothing in
the Insurance Law, which empowers the Insurance
 "Action" and "suit": Commissioner to adjudicate on disputes relating to an
insurance company's liability to an insured under a policy
issued by the former to the latter. The validity of an
Rule 2, Section 1 of the Rules of Court insured's claim under a specific policy, its amount, and all
such other matters as might involve the interpretation and
Section 1. Action defined.—Action means an ordinary suit in a Court construction of the insurance policy, are issues which only a
of Justice by which one party prosecutes another for the regular court of justice may resolve and settle.
enforcement or protection of a right, or the prevention or redress of Consequently, the complaint filed by the appellant herein
with the Office of the Insurance Commission could not have
a wrong. (Emphasis supplied.)
been an "action or suit."

Jurisprudence

Suit is the prosecution or pursuit of some claim or demand in a


court of justice or any proceeding in a court of justice in which a
plaintiff pursues his remedy to recover a right or claim. (Emphasis
supplied.)

- Upon the authorities, therefore, it is settled that the terms


"action" and "suit" are synonymous. Moreover, it is clear that the
determinative or operative fact which converts a claim into an
"action or suit" is the filing of the same with a "court or justice."
Filed elsewhere, as with some other body or office not a court of
Page 25 of 154
INSURANCE G01 CASE DIGESTS

83. ACCFA V. ALPHA INSURANCE , 24 SCRA 151 (1968) from the making of a claim for the loss upon which the action is
FACTS based, is valid or void? VOID

1. Alpha insurance issued 5,000 pesos bond to guarantee FACOMA HELD


against loss on account of personal dishonesty, amounting to
larceny or estafa of its Secretary Treasurer. Consequently, the condition of the bond in question, limiting the
period for bringing action thereon, is subject to the provisions of
2. FACOMA assigned its right to ACCFA, with approval of principal Section 61-A of the Insurance Act (No. 2427), as amended by Act
and surety 4101 of the pre-Commonwealth Philippine Legislature, prescribing
that —
3. During effectivity of bond, Secretary Treasurer converted and
misappropriated to his personal benefit 11,000 of FACOMA funds, SEC. 61-A — A condition, stipulation or agreement in any policy of
which 6,307.33 belonged to ACCFA. insurance, limiting the time for commencing an action thereunder
to a period of less than one year from the time when the cause of
4. Upon discovery of the loss, ACCFA immediately notified the action accrues is void.
surety company, but despite repeated demands, surety refused to
pay The cause of action does not accrue until the party obligated
refuses, expressly or impliedly, to comply with its duty (in this case,
5. Alpha insurance moved to dismiss the complaint for failure to to pay the amount of the bond). The year for instituting action in
state a cause of action, giving as reason that the same was filed court must be reckoned, therefore, from the time of appellee's
more than one year after plaintiff made claim for loss, contrary to refusal to comply with its bond; it can not be counted from the
the eighth condition of the bond creditor's filing of the claim of loss, for that does not import that the
surety company will refuse to pay.
6. Granted and motion to dismiss upon reconsideration kaya nag-
appeal si ACCFA. In so far, therefore, as condition eight of the bond requires action to
be filed within one year from the filing of the claim for loss, such
stipulation contradicts the public policy expressed in Section 61-A
ISSUE of the Philippine Insurance Act. Condition eight of the bond,
therefore, is null and void, and the appellant is not bound to comply
Whether or not the provision of a fidelity bond that no action shall with its provisions.
be had or maintained thereon unless commenced within one year
Page 26 of 154
INSURANCE G01 CASE DIGESTS

84. SAURA I MPORT & EXPORT V . PHIL. INT’L SURETY – 8 SCRA 143 7. The building and its contents, worth P40,685.69 were
(1963) burned. Saura filed a claim with the Insurer and mortgagee
Bank.
8. Upon the presentation of notice of loss with the PNB, Saura
learned for the first time that the policy had previously
G.R. No. L-15184, May 31, 1963
been cancelled by the insurer, when Saura's folder in the
Bank's filed was opened and the notice of cancellation
PAREDES, J.
(original and duplicate) sent by the Insurer to the Bank, was
found.
Topic: Cancellation of non-life policy – Sec. 62
9. Upon refusal of the Insurer Philippine International Surety
to pay the amount of the insurance this present case filed
FACTS:
with the Manila CFI against the Insurer, and the PNB was
later included as party defendant, after it had refused to
1. Saura Import & Export Co Inc., mortgaged to PNB, a parcel prosecute the case jointly with Saura Import & Export Co.,
of land, to secure the payment of promissory note of P27, Inc.
000.00.
2. The mortgage was amended to guarantee an increased ISSUE:
amount, bringing the total mortgaged debt to P37, 000.00.
3. Erected on the land mortgaged, was a building of strong
Whether or not there is a valid cancellation of the fire insurance
materials owned by the mortgagor Saura Import & Export
policy.
Co., Inc., which had always been covered by insurance,
many years prior to the mortgage contract.
HELD: NO
4. Pursuant to the requirement, Saura insured the building and
its contents with the Philippine International Surety for
RATIO DECIDENDI:
P29,000.00 against fire for the period of one year from
October 2, 1954.
The policy in question does not provide for the notice, its form
5. The insurance policy was endorsed to the mortgagee PNB,
or period. The Insurance Law, Act No. 2427, does not likewise
in a Memo which states — Loss if any, payable to the
provide for such notice. This being the case, it devolves upon the
Philippine National Bank as their interest may appear,
Court to apply the generally accepted principles of insurance,
subject to the terms, conditions and warranties of this
regarding cancellation of the insurance policy by the insurer.
policy.
6. Barely thirteen (13) days after the issuance of the policy, the
insurer cancelled the same. Notice of the cancellation was From what has been stated, actual notice of cancellation in
given to appellee bank in writing. . a clear and unequivocal manner, preferably in writing, in view of
the importance of an insurance contract, should be given by the
insurer to the insured, so that the latter might be given an

Page 27 of 154
INSURANCE G01 CASE DIGESTS

opportunity to obtain other insurance for his own protection. The 85. MALAYAN INSURANCE V . CRUZ-ARNALDO , 154 SCRA 672 (1987)
notice should be personal to the insured and not to and/or
through any unauthorized person by the policy.
Topic: Cancellation of non-life policy
In the case at bar, the defendant insurance company, must
have realized the paramount importance of sending a notice of Facts:
cancellation, when it sent the notice of cancellation of the policy to
the defendant bank (as mortgagee), but not to the insured with  In 1981, petitioner Malayan issued to the private
which it (insurance company) had direct dealing. It was the primary respondent Pinca, a Fire Insurance Policy on her property
duty of the defendant-appellee insurance company to notify the for P14,000 effective July 22, 1981 untul July 22, 1982
insured, but it did not. It should be stated that the house and its  Malayan allegedly cancelled the policy for non-payment of
contents were burned on April 6, 1955, at the time when the policy premiums and sent notice to Pinca (October 15, 1981)
was enforced (October 2, 1954 to October 2, 1955); and that under  A couple of months after, payment of the premium for
the facts, as found by the trial court, to which We are bound, it is Pinca was received by Adora, an agent of Malayan
evident that both the insurance company and the appellee bank (December 24, 1981)
failed, wittingly or unwittingly, to notify the insured appellant Saura  Adora remitted the payment to Malayan
of the cancellation made.  Three days after, Pinca’s property was completely burned
(Jan 18, 1982)
The defendant insurance company contends that it gave  A couple of weeks after, Malayan returned Pinca’s payment
notice to the defendant-appellee bank as mortgagee of the as the policy was previously cancelled
property, and that was already a substantial compliance with its  Pinca made demands for the proceeds of the policy, but
duty to notify the insured of the cancellation of the policy. But Malayan refused. The Insurance Commission decided in
notice to the bank, as far appellant herein is concerned, is not favor of Pinca.
effective notice.  Malayan: There was no payment of premium and that the
policy had been canceled before the occurrence of the loss

Issue: Whether or not Malayan should be liable for the proceeds of


the policy

Held: YES. Malayan’s argument was not acceptable.

Malayan relies on Sec 77 of the Insurance Code:

Page 28 of 154
INSURANCE G01 CASE DIGESTS

SEC. 77. An insurer is entitled to payment of the premium as Adora at her own risk as she was bound to first check his authority
soon as the thing is exposed to the peril insured against. to receive it.
Notwithstanding any agreement to the contrary, no policy
or contract of insurance issued by an insurance company is
valid and binding unless and until the premium thereof has MALAYAN is taking an inconsistent stand. While contending that
been paid, except in the case of a life or an industrial life acceptance of the premium payment was prohibited by the policy, it
policy whenever the grace period provision applies. at the same time insists that the policy never came into force
The above provision is not applicable because payment of the because the premium had not been paid. One surely, cannot have
premium was in fact eventually made. The premium invoice issued his cake and eat it too.
to Pinca at the time of delivery of the policy on June 7, 1981 was
We do not share MALAYAN's view that there was no existing
stamped, “Payment Received” of the amount of P930.60 on “12-24-
insurance at the time of the loss sustained by Pinca because her
81” by Adora. It suggests an understanding between Malayan and
policy never became effective for non-payment of premium.
the insured that payment could be made later.
Payment was in fact made, rendering the policy operative as of June
22, 1981, and removing it from the provisions of Article 77,
Thereafter, the policy could be cancelled on any of the supervening
The payment was made on December 24, 1981, and the fire grounds enumerated in Article 64 (except "nonpayment of
occured on January 18, 1982. One wonders: suppose the payment premium") provided the cancellation was made in accordance
had been made and accepted in, say, August 1981, would the therewith and with Article 65.
commencement date of the policy have been changed to the date
of the payment, or would the payment have retroacted to July 22,
1981? If MALAYAN accepted the payment in December 1981 and
Section 64 reads as follows:
the insured property had not been burned, would that policy not
have expired just the same on July 22, 1982, pursuant to its original
SEC. 64. No policy of insurance other than life shall be
terms, and not on December 24, 1982?
cancelled by the insurer except upon prior notice thereof to
the insured, and no notice of cancellation shall be effective
There is the petitioner's argument, however, that Adora was not
unless it is based on the occurrence, after the effective date
authorized to accept the premium payment because six months had
of the policy, of one or more of the following:
elapsed since the issuance by the policy itself. It is argued that this
(a) non-payment of premium;
prohibition was binding upon Pinca, who made the payment to
(b) conviction of a crime arising out of acts increasing the
hazard insured against;
Page 29 of 154
INSURANCE G01 CASE DIGESTS

(c) discovery of fraud or material misrepresentation; All MALAYAN's offers to show that the cancellation was
(d) discovery of willful, or reckless acts or commissions communicated to the insured is its employee's testimony that the
increasing the hazard insured against; said cancellation was sent "by mail through our mailing section."
(e) physical changes in the property insured which result in without more
the property becoming uninsurable;or
(f) a determination by the Commissioner that the It stands to reason that if Pinca had really received the said notice,
continuation of the policy would violate or would place the she would not have made payment on the original policy on
insurer in violation of this Code. December 24, 1981. Instead, she would have asked for a new
insurance, effective on that date and until one year later, and so
taken advantage of the extended period. The Court finds that if she
As for the method of cancellation, Section 65 provides as follows:
did pay on that date, it was because she honestly believed that the
SEC. 65. All notices of cancellation mentioned in the policy issued on June 7, 1981, was still in effect and she was willing
preceding section shall be in writing, mailed or delivered to to make her payment retroact to July 22, 1981, its stipulated
the named insured at the address shown in the policy, and commencement date.
shall state (a) which of the grounds set forth in section sixty-
Adora, incidentally, had not been informed of the cancellation
four is relied upon and (b) that, upon written request of the
either and saw no reason not to accept the said payment
named insured, the insurer will furnish the facts on which
the cancellation is based.

A valid cancellation must, therefore, require concurrence of the


following conditions:

(1) There must be prior notice of cancellation to the insured;


(2) The notice must be based on the occurrence, after the effective
date of the policy, of one or more of the grounds mentioned;
(3) The notice must be (a) in writing, (b) mailed, or delivered to the
named insured, (c) at the address shown in the policy;
(4) It must state (a) which of the grounds mentioned in Section 64 is
relied upon and (b) that upon written request of the insured, the
insurer will furnish the facts on which the cancellation is based.

Page 30 of 154
INSURANCE G01 CASE DIGESTS

WARRANTIES ISSUE: Whether or not the Court of Appeals erred in its legal
interpretation of 'Fire ExtinguishingAppliances Warranty' of the
policy.
86. AMERICAN HOME INSURANCE V. TANTUCO ENTERPRISES, 366
SCRA 740 (2011)
HELD: In construing the words used descriptive of a building
G.R. No. 138941October 8, 2001 insured, the greatest liberality is shown bythe courts in giving effect
to the insurance. In view of the custom of insurance agents to
examinebuildings before writing policies upon them, and since a
FACTS: Respondent Tantuco Enterprises, Inc. is engaged in the mistake as to the identity and character of thebuilding is extremely
coconut oil milling and refining industry.It owns two oil mills which unlikely, the courts are inclined to consider that the policy of
were separately covered by fire insurance policies issued by insurance covers anybuilding which the parties manifestly intended
petitionerAmerican Home Assurance Co., Philippine Branch. to insure, however inaccurate the description may
be.Notwithstanding, therefore, the misdescription in the policy, it is
The first oil mill was insured for P3,000,000.00 underPolicy No. 306- beyond dispute, to our mind, that whatthe parties manifestly
7432324-3 for the period March 1, 1991 to 1992. The new oil mill intended to insure was the new oil mill. If the parties really intended
was insured forP6,000,000.00 under Policy No. 306-7432321-9 for to protectthe first oil mill,then there is no need to specify it as new .
the same term. Official receipts indicating paymentfor the full
amount of the premium were issued by the petitioner's agent.A fire In determining what the parties intended, the courts will read and
that broke out in the early morning of September 30,1991 gutted construe the policy as a whole and if possible, give effect to all the
and consumed the new oil mill.Respondent immediately notified parts of the contract, keeping in mind always, however, the prime
the petitioner of the incident but petitioner rejected rulethat in the event of doubt, this doubt is to be resolved against
respondent'sclaim for the insurance proceeds on the ground that no the insurer. In determining the intent of the parties to the contract,
policy was issued by it covering the burned oilmill. It stated that the the courts will consider the purpose and object of the contract.
description of the insured establishment referred to another
building thus: "Ourpolicy nos. 306-7432321-9 (Ps 6M) and 306-
7432324-4 (Ps 3M) extend insurance coverage to your oilmill under
Building No. 5, whilst the affected oil mill was under Building No.
14."

Page 31 of 154
INSURANCE G01 CASE DIGESTS

87. ANG GIOK V. SPRINGFIELD FIRE & MUTUAL INSURANCE , 56 PHIL. reached as high as 39. We place reliance on the consular invoices
375 (1931) and on the testimony of the adjuster, Herridge. As such only one
issue is to be resolved.
56 Phil. 375 (1931)
Warranties Issue: WON a warranty referred to in the policy as forming part of
the contract of insurance and in the form of a rider to the insurance
Facts: Ang Giok Chip doing business under the name and style of policy, is null and void because not complying with the Philippine
Hua Bee Kong Si was formerly the owner of a warehouse situated in Insurance Act?
Manila.
Held: No, the Philippine Law was taken verbatim from law of
The contents of the warehouse were insured with the three California. Thus, the court should follow in fundamental points, at
insurance companies for the total sum of P60,000. One insurance least, the construction placed by California courts on a California
policy, in the amount of P10,000, was taken out with the Springfield Law. The SC reached the definite conclusion that warranty F, a rider
Fire & Marine Insurance Company – the warranty includes a Rider: attached to the face of the insurance policy, and referred to in
no hazardous material be stored in the building, that the Insured be contract of insurance, is valid and sufficient under section 65 of the
permitted to stored a small quantity of the hazardous goods Insurance Act.
specified below, but not exceeding in all 3 per cent of the total
value of the whole of the goods or merchandise contained in said Ratio: Section 65 of the Insurance Act and its counterpart, section
warehouse. 265 of the Civil Code of California, will bear analysis as tested by
reason and authority. The law says that every express warranty
The warehouse was destroyed by fire on January 11, 1928, while must be "contained in the policy itself." The word "contained,"
the policy issued by the latter company was in force. The plaintiff according to the dictionaries, means "included," inclosed,"
instituted action in the Court of First Instance of Manila against the "embraced," "comprehended," etc. When, therefore, the courts
defendant to recover a proportional part of the loss coming to speak of a rider attached to the policy, and thus "embodied"
P8,170.59. Four special defenses were interposed on behalf of the therein, or of a warranty "incorporated" in the policy, it is believed
insurance company, one being planted on a violation of warranty F that the phrase "contained in the policy itself" must necessarily
fixing the amount of hazardous goods which might be stored in the include such rider and warranty.
insured building.
The SC thinks it wrong to hold that the California law represents a
The CFI rendered in favor of the plaintiff. But the SC held that it is radical departure from the basic principles governing the law of
unnecessary for us to discuss three of the four special defenses insurance. We turn to two of such well recognized doctrines. In the
which were made by the insurance company. We think, however, first place, it is well settled that a rider attached to a policy is a part
that it would be a reasonable deduction to conclude that more than of the contract, to the same extent and with like effect as it actually
3 per cent of the total value of the merchandise contained in the embodied therein. (I Couch, Cyclopedia of Insurance Law, sec. 159.)
warehouse constituted hazardous goods, and that this per cent In the second place, it is equally well settled that an express

Page 32 of 154
INSURANCE G01 CASE DIGESTS

warranty must appear upon the face of the policy, or be clearly 88. QUA CHEE GAN V. LAW UNION AND ROCK INSURANCE , 98 PHIL
incorporated therein and made a part thereof by explicit reference, 85 (1955)
or by words clearly evidencing such intention. (4 Couch, Cyclopedia
of Insurance Law, sec. 862.) QUA CHEE GAN vs LAW UNION AND ROCK INSURANCE represented
by agent, Warner, Barnes and Co., Ltd. (December 17, 1955)
Referring to the jurisprudence of California, another rule of
insurance adopted in that State is in point. It is admitted that the FACTS:
policy before us was accepted by the plaintiff. The receipt of this
policy by the insured without objection binds both the acceptor and  Before the last war, Qua Chee Gan owned four warehouses
the insured to the terms thereof. The insured may not thereafter be or bodegas in Tabaco, Albay, used for the storage of stocks
heard to say that he did not read the policy or know its terms, since of copra and of hemp, baled and loose, in which the he
it is his duty to read his policy and it will be assumed that he did so. dealth extensively. They had been insured with Law Union
and Rock Insurance since 1937, and the loss made payable
to the Philippine National Bank as mortgage of the hemp
and crops, to the extent of its interest.
 On June, 1940, the insurance stood as follows:
Policy No. Property Insured Amount
2637164 Bodega No. 1 (Building) P15,000.00
(Exhibit "LL")
2637165 Bodega No. 2 (Building) 10,000.00
(Exhibit "JJ") Bodega No. 3 (Building) 25,000.00
Bodega No. 4 (Building) 10,000.00
Hemp Press — moved by steam 5,000.00
engine
2637345 Merchandise contents (copra and 150,000.00
(Exhibit "X") empty sacks of Bodega No. 1)
2637346 Merchandise contents (hemp) of 150,000.00
(Exhibit "Y") Bodega No. 3
2637067 Merchandise contents (loose 5,000.00
(Exhibit "GG") hemp) of Bodega No. 4
Total
P370,000.00

Page 33 of 154
INSURANCE G01 CASE DIGESTS

 On July 21, 1940, fire broke out near the Bodegas and last protected, with not less than 100 feet of hose piping and
for almost one week which completely destroyed Bodegas nozzles for every two hydrants kept under cover in
1,2 and 4. convenient places, the hydrants being supplied with water
 Qua Chee Gan claims 398,562.81 (reduced to 370,000, the pressure by a pumping engine, or from some other source,
amount in the insurance). Fire adjusters conducted an
capable of discharging at the rate of not less than 200
investigation. Law Union resisted the payment claiming
violation of warranties and conditions, filing of fraudulent gallons of water per minute into the upper story of the
claims, and that the fire had been deliberately caused by highest building protected, and a trained brigade of not less
the insured/other persons in connivance. than 20 men to work the same.
 Qua Chee Gan and Qua Chee Pao (brother)were tried for
Arson but the trial court acquitted them. Since the bodegas had an external wall perimeter of 1,640
 Qua Chee Gan then filed a civil case to claim the proceeds of feet, there should have been 11 fire hydrants and Qua Chee
the fire insurance policies. CFI ruled in favour of him. PNB Gan only had 2. Another pair is available also but it belongs
filed a complaint in intervention but it was dismissed to the municipality of Tabaco.
because Qua Chee Gan managed to pay his indebtedness.
 Law Union is barred by estoppel because they know that
the number of hydrants demanded in the rider never
ISSUE: WON Qua Chee Gan breached provisions of warranty that he existed from the very beginning but they still issued the
cannot be entitled to claim insurance proceeds. policies in question subject to such warranty, and received
the corresponding premiums. It would be perilously close to
HELD: NO conniving at fraud upon the insured to allow Law Union to
claim now as void ab initio the policies that it had issued to
1st cause of action - Hydrants: NO the Qua Chee Gan without warning of their fatal defect, of
which it was informed, and after it had misled the
 Memo. of Warranty. — The undernoted Appliances for the defendant into believing that the policies were effective.
extinction of fire being kept on the premises insured hereby, Court finds it unusual also that during the period for
and it being declared and understood that there is an ample insurance, the insurer suddenly realized that the premises
and constant water supply with sufficient pressure available are hazardous and offered the rider warranty for a discount
at all seasons for the same, it is hereby warranted that the on the premium.
said appliances shall be maintained in efficient working
 Rule: Where the insurer, at the time of the issuance of a
order during the currency of this policy, by reason whereof a
policy of insurance, has knowledge of existing facts which, if
discount of 2 1/2 per cent is allowed on the premium
insisted on, would invalidate the contract from its very
chargeable under this policy.
inception, such knowledge constitutes a waiver of
Hydrants in the compound, not less in number than one for
conditions in the contract inconsistent with the facts, and
each 150 feet of external wall measurement of building, the insurer is estopped thereafter from asserting the breach
Page 34 of 154
INSURANCE G01 CASE DIGESTS

of such conditions. The law is charitable enough to assume, require. Anyway, it would be unreasonable to expect the
in the absence of any showing to the contrary, that an insured to maintain for his compound alone a fire fighting
insurance company intends to execute a valid contract in force that many municipalities in the Islands do not even
return for the premium received; and when the policy possess.
contains a condition which renders it voidable at its 2nd cause of action – Hemp Warranty: No
inception, and this result is known to the insurer, it will be
presumed to have intended to waive the conditions and to  Prohibition on Oils (animal and/or vegetable and/or mineral
execute a binding contract, rather than to have deceived and/or their liquid products having a flash point below 300
the insured into thinking he is insured when in fact he is degrees Fahrenheit – Law Union argues that Qua Chee Gan
not, and to have taken his money without consideration. admitted that he had 36 cans of gasoline in Bodega 2.
(29 Am. Jur., Insurance, section 807, at pp. 611-612.)  It is well to note that gasoline is not specifically mentioned
 Ratio: To allow a company to accept one's money for a among the prohibited articles listed in the so-called "hemp
policy of insurance which it then knows to be void and of no warranty." It is ambiguous and uncertain; for in ordinary
effect, though it knows as it must, that the assured believes parlance, "Oils" mean "lubricants" and not gasoline or
it to be valid and binding, is so contrary to the dictates of kerosene. And how many insured, it may well be wondered,
honesty and fair dealing, and so closely related to positive are in a position to understand or determine "flash point
fraud, as to the abhorent to fairminded men. It would be to below 300 Fahrenheit. Here, again, by reason of the
allow the company to treat the policy as valid long enough exclusive control of the insurance company over the terms
to get the preium on it, and leave it at liberty to repudiate it and phraseology of the contract, the ambiguity must be
the next moment. This cannot be deemed to be the real held strictly against the insurer and liberraly in favor of the
intention of the parties. To hold that a literal construction of insured, specially to avoid a forfeiture.
the policy expressed the true intention of the company  We see no reason why the prohibition of keeping gasoline
would be to indict it, for fraudulent purposes and designs in the premises could not be expressed clearly and
which we cannot believe it to be guilty of (Wilson vs. unmistakably, in the language and terms that the general
Commercial Union Assurance Co., 96 Atl. 540, 543-544). public can readily understand, without resort to obscure
 The alleged violation of the warranty of 100 feet of fire hose esoteric expression. If the company intended to rely upon a
for every two hydrants, must be equally rejected, since it is condition of that character, it ought to have been plainly
based on the assumption that the insured was bound to expressed in the policy.
maintain no less than eleven hydrants (one per 150 feet of  Another point that is in favor of the insured is that the
wall), which requirement appellant is estopped from gasoline kept in Bodega No. 2 was only incidental to his
enforcing. business, being no more than a customary 2 day's supply for
 As to maintenance of a trained fire brigade of 20 men, the the five or six motor vehicles used for transporting of the
record is preponderant that the same was organized, and stored merchandise. "It is well settled that the keeping of
drilled, from time to give, although not maintained as a inflammable oils on the premises though prohibited by the
permanently separate unit, which the warranty did not policy does not void it if such keeping is incidental to the

Page 35 of 154
INSURANCE G01 CASE DIGESTS

business." Bachrach vs. British American Ass. Co., 17 Phil. 89. PIONEER INSURANCE V. YAP, 61 SCRA 426 (1974)
555, 560); and "according to the weight of authority, even
though there are printed prohibitions against keeping GR L-36232; 61 SCRA 426, (1947)
certain articles on the insured premises the policy will not
be avoided by a violation of these prohibitions, if the Topic: Warranties: Effect Warranties
prohibited articles are necessary or in customary use in
Facts:
carrying on the trade or business conducted on the Olivia Yap owned a two-storey building in Manila where she sold
premises." (45 C. J. S., p. 311; also 4 Couch on Insurance, shopping bags and footwear. Her son-in-law Chua Soon Poon was in charge
section 966b). It should also be noted that the "Hemp of the store. She took a fire insurance of Php25,000 on the building with
Warranty" forbade storage only "in the building to which the petitioner to cover her stocks and furniture. One of the conditions in
this insurance applies and/or in any building communicating the policy stated that “the insured should notify the insurance company of
therewith", and it is undisputed that no gasoline was stored any insurance policies already effected or may be subsequently effected
in the burned bodegas, and that "Bodega No. 2" which was covering the property unless such notice be given and the particulars of
not burned and where the gasoline was found, stood such insurance or insurances be stated in, or endorsed on this Policy by or
isolated from the other insured bodegas. on behalf of the Company before the occurrence of any loss or damage, all
benefits under this Policy shall be forfeited… any false declarations will
render this policy null and void.”
At the time the petitioner issued the policy, Great American
Insurance issued another policy on the same properties of Yap. This
issuance was noted with the petitioner. Later in the year however, she
took out another policy with Federal Insurance covering the same
properties again, only this time she did not notify the petitioner.
In December 1962, a fire burned down the building. Yap filed an
insurance claim against the petitioner but the latter denied. The petitioner
claimed there was a breach of contract. Yap filed a complaint for payment
of the value of the insurance policy with the petitioner. Petitioner claimed
she is not entitled because (1) none of the properties covered by the policy
were destroyed by the fire, and (2) even if these properties were burned,
she is still not entitled for breach of contract. Petitioner claims the
proceeds were forfeited.
The RTC decided in favor of Yap. The CA affirmed.

Issue:
Should the petitioner be liable based on the fire insurance policy?

Held:
NO

Page 36 of 154
INSURANCE G01 CASE DIGESTS

Rationale: the effect that a clause in a policy to the effect that the
There was a violation by respondent Oliva Yap of the co-insurance clause procurement of additional insurance without the consent
contained in Policy No. 4219 that resulted in the avoidance of petitioner's of the insurer renders the policy void is a valid provision.
liability. The insurance policy for P20,000.00 issued by the Great American The earlier cases of Planters Mutual Insurance Co., vs.
Insurance Company covering the same properties of respondent Yap and Green, 72 Ark. 305, 80 S.W. 92, are to the same effect."
duly noted on Policy No. 4219 as c-insurance, ceased, by agreement of the And see Vance, Insurance, 2nd Ed., 725. (Reach vs.
parties (Exhibit "1-L"), to be recognized by them as a co-insurance policy. Arkansas Farmers Mut. Fire Ins. Co., [Ark. Nov. 14, 1949]
The Court of Appeals says that the Great American Insurance policy was 224 S. W. 2d 48, 49.)
substituted by the Federal Insurance policy for the same amount, and 2. Where a policy contains a clause providing that the
because it was a mere case of substitution, there was no necessity for its policy shall be void if insured has or shall procure any
endorsement on Policy No. 4219. This finding, as well as reasoning, suffers other insurance on the property, the procurement of
from several flaws. There is no evidence to establish and prove such a additional insurance without the consent of the insurer
substitution. If anything was substituted for the Great American Insurance avoids the policy." (Planters' Mut. Ins. Ass'n vs. Green
policy, it could only be the Northwest Insurance policy for the same [Supreme Court of Arkansas, March 19, 1904] 80 S.W.
amount of P20,000.00. The endorsement (Exhibit "1-K") quoted above 151.)
shows the clear intention of the parties to recognize on the date the The obvious purpose of the aforesaid requirement in the policy is to
endorsement was made (August 29, 1962), the existence of only one co- prevent over-insurance and thus avert the perpetration of fraud. The
insurance, and that is the Northwest Insurance policy, which according to public, as well as the insurer, is interested in preventing the situation in
the stipulation of the parties during the hearing, was issued on August 20, which a fire would be profitable to the insured. According to Justice Story:
1962 (t.s.n., January 12, 1965, pp. 3-4) and endorsed only on August 20, "The insured has no right to complain, for he assents to comply with all the
1962. The finding of the Court of Appeals that the Great American stipulation on his side, in order to entitle himself to the benefit of the
Insurance policy was substituted by the Federal Insurance policy is contract, which, upon reason or principle, he has no right to ask the court
unsubstantiated by the evidence of record and indeed contrary to said to dispense with the performance of his own part of the agreement, and
stipulation and admission of respondent, and is grounded entirely on yet to bind the other party to obligations, which, but for those stipulation
speculation, surmises or conjectures, hence, not binding on the Supreme would not have been entered into."
Court.
By the plain terms of the policy, other insurance without the consent of
petitioner would ipso facto avoid the contract. It required no affirmative
act of election on the part of the company to make operative the clause
avoiding the contract, wherever the specified conditions should occur. Its
obligations ceased, unless, being informed of the fact, it consented to the
additional insurance.
The validity of a clause in a fire insurance policy to the effect that the
procurement of additional insurance without the consent of the insurer
renders ipso facto the policy void is well-settled:
In Milwaukee Mechanids' Lumber Co., vs. Gibson, 199
Ark. 542, 134 S. W. 2d 521, 522, a substantially identical
clause was sustained and enforced, the court saying:
"The rule in this state and practically all of the states is to

Page 37 of 154
INSURANCE G01 CASE DIGESTS

90. PRUDENTIAL GUARANTEE V. TRANS A SIA SHIPPING, 491 SCRA 9. Prudential requested for the return of the 3M.
411 (2006) 10. TRANS-ASIA filed a Complaint5 for Sum of Money against
PRUDENTIAL with the RTC of Cebu City alleging that the 3M
Chico-Nazario, J. was the balance of the indemnity due.
Facts 11. Prudential answered saying that they wanted the 3M back
because Trans-Asia violated the warranty abovementioned.
1. TRANS-ASIA is the owner of the vessel M/V Asia Korea. 12. RTC- in favor of Prudential.
2. In consideration of payment of premiums, defendant a. Trans-Asia was required to maintain the vessel and
[PRUDENTIAL] insured M/V Asia Korea for loss/damage of found that Trans-Asia violated it.
the hull and machinery arising from perils, inter alia, of fire 13. CA reversed.
and explosion for the sum of P40 Million, beginning [from] a. Prudential was not able to prove that Trans-Asia
the period [of] July 1, 1993 up to July 1, 1994. violated the warranty.
3. This is evidenced by a Marine Policy. b. The rider stating the warranty clause was attached
4. On October 25, 1993, while the policy was in force, a fire without the intervention of Trans-Asia.
broke out while [M/V Asia Korea was] undergoing repairs at c. The loan was in effect a subrogation. 3M was partial
the port of Cebu. payment of the indemnity.
5. On October 26, 1993 plaintiff [TRANS-ASIA] filed its notice 14. Hence, this petition.
of claim for damage sustained by the vessel. This is
evidenced by a letter/formal claim of even date.
6. Plaintiff [TRANS-ASIA] reserved its right to subsequently
Issues
notify defendant [PRUDENTIAL] as to the full amount of the
claim upon final survey and determination by average 1. Whether or not Trans-Asia violated its warranty to maintain
adjuster Richard Hogg International (Phil.) of the damage the vessel.
sustained by reason of fire. An adjuster’s report on the fire o NO.
in question was submitted by Richard Hogg International. 2. Whether or not the loan agreement as evidenced by the
7. Plaintiff [TRANS-ASIA] executed a document denominated trust receipt was a form of subrogation.
"Loan and Trust receipt in which Trans-Asia received from o Yes.
Prudential 3M.
8. Thereafter, Prudential denied the insurance claim of Trans- Ruling:
Asia because Trans-Asia violated the Warranty which stated, First Issue
“"WARRANTED VESSEL CLASSED AND CLASS MAINTAINED".
Page 38 of 154
INSURANCE G01 CASE DIGESTS

 We sustain the findings of the Court of Appeals that the RTC which held that TRANS-ASIA breached the warranty
PRUDENTIAL was not successful in discharging the burden provision on CLASSED AND CLASS MAINTAINED,
of evidence that TRANS-ASIA breached the subject policy underscored that PRUDENTIAL can be deemed to have
condition on CLASSED AND CLASS MAINTAINED. made a valid waiver of TRANS-ASIA’s breach of warranty as
 Foremost, PRUDENTIAL, through the Senior Manager of its alleged, ratiocinating, thus: Third, after the loss, Prudential
Marine and Aviation Division, Lucio Fernandez, made a renewed the insurance policy of Trans-Asia for two (2)
categorical admission that at the time of the procurement consecutive years, from noon of 01 July 1994 to noon of 01
of the insurance contract in July 1993, TRANS-ASIA’s vessel, July 1995, and then again until noon of 01 July 1996. This
"M/V Asia Korea" was properly classed by Bureau Veritas. renewal is deemed a waiver of any breach of warrant.
 We are not unmindful of the clear language of Sec. 74 of the
Second Issue
Insurance Code which provides that, "the violation of a
material warranty, or other material provision of a policy on  What is clear from the wordings of the so-called "Loan and
the part of either party thereto, entitles the other to Trust Receipt Agreement" is that appellant is obligated to
rescind." It is generally accepted that "[a] warranty is a hand over to appellee "whatever recovery (Trans Asia) may
statement or promise set forth in the policy, or by reference make and deliver to (Prudential) all documents necessary to
incorporated therein, the untruth or non-fulfillment of prove its interest in the said property." For all intents and
which in any respect, and without reference to whether the purposes therefore, the money receipted is payment under
insurer was in fact prejudiced by such untruth or non- the policy, with Prudential having the right of subrogation
fulfillment, renders the policy voidable by the insurer."25 to whatever net recovery Trans-Asia may obtain from third
However, it is similarly indubitable that for the breach of a parties resulting from the fire. In the law on insurance,
warranty to avoid a policy, the same must be duly shown by subrogation is an equitable assignment to the insurer of all
the party alleging the same. We cannot sustain an allegation remedies which the insured may have against third person
that is unfounded. Consequently, PRUDENTIAL, not having whose negligence or wrongful act caused the loss covered
shown that TRANS-ASIA breached the warranty condition, by the insurance policy, which is created as the legal effect
CLASSED AND CLASS MAINTAINED, it remains that TRANS- of payment by the insurer as an assignee in equity. The loss
ASIA must be allowed to recover its rightful claims on the in the first instance is that of the insured but after
policy. reimbursement or compensation, it becomes the loss of the
 Assuming arguendo that TRANS-ASIA violated the policy insurer. It has been referred to as the doctrine of
condition on WARRANTED VESSEL CLASSED AND CLASS substitution and rests on the principle that substantial
MAINTAINED, PRUDENTIAL made a valid waiver of the justice should be attained regardless of form, that is, its
same. The Court of Appeals, in reversing the Judgment of
Page 39 of 154
INSURANCE G01 CASE DIGESTS

basis is the doing of complete, essential, and perfect justice


between all the parties without regard to form.31
 We agree. Notwithstanding its designation, the tenor of the
"Loan and Trust Receipt" evidences that the real nature of
the transaction between the parties was that the amount of
P3,000,000.00 was not intended as a loan whereby TRANS-
ASIA is obligated to pay PRUDENTIAL, but rather, the same
was a partial payment or an advance on the policy of the
claims due to TRANS-ASIA.

SC affirms. Petition denied.

Side Issue on attorney’s fees.

 In pursuant to Sec. 244 of the Insurance Code, there was an


unreasonable delay on the part of PRUDENTIAL to pay
TRANS-ASIA, as in fact, it refuted the latter’s right to the
insurance claims, from the time proof of loss was shown
and the ascertainment of the loss was made by the
insurance adjuster. Evidently, PRUDENTIAL’s unreasonable
delay in satisfying TRANS-ASIA’s unpaid claims compelled
the latter to file a suit for collection.

Page 40 of 154
INSURANCE G01 CASE DIGESTS

PREMIUM COMPANY to have his signature on the check


Identified preparatory to encashment.
o At that time the COMPANY reiterated that the
check was given "not as an obligation, but as a
91. ARCE V. CAPITAL INSURANCE , 117 SCRA 63 (1982) concession" because the renewal premium had not
been paid, The INSURED cashed the check but then
COMPANY/INSURER: Capital Insurance sued the COMPANY on the policy.
INSURED: Pedro Arce
Issue

Whether or not the insurer can be held liable despite the non-
Facts payment of the premiums

 Arce’s house was insured under a fire insurance policy Held and Ratio
 On November 27, 1965, the insurer sent to the insured a
Renewal Certificate to cover the period December 5, 1965 No
to December 5, 1966.
Relevant Provisions
 The COMPANY also requested payment of the
corresponding premium in the amount of P 38.10. Insurance Code
 Anticipating that the premium could not be paid on time,
the INSURED, thru his wife, promised to pay it on January 4, SEC. 72. An insurer is entitled to payment of premium as soon as the
1966. thing insured is exposed to the perils insured against, unless there is
 The COMPANY accepted the promise but the premium was
clear agreement to grant credit extension for the premium due. No
not paid on January 4, 1966.
policy issued by an insurance company is valid and binding unless
 On January 8, 1966, the house of the INSURED was totally
destroyed by fire. and until the premium thereof has been paid
 On January 10, 1966, INSURED's wife presented a claim for
Contract Stipulation
indemnity to the COMPANY.
o She was told that no indemnity was due because
IT IS HEREBY DECLARED AND AGREED that not. withstanding
the premium on the policy was not paid.
o Nonetheless the COMPANY tendered a check for anything to the contrary contained in the within policy, this
P300.00 as financial aid which was received by the insurance will be deemed valid and binding upon the Company only
INSURED's daughter, Evelina R. Arce as full when the premium and documentary stamps therefor have actually
settlement been paid in full and duly acknowledged in an official receipt signed
o The INSURED and his wife went to the office of the by an authorized official/representative of the Company,
Page 41 of 154
INSURANCE G01 CASE DIGESTS

Ruling WON the non payment of premium due resulted in the cancellation
of the contract of insurance
It is obvious from both the Insurance Act, as amended, and the
stipulation of the parties that time is of the essence in respect of the Held:
payment of the insurance premium so that if it is not paid the No
contract does not take effect unless there is still another stipulation
Ratio:
to the contrary. In the instant case, the INSURED was given a grace
We can not agree with appellant's theory that non-payment by it of
period to pay the premium but the period having expired with no the premium due, produced the cancellation of the contract of
payment made, he cannot insist that the COMPANY is nonetheless insurance. Such theory would place exclusively in the hands of one
obligated to him. of the contracting parties the right to decide whether the contract
should stand or not. Rather the correct view would seem to be this:
as the contract had become perfected, the parties could demand
from each other the performance of whatever obligations they had
92. PHIL. PHOENIX SURETY & ASSURANCE V. WOODWORKS, I NC., 20 assumed. In the case of the insurer, it is obvious that it had the right
SCRA 1270 (1967) to demand from the insured the completion of the payment of the
premium due or sue for the rescission of the contract. As it chose to
Facts: demand specific performance of the insured's obligation to pay the
balance of the premium, the latter's duty to pay is indeed
That on April 1, 1960, plaintiff issued to defendant Fire Policy No. indubitable.
9652 for the amount of P300,000.00, under the terms and
conditions therein set forth in said policy a copy of which is hereto PHIL PHOENIX V WOODWORKS 92 SCRA 419 (1979)
attached and made a part hereof as Annex "A";
Facts:
That the premiums of said policy as stated in Annex "A" amounted  July 21, 1960: Woodworks, Inc. was issued a fire policy for its
to P6,051.95; the margin fee pursuant to the adopted plan as an building machinery and equipment by Philippine Phoenix Surety
implementation of Republic Act 2609 amounted to P363.72, copy of & Insurance Co. for P500K covering July 21, 1960 to July 21,
said adopted plan is hereto attached as Annex "B" and made a part 1961. Woodworks did not pay the premium totalling
hereof, the documentary stamps attached to the policy was P96.42; to P10,593.36.
 April 19, 1961: It was alleged that Woodworks notified
That the defendant paid P3,000.00 on September 22, 1960 under Philippine Phoenix the cancellation of the Policy so Philippine
official receipt No. 30245 of plaintiff; Phoenix credited P3,110.25 for the unexpired period of 94 days
and demanded in writing the paymentof P7,483.11
That plaintiff made several demands on defendant to pay the
amount of P3,522.09.
Issue:
Page 42 of 154
INSURANCE G01 CASE DIGESTS

 Woodworks refused stating that it need not pay premium  The burden is on an insured to keep a policy in force by
"because the Insurer did not stand liable for any indemnity the payment of premiums, rather than on the insurer to exert
during the period the premiums were not paid." every effort to prevent the insured from allowing a policy to
 Philippine Phoenix filed with the CFI to recover its earned elapse through a failure to make premium payments.
premium of P7,483.11
 Woodworks: to pay the premium after the issuance of the
policy put an end to the insurance contract and rendered the 93. MAKATI TUSCANY CONDOMINIUM V. COURT OF APPEALS, 215
policy unenforceable SCRA 463 (1992)
 CFI: favored Philippine Phoenix
215 SCRA 462
ISSUE: W/N there was a valid insurance contract despite no BELLOSILLO; November 6, 1992
premium payment was paid
NATURE
Appeal from decision of the CA
HELD:
NO. Reversed
FACTS
- American Home Assurance Co. (AHAC), represented by
 Policy provides for pre-payment of premium. To constitute an American International Underwriters (Phils.), Inc., issued in
extension of credit there must be a clear and express favor of petitioner Makati Tuscany Condominium
agreement therefor and there nust be acceptance of the
Corporation an insurance policy on the latter's building and
extension - none here
premises, for the period 1 March 1982 to1 March 1983. The
 Since the premium had not been paid, the policy must be
deemed to have lapsed. premium was paid on installments all of which were
 failure to make a payment of a premium or assessment at the accepted by AHAC.
time provided for, the policy shall become void or forfeited, or
the obligation of the insurer shall cease, or words to like effect,
because the contract so prescribes and because such a - A second policy was issued to renew the first one, this
stipulation is a material and essential part of the contract. This time covering the period 1 March 1983 to 1 March 1984.
is true, for instance, in the case of life, health and accident, fire This was also pain in installment basis.
and hail insurance policies
 Explicit in the Policy itself is plaintiff's agreement to indemnify
defendant for loss by fire only "after payment of - A third policy was again issued for the period 1 March
premium" Compliance by the insured with the terms of the
1984 to 1 March 1985. For this, petitioner made two
contract is a condition precedent to the right of recovery.
installment payments, both accepted by AHAC. Thereafter,
Page 43 of 154
INSURANCE G01 CASE DIGESTS

petitioner refused to pay the balance of the premium. AHAC the receipts ordinarily issued by AHAC on premium
filed an action to recover the unpaid balance of payments the only plausible conclusion is that AHAC has no
P314,103.05. right to demand their payment after the lapse of the term
of said policy on March 1, 1985. Therefore, Tuscany was
justified in refusing to pay the same.
- Petitioner explained that it discontinued the payment of
premiums because the policy did not contain a credit clause
in its favor and the receipts for the installment payments - CA modified the decision by ordering Tuscany to pay the
covering the policy for 1984-85, as well as the two (2) balance of the premiums due on the third policy plus legal
previous policies, stated the following reservations: interest until fully paid, and affirming the denial of the
2. Acceptance of this payment shall not waive any of the counterclaim.
company rights to deny liability on any claim under the
policy arising before such payments or after the expiration
of the credit clause of the policy; and Petitioner’s Claims
Petitioner argues that where the premiums is not actually
3. Subject to no loss prior to premium payment. If there be
any loss such is not covered. paid in full, the policy would only be effective if there is an
acknowledgment in the policy of the receipt of premium
pursuant to Sec. 78 of the Insurance Code. The absence of
- Petitioner further claimed that the policy was never an express acknowledgment in the policies of such receipt
binding and valid, and no risk attached to the policy. It then of the corresponding premium payments, and petitioner's
pleaded a counterclaim for P152k for the premiums already failure to pay said premiums on or before the effective
paid for 1984-85, and in its answer with amended dates of said policies rendered them invalid. Petitioner thus
counterclaim, sought the refund of P924,206.10 concludes that there cannot be a perfected contract of
representing the premium payments for 1982-85. insurance upon mere partial payment of the premiums
because under Sec. 77 of the Insurance Code, no contract of
insurance is valid and binding unless the premium thereof
- Trial court dismissed the complaint and the counterclaim has been paid, notwithstanding any agreement to the
upon the following findings: (1) payment of the premiums contrary.
of the three policies were made during the term of said
policies, hence, it could not be said, inspite of the ISSUE
reservations, that no risk attached under the policies; (2) as WON payment by installment of the premiums due on an
regards the unpaid premiums, in view of the reservation in insurance policy invalidates the contract of insurance
Page 44 of 154
INSURANCE G01 CASE DIGESTS

HELD - Section 78 of the Insurance Code in effect allows waiver by


Ratio Where the risk is entire and the contract is indivisible, the insurer of the condition of prepayment by making an
the insured is not entitled to a refund of the premiums paid acknowledgment in the insurance policy of receipt of
if the insurer was exposed to the risk insured for any period, premium as conclusive evidence of payment so far as to
however brief or momentary. make the policy binding despite the fact that premium is
actually unpaid. Section 77 merely precludes the parties
from stipulating that the policy is valid even if premiums are
Reasoning not paid, but does not expressly prohibit an agreement
- The obligation to pay premiums when due is ordinarily as
granting credit extension, and such an agreement is not
indivisible obligation to pay the entire premium. Here, the contrary to morals, good customs, public order or public
parties herein agreed to make the premiums payable in
policy.
installments, and there is no pretense that the parties never
envisioned to make the insurance contract binding between
them. And the insured never informed the insurer that it - At the very least, both parties should be deemed in
was terminating the policy because the terms were estoppel to question the arrangement they have voluntarily
unacceptable. accepted.
Disposition Judgment affirmed. Costs against petitioner.

- There is nothing in Section 77 which suggests that the


parties may not agree to allow payment of the premiums in
installment, or to consider the contract as valid and binding
upon payment of the first premium.

- The records clearly show that petitioner and private


respondent intended subject insurance policies to be
binding and effective notwithstanding the staggered
payment of the premiums. Acceptance of payments speaks
loudly of the insurer's intention to honor the policies it
issued to petitioner.

Page 45 of 154
INSURANCE G01 CASE DIGESTS

94. TIBAY V. COURT OF APPEALS, 257 SCRA 126 (1996)  No policy of insurance can ever pretend to be efficacious or
effective until premium has been fully paid.
FACTS:
 Premium is the elixir vitae of insurance business, and all
 Insurer: Fortune actuarial calculations and various tabulations of
 Insured: Violeta Tibay and/or Nicolas Ronaldo probabilities of losses under the risks insured against are
 Property Insured: 2 storey residential building located in based on the sound hypothesis of prompt payment of
Makati premium.
 Insurance Contract: Fire Insurance Policy for 600,000php
 Total Amount of Premium: 2,983.50php but Tibay only paid
600php thus leaving a balance. 95. MALAYAN INSURANCE V. COURT OF APPEALS, 154 SCRA 672
 Building completely destroyed by fire and two days after, (1987)
Tibay paid the unpaid premium and filed with Fortune a
MALAYAN INSURANCE CO., INC. v. ARNALDO and PINCA 154 SCRA
claim on fire insurance policy.
672 CRUZ;
ISSUE:
October 12, 1987 !
W/N fire insurance policy is valid, binding, and enforceable upon
FACTS !
mere partial payment of premium?
- On June 7, 1981, Malayan Insurance Co. (MICO), issued fire
HELD:
insurance for the amount of P14,000 on the property of private
 NO respondent, Pinca, effective July 1981-1982. MICO later allegedly
 The consideration is the premium, which must be paid at cancelled the policy for non-payment of the premium and sent a
the time and in the way and manner specified in the policy, notice to Pinca. On Dec. 24 Adora, an agent of MICO, received
if not paid, the policy will lapse and be forfeited by its own Pinca's payment, which was remitted to MICO. On Jan. 18, 1982,
terms. Pinca's property was completely burned. On Feb. 5, MICO
 Clearly the policy provides for payment in FULL returned Pinca's payment to Adora on the ground that her
 The premium has only been partially paid and balance paid policy had been cancelled; the latter refused to accept it. Her
only after the peril insured against occurred thus contract demand for payment having been rejected by MICO, Pinca went to
did not take effect and insured cannot collect at all on the the Insurance Commission. Public respondent Arnaldo, the
policy. But the controversy lies in the phrase “unless and Insurance Commissioner, sustained Pinca, hence this petition
until the premium thereof has been paid”. from MICO. Records show MICO received Arnaldo's decision on
Page 46 of 154
INSURANCE G01 CASE DIGESTS

April 10; MICO filed a MFR on April 25 which was denied on June 4; of any court in all cases at 15 days from the notice of the
MICO received notice of this denial on June 14; instant petition was decision appealed from. Since the MFR was filed only 15 days
filed on July 2. after receiving notice of the decision, it was already 18 days late by
July 2. So whichever is applied, the petition is still late.

ISSUES
Substantive
Procedural
2. YES - A valid cancellation requires the following conditions based
1. WON the petition should be dismissed for late filing on Sections 64-65 of the Code: prior notice which must be based on
Substantive the occurrence of one or more of the grounds mentioned in Sec 64
(in this case, non-payment of premium), after the effective date of
2. WON there was a valid insurance contract at the time of the loss the policy; the notice must be written and mailed to the address on
the policy; it must state the ground(s) for cancellation and the
3. WON Adora was authorized to receive such payment
insurer must furnish details upon the request of the insured. - It
4. WON an adjuster is indispensable in the valuation of the loss is undisputed that payment of premium was made. Petitioner relies
heavily on Sec 77 of the Insurance Code to contest this, the
said provision requiring payment of premium as soon as the
thing is exposed to the peril insured against and that the policy
HELD
is invalid without it. However, this is not applicable in the
Procedural instant case as payment was eventually made. It is to be noted that
the premium invoice was stamped "Payment Received#,
1. YES - Petitioner invokes Sec 416 of the Insurance Code which indicating an understanding between the parties that payment
grants it 30 days from notice of the Insurance Commission within could be made later. This is furthered by the fact that Adora had
which to appeal by certiorari with the Court. MICO filed its MFR on earlier told her to call him anytime she was ready with her payment.
April 25, 15 days after the notice; the reglementary period began to The Court also finds it strange that MICO only sought to return
run again after June 13. Since the petition was filed only on July 2, it Pinca's Jan. 15 payment only on Feb. 5, long after her house had
was tardy by 4 days. Alternatively it invokes Rule 45 of the Rules of burned down—this makes petitioner's motives highly suspect. -
Court for certiorari but the petition still exceeds the 15 day limit MICO claims to have sent a notice to Pinca, who flatly denied
from the June 13 notice. -Respondents, on the other hand, invoke receiving one. Pinca did not have to prove this since the strict
Sec. 39 of B.P. 129 which pegs the period for appeal from decisions

Page 47 of 154
INSURANCE G01 CASE DIGESTS

language of Sec 64 requires that MICO ensure the cancellation was 96. UCPB GENERAL INSURANCE V. MASAGANA TELEMART, 356
actually sent to and received by the insured. SCRA 307 (2001)

- MICO also suggests that Pinca knew the policy had been Facts:
cancelled and was paying the premium in order to renew the
policy. A close study of the transcripts show, however, that Pinca  On April 15, 1991, petitioner issued five (5) insurance
only meant to renew the policy had it been cancelled but not if it policies covering respondent's various property described
was still in effect—it was conditional. Payment was thus legally therein against fire, for the period from May 22, 1991 to
made on the original transaction and validly received by Adora, May 22, 1992.
 In March 1992, petitioner evaluated the policies and
who was not informed of the alleged cancellation and thus saw no
decided not to renew them upon expiration of their terms
reason to reject the payment. on May 22, 1992. Petitioner advised respondent's broker,
Zuellig Insurance Brokers, Inc. of its intention not to renew
3. YES - Sec. 306 of the Insurance Code provides that any
the policies.
insurance company that delivers a policy to its agent is deemed  On April 6, 1992, petitioner gave written notice to
to have authorized such agent to receive payment of premium respondent of the non-renewal of the policies at the
on its behalf. It is a well-known principle under the law of address stated in the policies.
agency that payment to an authorized agent is equivalent to  On June 13, 1992, fire razed respondent's property covered
payment to the principal himself. MICO's acknowledgement of by three of the insurance policies petitioner issued.
Adora as its agent thus defeats its contention that he was not  On July 13, 1992, respondent presented to petitioner's
cashier at its head office five (5) manager's checks in the
authorized to receive payments on its behalf. 4.
total amount of P225,753.95, representing premium for the
NO - In absence of fraud, the amount of the loss may be renewal of the policies from May 22, 1992 to May 22, 1993.
No notice of loss was filed by respondent under the policies
determined on the basis of such proof offered by the insured. Here,
prior to July 14, 1992.
the certification of the Integrated National Police as the extent of
 On July 14, 1992, respondent filed with petitioner its formal
the loss should suffice. claim for indemnification of the insured property razed by
fire.
Disposition petition is DENIED.  On the same day, July 14, 1992, petitioner returned to
respondent the five (5) manager's checks that it tendered,
and at the same time rejected respondent's claim for the
reasons:
o that the policies had expired and were not
renewed, and

Page 48 of 154
INSURANCE G01 CASE DIGESTS

o that the fire occurred on June 13, 1992, before premium for renewal of the policies was tendered on July 13, 1992,
respondent's tender of premium payment. a month after the fire occurred on June 13, 1992. The assured did
 On July 21, 1992, respondent filed with the Regional Trial not even give the insurer a notice of loss within a reasonable time
Court, Branch 58, Makati City, a civil complaint against after occurrence of the fire.
petitioner for recovery of P18,645,000.00, representing the
face value of the policies covering respondent's insured
property razed by fire, and for attorney's fees.
 After due trial, on March 10, 1993, the Regional Trial Court, 97. AMERICAN HOME INSURANCE V. TANTUCO ENTERPRISES, 366
Branch 58, Makati, rendered decision in favor of the SCRA 740 (2001)
plaintiff and against the defendant.
 In due time, petitioner appealed to the Court of PREMIUMS
Appeals. The CA promulgated its decision affirming that of
the RTC. AMERICAN HOME ASSURANCE COMPANY V TANTUCO ENTERPRISES
 Hence, this appeal.
FACTS:
Issue: Whether the fire insurance policies issued by petitioner to the
1. Respondent Tantuco Enterprises, Inc. is engaged in the coconut
respondent covering the period May 22, 1991 to May 22, 1992, had
expired on the latter date or had been extended or renewed by an oil mining and refining industry.
implied credit arrangement though actual payment of premium was
a. It owns 2 mills.
tendered on a later date after the occurrence of the risk (fire)
insured against. b. Both located at its factory compound in Lucena City.
HELD: NO 2. Respondent Tantuco commenced its business in ONLY ONE OIL
MILL. In 1998, it started operating its second mill and it was referred
An insurance policy, other than life, issued originally or on
to as the new oil mill.
renewal, is not valid and binding until actual payment of the
premium. Any agreement to the contrary is void. 11The parties may 3. The two mills were separately covered by fire insurance policies
not agree expressly or impliedly on the extension of creditor time to
issued by Petitioner American Home Assurance Co., Philippine
pay the premium and consider the policy binding before actual
payment. Branch.

a. FIRST OIL MILL: insured for P.3,000,000.00


The case of Malayan Insurance Co., Inc. vs. Cruz-Arnaldo, 12 cited by
the Court of Appeals, is not applicable. In that case, payment of the
b. SECOND OIL MILL: insured for P.6,000,000.00
premium was in fact actually made on December 24, 1981, and the
fire occurred on January 18, 1982. Here, the payment of the

Page 49 of 154
INSURANCE G01 CASE DIGESTS

c. Official receIpts indicating PAYMENT FOR THE FULL AMOUNT OF Whether or not Petitioner American Home Assurance is liable on
THE PREMIUM were issued by the petitioner's agent. the insurance policy?

4. Fire broke out and consumed the new oil mill. Respondent HELD:
Tantuco immediately informed the petitioner of the incident.
Yes. The Petitioner American Home Assurance is liable on the
5. PETITIONER REJECTED THE RESPONDEN'D CLAIM FOR THE insurance policy.
INSURANCE PROCEEDS
RATIONALE:
BASIS: a. that no policy was issued by it covering the burned oil mill.
It stated that the description of the insured establishment referred The petition is lack of merit.
to another building (the 2 policies extended the coverage to oil mill In construing the words used descriptive of a building insured, the
under Building No. 5 but the affected oil mill was under Building no. greatest liberality is shown by the courts in giving effect to the
14). insurance. The courts are inclined to consider that the policy of
b. The respondent is barred by estoppel from claiming that the insurance covers any building which the parties manifestly intended
description of the insured oil mill in the policy was wrong, because it to insure, however inaccurate the description may be. Also, in
determining what the parties intended, the courts will read and
retained the policy without having the same corrected before the
fire. construe the policy as a whole and if possible, give effect to all the
parts of the contract, that in case there is doubt, this doubt is to be
c. The petitioner included the Important Notice in the policy that resolved against the insurer. Object and purpose of the contract
states Please read and examine the policy and if incorrect, return it must be considered.
immediately for alteration.
PREMIUM ISSUE:

PETITIONER'S CONTENTIONS:
6. RTC: Petitioner Insurance is liable on the insurance policy.
a. To further attempt to avoid the liability, petitioner claims that
7. CA: Affirmed the decision of the RTC in toto. respondent forfeited the renewal policy for it failed to pay the full
amount of the premium and breach of the Fire

Extinguishing Appliances Warranty.


ISSUE:

Page 50 of 154
INSURANCE G01 CASE DIGESTS

b. Respondent only paid for P75,147.00 instead of P89,770.20 which 98. GREAT PACIFIC LIFE INSURANCE V. COURT OF APPEALS, 184
leaves the difference of 14,623.20. The petitioner argues that the SCRA 501 (1990)
deficiency suffices to invalidate the policy under Section 77 of the
Insurance Code.
Facts:
c. Also, that even if Mr. Borja, who acknowledged the that the
1. This case involves an insured's claim for refund of the first
amount was lacking by reason of rebate is illegal.
premium on the endowment policy on his life, upon being
notified by the insurer that the policy never took effect
SC ruled that it fails to impress. It is true that there is a policy's
despite the premium payment.
condition for payment of the renewal premium on time and the 2. Teodoro Cortez applied for a 20-year endowment policy for
respondent's non-compliance with it. Yet, it did not contain any 30,000.
specific and definite allegation that the respondent did not pay the 3. His application was accepted, approved and a policy was
premium, or that it did not pay the full amount, or that it sis not pay issued.
the amount on time. 4. The policy was delivered to him by the underwriter
Margarita Seiga.
The issue of inadequacy of payment was never raised in the pre-trial 5. The effective date indicated on the face of the policy in
proceedings and there is the petitioner failed to present any witness question was December 25, 1972. The annual premium was
to testify that respondent indeed failed to pay the full amount of P1,416.60. Mrs. Siega assured him that the first premium
may be paid within the grace period of thirty (30) days from
the premium.
date of delivery of the policy
6. The first premium was then paid in 3 installments.
NO BREACH FOR NOT PROVIDING FIRE EXTINGUISHING
7. In a letter dated June 1, 1973 (Exh. E), defendant advised
APPLIANCES. plaintiff that Policy No. 221944 (Exh. A) was not in force. To
make it enforceable and operative, plaintiff was asked to
remit the balance of P1,015.60 to complete his initial annual
premium due December 15, 1972, and to see Dr. Felipe V.
Remollo for another full medical examination at his own
expense.
8. Cortez' reaction to the company's act was to immediately
inform it that he was cancelling the policy and he
demanded the return of his premium plus damages.
9. RTC Ruled in favour of Cortez
10. CA Affirmed

Issue:
Page 51 of 154
INSURANCE G01 CASE DIGESTS

1. Whether or not Cortez is entitled to a refund of his 99. STOKES V. MALAYAN, 127 SCRA 766 (1984)
premium.
FACTS:
Held: 1. Daniel Adolfson had a subsisting Malayan car insurance
1. YES policy with coverage against own damage as well as 3rd
2. When the petitioner advised private respondent on June 1, party liability when his car figured in a vehicular accident
1973, four months after he had paid the first premium, that with another car, resulting to damage to both vehicles.
his policy had never been in force, and that he must pay 2. At the time of the accident, Adolfson’s car was being driven
another premium and undergo another medical by James Stokes, who was authorized to do so by Adolfson.
examination to make the policy effective, the petitioner Stokes, an Irish tourist who had been in the Philippines for
committed a serious breach of the contract of insurance. only 90 days, had a valid and subsisting Irish driver’s license
3. Petitioner should have informed Cortez of the deadline for but without a Philippine driver’s license.
paying the first premium before or at least upon delivery of 3. Adolfson filed a claim with Malayan but the latter refused to
the policy to him, so he could have complied with what was pay contending that Stokes was not an authorized driver
needful and would not have been misled into believing that under the “Authorized Driver” clause of the insurance policy
his life and his family were protected by the policy, when in relation to Section 21 of the Land Transportation Office.
actually they were not.
4. And, if the premium paid by Cortez was unacceptable for ISSUE: Whether or not Malayan is liable to pay the insurance claim
being late, it was the company's duty to return it. of Adolfson
5. By accepting his premiums without giving him the
corresponding protection, the company acted in bad faith. HELD: NO. A contract of insurance is a contract of indemnity upon
6. Sections 79, 81 and 82 of P.D. 612 of the Insurance Code of the terms and conditions specified therein. When the insurer is
1978 provide when the insured is entitled to the return of called upon to pay in case of loss or damage, he has the right to
premium paid. insist upon compliance with the terms of the contract. If the insured
7. Since his policy was in fact inoperative or ineffectual from cannot bring himself within the terms and conditions of the
the beginning, the company was never at risk, hence, it is contract, he is not entitled as a rule to recover for the loss or
not entitled to keep the premium. damage suffered. For the terms of the contract constitute the
measure of the insurer’s liability, and compliance therewith is a
condition precedent to the right of recovery.

At the time of the accident, Stokes had been in the Philippines for
more than 90 days. Hence, under the law, he could not drive a
motor vehicle without a Philippine driver’s license. He was therefore
not an “authorized driver” under the terms of the insurance policy
in question, and Malayan was right in denying the claim of the
insured.
Page 52 of 154
INSURANCE G01 CASE DIGESTS

100. CAPITAL INSURANCE V. PLASTIC ERA, 65 SCRA 134 (1975)


Acceptance of premium within the stipulated period for payment
thereof, including the agreed period of grace, merely assures GR No. L-22375 July 18, 1975
continued effectivity of the insurance policy in accordance with its
terms. Such acceptance does not estop the insurer from interposing Facts:
any valid defense under the terms of the insurance policy.  Capital Insurance delivered to Plastic Era its open fire policy.
Capital Insurance undertook to insure Plastic Era's building,
The principle of estoppel is an equitable principle rooted upon equipments, raw materials, products and accessories
natural justice which prevents a person from going back on his own located in Mandaluyong, Rizal.
acts and representations to the prejudice of another whom he has  The policy expressly provided that if the property insured
led to rely upon them. The principle does not apply to the instant would be destroyed or damaged by fire after the payment
case. In accepting the premium payment of the insured, Malayan of the premiums, at any time between Dec. 15, 1960 and
was not guilty of any inequitable act or representation. There is 1PM of Dec. 15, 1961, the insurance company shall make
nothing inconsistent between acceptance of premium due under an good all such loss or damage in an amount not exceeding
insurance policy and the enforcement of its terms. 100K.
 Plastic Era failed to pay the corresponding insurance
premium. However, its duly authorized representative
executed an acknowledgment receipt with a corresponding
'promise to pay.'
 Plastic Era delivered to Capital Insurance a check for the
amount of 1K, as partial payment of the premium,
postdated Jan 16, 1961 and drawn against the Bank of
America. When Capital Insurance tried to deposit the check
a month after the date, it was dishonored for lack of funds.
 January 18, 1961, 2 days after the premium was due, the
property insured by Plastic Era was destroyed by fire. They
notified Capital of the loss and filed its claim for indemnity
through the Manila Adjustment Company, who estimated
the loss at P283,875.
 The same property was insured with Philamgen for 200K.
 Plastic Era demanded from Capital the payment of the sum
of 100K as indemnity, but was denied because they had
failed to pay the premium.

Issue:

Page 53 of 154
INSURANCE G01 CASE DIGESTS

Whether there was a perfected contract of insurance between  When the damage or loss of the insured property occurred,
Capital and Plastic Era the insurance policy was in full force and effect. The fact
Whether there was payment of premium (either through the that the check issued by Plastic Era in partial payment of the
acknowledgment receipt with a promise to pay, or through the promissory note was later on dishonored did not in any way
postdated check) operate as a forfeiture of its rights under the policy, there
being no express stipulation therein to that effect.
Ruling:  Moreover, by accepting its promise to pay the insurance
Capital Insurance was held to be liable to Plastic Era premium within 30 days from the effectivity date of the
 It is clear from the terms of the policy that it is only upon policy, Capital Insurance had in effect extended credit to
payment of the premiums that Capital agrees to insure the Plastic Era. The payment of the premium on the insurance
properties of Plastic Era against loss or damage in an policy therefore became an independent obligation the
amount not exceeding P100K. non-fulfillment of which would entitle Capital Insurance to
 The only question/issue is whether there was payment. recover. It did not have the right to cancel the policy for
 The issuance of the postdated check did not produce the nonpayment of the premium except by putting Plastic Era in
effect of payment. default and giving it personal notice to that effect.
o The mere delivery of a bill of exchange in payment  Capital is likewise estopped from claiming a forfeiture of its
of a debt does not immediately effect payment. It policy for non-payment even if the check was dishonored.
simply suspends the action arising from the original Although the check was due for payment on Jan 16, and
obligation in satisfaction of which it was delivered, Plastic Era had sufficient funds during that date, Capital
until payment is accomplished either actually or decided to hold the check of 35 days before presenting it
presumptively. It shall only produce the effect of for payment.
payment when they have been encashed, or when
through the fault of the creditor they have been
impaired.
 However, in Capital's acceptance of the promissory note, it
implicitly agreed to modify the tenor of the insurance policy
and in effect, waived the provision that it would only pay for
the loss or damage in case the same occurs after the
payment of the premium.
 The insurance policy is silent as to the mode of payment.
Therefore, Capital Insurance is deemed to have accepted
the promissory note in payment of the premium. This
acceptance of the note rendered the policy immediately
operative on the date it was delivered.

Page 54 of 154
INSURANCE G01 CASE DIGESTS

LOSS AUTHORIZED DRIVER:

Any of the following:

101. CCC INSURANCE V. COURT OF APPEALS, 31 SCRA 264 (a) The insured;
Facts: (b) Any person driving on the Insured's order or with his
1. Carlos F. Robes took an insurance, with the CCC Insurance permission, provided that the person driving
Corporation, on his Dodge Kingsway car against loss or is permitted in accordance with licensing laws or
damage through accident for an amount not exceeding regulations to drive the motor vehiclecovered by this
P8,000.00 Policy, or has been so permitted and is not disqualified
2. The insured vehicle, while being driven by the owner's by order of a court of law or by reason of any
driver, became involved in a vehicular collision along Rizal enactment or regulation from driving such Motor
Avenue Extension, Potrero, Malabon, Rizal. The car was Vehicle. (Emphasis ours)
damaged, and the repair was estimated to cost P5,300.00.
3. The insurance company refused to pay for the repair or
3. It has been found as a fact by the Court of Appeals that
restore the car.
Domingo Reyes, the, driver who was at the wheel of the
4. The insurance company disclaimed liability for payment,
insured car at the time of the accident, does not know how
alleging that there had been violation of the insurance
to read and write; that he was able to secure a driver's
contract because the one driving the car at the time of the
license, without passing any examination therefor, by
incident was not an "authorized driver."
paying P25.00 to a certain woman
5. RTC Ruled for plaintiff and ordered defendant insurer to
4. The appellant insurer insists that, under the established
pay.
facts of this case, Reyes, being admittedly one who cannot
6. The CA affirmed the RTC
read and write, who has never passed any examination for
drivers, and has not applied for a license from the duly
Issue:
constituted government agency entrusted with the duty of
1. Whether or not the damage to the insured car was not
licensing drivers, cannot be considered an authorized driver.
covered by the insurance policy because at the time of the
5. Under Section 24 of the Revised Motor Vehicles Law, Act
accident it was being driven by one who was not an
3992 of the Philippine Legislature, as amended by Republic
authorized driver.
Acts Nos. 587, 1204 and 2863, the issuance of a driving
license without previous examination does not necessarily
Held:
imply that the license issued is invalid.
1. The damage is under the insurance policy. The driver falls
under the definition of an authorized driver
2. Under the law
Page 55 of 154
INSURANCE G01 CASE DIGESTS

This insurance does not cover any loss or damage


102. COUNTRY BANKERS INSURANCE V. LLANGA BAY AND COMMUNITY occasioned by or through or in consequence,
MULTI-PURPOSE COOPERATIVE , 374 SCRA 653 (2002) directly or indirectly, of any of the following
occurrences, namely:
xxx
COUNTRY BANKERS INSURANCE CORPORATION, petitioner,
vs. LIANGA BAY AND COMMUNITY MULTI-PURPOSE xxx
COOPERATIVE, INC., respondent.
(d) Mutiny, riot, military or popular uprising,
DECISION insurrection, rebellion, revolution, military or
usurped power.
DE LEON, JR., J.:

FACTS:
~Respondent filed in the trial court the complaint for recovery of
“loss, damage or liability” against petitioner.
~Petitioner Country Bankers is a domestic corporation principally
engaged in the insurance business. RTC --- Trial court decided in favor of the respondent ordering
petitioner to pay respondent P200,000.00, with interest at twelve
~Respondent Lianga Coop is a duly registered cooperative judicially percent (12%) per annum from the date of filing of the complaint
declared insolvent. until fully paid, as well as Fifty Thousand Pesos (P50,000.00) as
~Petitioner and the respondent entered into a 1-year fire insurance actual damages, Fifty Thousand Pesos (P50,000.00) as exemplary
contract over respondent’s stocks-in-trade (inside their building at damages, Five Thousand Pesos (P5,000.00) as litigation expenses,
Lianga Bay, Surigao del Sur) for the sum of Two Hundred Thousand Ten Thousand Pesos (P10,000.00) as attorney’s fees, and the costs
Pesos (P200,000.00). of suit.

~Respondent’s building was gutted by fire and reduced to ashes. CA --- affirmed

~Respondent filed an insurance claim submitting: (a) the Spot


Report of Pfc. Arturo V. Juarbal, INP Investigator; (b) the Sworn ISSUE: Whether petitioner Country Bankers is liable to pay the
Statement of Jose Lomocso; and (c) the Sworn Statement of Ernesto claim plus interest, damages, attorney’s fees and costs of suit.
Urbiztondo.
~The petitioner denied the insurance claim on the ground that,
based on the submitted documents, the building was set on fire by HELD: NO, Country Bankers is liable only for the insurance claim
two (2) NPA rebels who wanted to obtain canned goods, rice and but not for 12% interest, damages and fees.
medicines as provisions for their comrades in the forest, and that
Since petitioner’s defense is non-coverage by reason of the
such loss was an excepted risk under paragraph No. 6 of the policy,
exemption or exception clause in the fire insurance policy, it has the
which states:
Page 56 of 154
INSURANCE G01 CASE DIGESTS

burden of proving the facts upon which such excepted risk is based, 1. When the obligation is breached, and it
by a preponderance of evidence.[7] But petitioner failed to do so. consists in the payment of a sum of money, i.e., a
loan or forbearance of money, the interest due
The petitioner relied on the Sworn Statements of Lomocso and
should be that which may have been stipulated in
Urbiztondo and the Spot Report of Pfc. Arturo V. Juarbal.
writing. Furthermore, the interest due shall itself
But the Sworn Statements were found to be inadmissible in earn legal interest from the time it is judicially
evidence, for being hearsay, inasmuch as they did not take the demanded. In the absence of stipulation, the rate
witness stand and could not therefore be cross-examined. of interest shall be 12% per annum to be computed
from default, i.e., from judicial or extrajudicial
As to Investigator Juarbal’s Spot Report, it “was based on the demand under and subject to the provisions of
personal knowledge of the caretaker Lomocso who witnessed every Article 1169 of the Civil Code.
single incident surrounding the facts and circumstances of the case.” 2. When an obligation, not constituting a loan
And so said Spot Report was also inadmissible for being hearsay as it or forbearance of money, is breached, an interest
was not based on the investigator’s personal knowledge although on the amount of damages awarded may be
he took the witness stand. imposed at the discretion of the court at the rate of
The petitioner’s evidence to prove its defense is sadly wanting 6% per annum. No interest, however, shall be
and thus, it is liable to the respondent under the Fire Insurance adjudged on unliquidated claims or damages except
Policy. But we do not sustain the trial court’s imposition of 12% when or until the demand can be established with
interest on the insurance claim as well as the monetary award for reasonable certainty. Accordingly, where the
actual and exemplary damages, litigation expenses and attorney’s demand is established with reasonable certainty,
fees for lack of legal and valid basis. the interest shall begin to run from the time the
claim is made judicially or extrajudicially (Art. 1169,
Concerning the application of the proper interest rates, the Civil Code) but when such certainty cannot be so
following guidelines were set in Eastern Shipping Lines, Inc. v. Court reasonably established at the time the demand is
of Appeals and Mercantile Insurance Co., Inc.:[15] made, the interest shall begin to run only from the
I. When an obligation, regardless of its source, date the judgment of the court is made (at which
i.e., law, contracts, quasi-contracts, delicts or quasi- time the quantification of damages may be deemed
delicts, is breached, the contravenor can be held to have been reasonably ascertained). The actual
liable for damages. The provisions under Title XVIII base for the computation of legal interest shall, in
on “Damages” of the Civil Code govern in any case, be on the amount finally adjudged.
determining the measure of recoverable damages. 3. When the judgment of the court awarding
II. With regard particularly to an award of a sum of money becomes final and executory, the
interest in the concept of actual and compensatory rate of legal interest, whether the case falls under
damages, the rate of interest, as well as the accrual paragraph 1 or paragraph 2, above, shall be 12% per
thereof, is imposed, as follows: annum from such finality until its satisfaction, this

Page 57 of 154
INSURANCE G01 CASE DIGESTS

interim period being deemed to be by then an damages, Five Thousand Pesos (P5,000.00) as litigation expenses,
equivalent to a forbearance of credit. and Ten Thousand Pesos (P10,000.00) as attorney’s fees are hereby
DELETED. Costs against the petitioner.
SO ORDERED.
The insurance claim in this case is evidently not a forbearance
of money, goods or credit, and thus the interest rate should be 6%
from the date of filing of the complaint.
103. PARIS-MANILA PERFUMERY V. PHOENIX ASSURANCE , 49 PHIL.
We find no justification for the award of actual 753 (1926)
damages. Well-entrenched is the doctrine that actual,
compensatory and consequential damages must be proved, and
cannot be presumed. The justification, if any, for such an award of Paris-Manila Perfume Co. v. Phoenix Assurance Co.
actual damages does not appear in the body of the decision of the
trial court. G.R. No. L-25845
We likewise find no legal and valid basis for granting December 17, 1926
exemplary damages. Article 2229 of the New Civil Code provides
that exemplary damages may be imposed by way of example or Lessons Applicable: Loss, the immediate cause of which was the
correction for the public good. However, it cannot be recovered as a peril insured against, if the proximate cause thereof was NOT
matter of right. It is based entirely on the discretion of the court.
excepted in the contract (Insurance)
And we find no justification for the award of litigation
expenses and attorney’s fees. Article 2208 of the New Civil Code Facts:
enumerates the instances where such may be awarded and, in all
 May 22, 1924: A fire insurance policy was issued by Phoenix
cases, it must be reasonable, just and equitable if the same were to
Assurance Company, Limited to Messrs. Paris-Manila
be granted. The award of attorney’s fees is the exception rather
Perfumery Co. (Peter Johnson, Prop.) for P13,000
than the general rule. As such, it is necessary for the court to make
o also insured with other insurance companies for
findings of facts and law that would bring the case within the
P1,200 and P5,000 respectively
exception and justify the grant of such award. We find none in this
 July 4, 1924: The Perfumery was burned unknown of the
case.
cause totalling a loss of P38.025.56
WHEREFORE, the appealed Decision is MODIFIED. The rate of  Phoenix refused to pay nor to appoint an arbitrator stating
interest on the adjudged principal amount of Two Hundred that the policy did not cover any loss or damage occasioned
Thousand Pesos (P200,000.00) shall be six percent (6%) per annum by explosion and stating that the claim was fraudulent
computed from the date of filing of the Complaint in the trial court.  RTC: ordered Phoenix to pay P13,000
The awards in the amounts of Fifty Thousand Pesos (P50,000.00) as  Phoenix appealed
actual damages, Fifty Thousand Pesos (P50,000.00) as exemplary o The insurance policy contains:

Page 58 of 154
INSURANCE G01 CASE DIGESTS

Unless otherwise expressly stated in the policy the 104. FGU INSURANCE V. COURT OF APPEALS, 454 SCRA 337 (2005)
insurance does not cover
(h) Loss or damage occasioned by the explosion; but loss FGU Insurance vs. Court of Appeals
or damage by explosion of gas for illuminating or domestic
G.R. No. 137775; March 31, 2005
purposes in a building in which gas is not generated and
which does not form a part of any gas works, will be
deemed to be loss by fire within the meaning of this policy.
Facts:
ISSUE:  Anco Enterprises, a partnership between Ang Gui and Co To, was
engaged in the shipping business. It owned the M/T ANCO
W/N Phoenix should be liable for the loss because there was no tugboat and the D/B Lucio barge. Since the D/B Lucio had no
explosion which is an exemption from the policy engine of its own, it could not maneuver by itself and had to be
towed by a tugboat for it to move from one place to another.
HELD:
 On Sept. 23, 1979, San Miguel Corporation shipped from
YES. Mandaue City, Cebu on board the D/B Lucio, for towage by M/T
 If it be a fact that the fire resulted from an explosion that ANCO, the following cargoes:
fact, if proven, would be a complete defense, the burden of
the proof of that fact is upon the defendant, and upon that
point, there is a failure of proof  25,000 cases Pale Pilsen and 350
 lower court found as a fact that there was no fraud in the cases Cerveza Negra - consignee SMC’s
insurance, and that the value of the property destroyed by Beer Marketing Division (BMD)-
the fire was more than the amount of the insurance. Estancia Beer Sales Office, Estancia,
Iloilo
 15,000 cases Pale Pilsen and 200
cases Cerveza Negra - consignee SMC’s
BMD-San Jose Beer Sales Office, San
Jose, Antique

 The D/B Lucio was towed by the M/T ANCO all the way from
Mandaue City, Cebu to San Jose, Antique. It arrived at San Jose
at around 1:00 PM of September 30, 1979.

Page 59 of 154
INSURANCE G01 CASE DIGESTS

ANCO, as a partnership, was dissolved hence, on January 26,


1993, SMC filed a second amended complaint which was
 In the afternoon of the same day, the clouds over the area were admitted by the Court impleading the surviving partner, Co To
dark and the waves were already big. The arrastre workers and the Estate of Ang Gui represented by Lucio, Julian and Jaime,
unloading the cargoes of SMC on board the D/B Lucio began to all surnamed Ang.
complain about their difficulty in unloading the cargoes. SMC’s
District Sales Supervisor, Fernando Macabuag, requested ANCO’s
representative to transfer the barge to a safer place because the  ANCO admitted that the cases of beer Pale Pilsen and Cerveza
vessel might not be able to withstand the big waves. Negra mentioned in the complaint were indeed loaded on the
vessel belonging to ANCO. It claimed however that it had an
agreement with SMC that ANCO would not be liable for any
 ANCO’s representative did not heed the request because he was losses or damages resulting to the cargoes by reason of
confident that the barge could withstand the waves. At that fortuitous event. Since the cases of beer Pale Pilsen and Cerveza
time, only the M/T ANCO was left at the wharf of San Jose, Negra were lost by reason of a storm, a fortuitous event which
Antique, as all other vessels already left the wharf to seek battered and sunk the vessel in which they were loaded, they
shelter. With the waves growing bigger and bigger, only 10,790 should not be held liable.
cases of beer were discharged into the custody of the arrastre  ANCO further asserted that there was an agreement between
operator. them and SMC to insure the cargoes in order to recover
indemnity in case of loss. Pursuant to that agreement, the
cargoes to the extent of 20,000 cases was insured with FGU
 At about 10:00 to 11:00 pm of October 1, 1979, the crew of D/B Insurance Corporation (FGU) for the total amount of
Lucio abandoned the vessel because the barge’s rope attached P858,500.00 per Marine Insurance Policy No. 29591.
to the wharf was cut off by the big waves. At around midnight,
the barge run aground and was broken and the cargoes of beer
in the barge were swept away.  In its answer to the Third-Party complaint, FGU admitted the
existence of the Insurance Policy but maintained that the alleged
loss of the cargoes covered by the said insurance policy cannot
 As a result, ANCO failed to deliver to SMC’s consignee 29,210 be attributed directly or indirectly to any of the risks insured
cases of Pale Pilsen and 550 cases of Cerveza Negra. The value against in the said insurance policy. According to FGU, it is only
per case of Pale Pilsen was P42.50. The value of a case of liable under the policy to Third-party Plaintiff ANCO and/or
Cerveza Negra was P47.10, hence, SMC’s claim against ANCO Plaintiff SMC in case of any of the following:
amounted P1,346,197.00.

a) Total loss of the entire shipment;


 SMC filed a complaint for breach of contract of carriage against b) Loss of any case as a result of the sinking of the vessel; or
ANCO for the amount of P1,346,197. Upon Ang Gui’s death, c) Loss as a result of the vessel being on fire.
Page 60 of 154
INSURANCE G01 CASE DIGESTS

thus, making said blatant negligence the proximate cause of the


 Furthermore, FGU alleged that the Third-Party Plaintiff ANCO loss of the cargoes.
and Plaintiff SMC failed to exercise ordinary diligence or the
diligence of a good father of the family in the care and
supervision of the cargoes insured to prevent its loss and/or  One of the purposes for taking out insurance is to protect the
destruction. insured against the consequences of his own negligence and that
of his agents. Thus, it is a basic rule in insurance that the
 The RTC held that while the cargoes were indeed lost due to carelessness and negligence of the insured or his agents
fortuitous event, there was failure on ANCO’s part, through their constitute no defense on the part of the insurer. This rule
representatives, to observe the degree of diligence required that however presupposes that the loss has occurred due to causes
would exonerate them from liability. The trial court thus held which could not have been prevented by the insured, despite the
the Estate of Ang Gui and Co To liable to SMC for the amount of exercise of due diligence.
the lost shipment.
 The question now is whether there is a certain degree of
 With respect to the Third-Party complaint, the RTC found FGU negligence on the part of the insured or his agents that will
liable to 53% of the amount of the lost cargoes. It was upheld by deprive him the right to recover under the insurance contract.
the Court of Appeals. We say there is. However, to what extent such negligence must
go in order to exonerate the insurer from liability must be
Issue: Whether or not FGU can be held liable under the insurance evaluated in light of the circumstances surrounding each case.
policy to reimburse ANCO for the loss of the cargoes despite the When evidence show that the insured’s negligence or
findings of the Court of Appeals that such loss was occasioned by recklessness is so gross as to be sufficient to constitute a willful
the blatant negligence of the latter’s employees. act, the insurer must be exonerated.

Held: NO. The third-party complaint against FGU insurance is  The United States Supreme Court has made a distinction
dismissed but ANCO is still liable to SMC. between ordinary negligence and gross negligence or negligence
amounting to misconduct and its effect on the insured’s right to
Ratio: recover under the insurance contract. According to the Court,
 To be exempt from responsibility, the natural disaster should while mistake and negligence of the master or crew are incident
have been the proximate and only cause of the loss. But, to navigation and constitute a part of the perils that the insurer
according to the Court of Appeals, there was blatant negligence is obliged to incur, such negligence or recklessness must not be
on the part of M/T ANCO’s crewmembers, first in leaving the of such gross character as to amount to misconduct or wrongful
engine-less barge D/B Lucio at the mercy of the storm without acts; otherwise, such negligence shall release the insurer from
the assistance of the tugboat, and again in failing to heed the liability under the insurance contract.
request of SMC’s representatives to have the barge transferred
to a safer place, as was done by the other vessels in the port;

Page 61 of 154
INSURANCE G01 CASE DIGESTS

 In the case at bar, both the trial court and the appellate court
had concluded from the evidence that the crewmembers of both
the D/B Lucio and the M/T ANCO were blatantly negligent. To
wit:

There was blatant negligence on the part of the


employees of defendants-appellants when the patron
(operator) of the tug boat immediately left the barge at
the San Jose, Antique wharf despite the looming bad
weather. Negligence was likewise exhibited by the
defendants-appellants’ representative who did not heed
Macabuag’s request that the barge be moved to a more
secure place. The prudent thing to do, as was done by
the other sea vessels at San Jose, Antique during the
time in question, was to transfer the vessel to a safer
wharf. The negligence of the defendants-appellants is
proved by the fact that on 01 October 1979, the only
simple vessel left at the wharf in San Jose was the D/B
Lucio.

Page 62 of 154
INSURANCE G01 CASE DIGESTS

NOTICE AND PROOF OF LOSS c. As the herein petition was filed on July 2, 1981, or 19
days later, there is no question that it is tardy by four
days.
8. Insurance Commission ruling: in favor of Pinca
105. MALAYAN INSURANCE VS. CRUZ-ARNALDO, 154 SCRA 672 9. Hence this appeal
(1987) a. MICO contends that it should not be held liable
MALAYAN INSURANCE CO., INC. (MICO), petitioner, vs. GREGORIA because:
CRUZ ARNALDO, in her capacity as the INSURANCE i. Adora was not authorized to receive premium
COMMISSIONER, and CORONACION PINCA, respondents. payment because 6 months had elapsed since
the issuance by the policy itself... this
G.R. No. L-67835 October 12, 1987 prohibition was binding upon Pinca, who made
the payment to Adora at her own risk as she
Facts: was bound to first check his authority to receive
1. Jun 7 1981: MICO issued to Coronacion Pinca a Fire Insurance it
Policy for her property effective July 22, 1981 to July 22, 1982 ii. No payment of premium and that the policy had
2. Oct 151981: MICO allegedly cancelled the policy for non- been cancelled before the occurrence of the
payment, of the premium and sent the corresponding notice to loss
Pinca iii. The claim was allowed without sufficient proof
3. Dec 24 1981: payment of the premium for Pinca was received of loss
by Domingo Adora, as agent of MICO
4. Jan 15 1982: Adora remitted this payment to MICO, together
with other payments ISSUE: (We will now focus on the final issue raised by MICO, the
5. Jan 18 1982: Pinca's property was completely burned proper valuation of the loss incurred.) Whether the valuation fixed
6. Feb 5 1982: Pinca's payment was returned by MICO to Adora on based on the certification issued by the Integrated National Police is
the ground that her policy had been cancelled earlier but Adora
sufficient
refused to accept it and instead demanded for payment
7. MICO filed its MR on April 25, 1981 (or 15 days after such
HELD: Yes, petition DENIED.
notice)
a. The reglementary period began to run again after Jun RATIO:
13, 1981 (date of its receipt of notice of the denial of
the said MR) 1. The last point raised by the petitioner should not pose much
b. Under Section 416 of the Insurance Code, the period for difficulty. The valuation fixed in fire insurance policy is
appeal is thirty days from notice of the decision of the
conclusive in case of total loss in the absence of fraud,
Insurance Commission.
which is not shown here.

Page 63 of 154
INSURANCE G01 CASE DIGESTS

a. Loss and its amount may be determined on the proceedings and justify the filing of the petition with this
basis of such proof as may be offered by the Court fourteen days after June 18, 1982. We also look
insured, which need not be of such persuasiveness askance at the alleged cancellation, of which the insured
as is required in judicial proceedings. and MICO's agent himself had no knowledge, and the
b. If, as in this case, the insured files notice and curious fact that although Pinca's payment was remitted to
preliminary proof of loss and the insurer fails to MICO's by its agent on January 15, 1982, MICO sought to
specify to the former all the defects thereof and return it to Adora only on February 5, 1982, after it
without unnecessary delay, all objections to notice presumably had learned of the occurrence of the loss
and proof of loss are deemed waived under Section insured against on January 18, 1982. These circumstances
90 of the Insurance Code. make the motives of the petitioner highly suspect, to say
2. The certification 26 issued by the Integrated National the least, and cast serious doubts upon its candor and bona
Police, Lao-ang, Samar, as to the extent of Pinca's loss fides.
should be considered sufficient. Notably,MICO submitted
no evidence to the contrary nor did it even question the
extent of the loss in its answer before the Insurance
Commission. It is also worth observing that Pinca's property
was not the only building bumed in the fire that razed the
commercial district of Lao-ang, Samar, on January 18, 1982.
3. There is nothing in the Insurance Code that makes the
participation of an adjuster in the assessment of the loss
imperative or indespensable, as MICO suggests. Section
325, which it cites, simply speaks of the licensing and duties
of adjusters.
4. We see in this cases an obvious design to evade or at least
delay the discharge of a just obligation through efforts
bordering on bad faith if not plain duplicity, We note that
the motion for reconsideration was filed on the fifteenth
day from notice of the decision of the Insurance
Commission and that there was a feeble attempt to show
that the notice of denial of the said motion was not
received on June 13, 1982, to further hinder the
Page 64 of 154
INSURANCE G01 CASE DIGESTS

106. YU BAN CHUAN V. FIELDMEN’S INSURANCE , 14 SCRA 491 Adjustment Co. and Manila Adjustment Co., adjusters of defendants
(1965) Fieldmen's and Paramount, respectively, executed "non-waiver"
agreements for the purpose of determining the circumstances of
Yu Ban Chuan vs. Fieldmen's Insurance Co. Inc.
the fire and the value or amount of loss and damage to the
G.R. No. L-19851. June 29, 1965
merchandise insured under said policies.

Facts:
Pursuant to such agreements, H. H. Bayne Adjustment Co. and
On March, 1959, plaintiff Yu Ban Chuan began his business
Manila Adjustment Co. sent letters requiring the plaintiff to submit
enterprise under the name of "CMC Trading," which was engaged in
certain papers and documents. On February
the wholesale dealing of general merchandise and school supplies.
8, 1960, plaintiff gave a written notice of the occurrence of the fire
to the defendants, and, in answer to the letters of the adjusters,
The plaintiff insured against fire the stock merchandise with
plaintiff submitted his separate formal fire claims, together with
defendant Fieldmen's
some of the supporting papers that where required.
Insurance Co. Inc. on December 14, 1959, with an "open" policy
limiting the insurer's liability to the amount of P200,000 for a period
Because of plaintiff's non-compliance or failure to submit the
of one year. The plaintiff again insured against fire the same stock of
required documents the defendants rejected plaintiff's claims, and
merchandise with another company which was Paramount Surety &
denied liability under their respective policies, evidently upon their
Insurance Co., on January 7, 1960, with another "open" policy
respective adjusters' recommendations.
limiting liability thereunder to P140,000 for also a period of one
year.
The plaintiff commenced suit in the Court of First Instance of
Manila. The defendants answered the complaint with identical
Plaintiff transferred his business establishment. Both Fieldmen's and
special defenses; to wit:
Paramount acknowledged the existence of its co-insurance and
both agreed to transfer the coverage of its respective insurance
1) insured's failure to prove the loss claimed;
policies to the plaintiff's new store.
2) false and fraudulent claim; and
3) arson or causes not independent of the will of the insured;
On January 31, 1960, while both insurance policies were in full force
and effect, plaintiff's business establishment was totally destroyed
After trial, the CFI upheld the claim of the plaintiff, but refused to
by fire. The next day after the occurrence of the fire, plaintiff
award damages or interest at more than the legal rate. Both parties
verbally notified the respective agents of the defendants- insurers
appealed. In proving the value of his loss, the plaintiff relied upon a
of such incident; and on the same day, plaintiff and H. H. Bayne
merchandise inventory as of December 31, 1959, which he had
Page 65 of 154
INSURANCE G01 CASE DIGESTS

allegedly submitted on January 15, 1960 to the Bureau of Internal merchandise at the companies' addresses but bought from the
Revenue. agents who brought the goods to him and that the originals of the
invoices were burned and that he requested for true copies from
The inventory reflected the total value of stocks at P328,202.67. The the agents whom he met casually in the streets after the fire and
plaintiff also claimed purchases for the month of January 1960 in these agents delivered the exhibits to him but he did not remember
the amount of P34,505.08 and sales in the amount of P12,000, thus or know the names of these agents, nor did he know their
the resulting balance of the stocks allegedly burned was estimated whereabouts. In other words, he wants the court to believe also
by the plaintiff to be P350,707.75. that these agents performed a vanishing act after each one of them
had turned in the copy of each invoice to the plaintiff.
Issue:
Whether or not plaintiff's appeal for the award of damages at more The plaintiff adhered to the inventory as the immaculate basis for
than the legal rate must be allowed? No. the actual worth of stocks that were burned, on the ground that it
was made from actual count, and in compliance with law. But this
Ratio: inventory is not binding on the defendants, since it was prepared
The fact of the filing of the inventory as of January 15, 1960 should without their intervention.
be considered as true, since there was no evidence to the contrary
that would necessarily show the falsity of the inventory as of the It is well to note that plaintiff had every reason to show that the
date stamped therein as certified by the chief of the administrative value of his stock of goods exceeded the amount of insurance that
division. he carried. The inventory having been made prior to the fire, was no
proof of the existence of these goods at the store when the fire
The court a quo, however, committed error in accepting as true the occurred.
actual existence at the burned premises of the stocks mentioned in
the inventory. Six of the many copies of the invoices submitted by True, there were merchandise that were actually destroyed by fire,
the plaintiff to the adjusters uncover a clear case of fraud and but when fraud is conceived, what is true is subtly hidden by the
misrepresentation as to the liability of Fieldmen's and Paramount's schemer beneath proper and legal appearances, including the
respective policies. preparation of the inventory.

Five of the invoices alone inflate the supposed stocks by Shielding himself under Section 82 of the Insurance Act, the plaintiff
P248,370.00. On sight, the exhibit excites incredulity. The plaintiff, asserts that in submitting his proof of loss he was "not bound to
Yu Ban Chuan, adopted a uniform, too uniform, in fact, to be give such proof as would be necessary in a court of justice". The
believed, explanation for all the invoices, that he did not buy the assertion is correct, but does not give him any justification for
Page 66 of 154
INSURANCE G01 CASE DIGESTS

submitting false proofs. Their falsity is the best evidence of the


fraudulent character and the unmeritoriousness of plaintiff's claim. HELD

For the foregoing reasons, the appealed judgment was hereby (16 cases only and not 66cases)
reversed, and the plaintiff's appeal against the non-award of The fire was seen by a large number of people, and if it be a
damages to him must be dismissed. fact that the plaintiff had 66 cases of piece goods in the building at
the time, it was his duty to have offered the evidence of some
disinterested eyewitness as to the identity of the pieces or particles
remaining of the 50 cases, and of the physical facts, for the purpose
107. GO LU V. YORKSHIRE INSURANCE , 43 PHIL. 633 (1922) of showing that the 50 cases were in the bodega at the time of the
fire.
GO LU V. YORKSHIRE
(Proof of Loss)
Although the original entries in plaintiff's books would be
evidence which should have some weight as to the amount of stock
FACTS
which he had in March, and which he purchased during the months
of April, May, and June, and what he sold during that time, such
1. Go Lu was a merchant buying and selling Bolt goods which he
entries are of but little, if any, value as to the amount of goods
stored in a bodega in Manila.
which he had in the bodega at the time of the fire. In any event,
2. Yorkshire issued insurance for the goods stored in the bodega
they are not sufficient to overcome the absence of any evidence of
3. Unfortunately, the bodega was razed by a fire
the physical facts existing after the fire, and the rule of reason that
4. At the time of the fire, Go Lu claims that there were 66 cases of
the 50 cases of goods would not be consumed and completely
bolt goods stored in the bodega
wiped out of existence, without leaving some evidence of their
5. Yorkshire admit the issuance of the policies but contends that not
destruction, which could be found among the remains and debris in
more than 16 cases were destroyed. It alleges that Go Lu submitted
the building after the fire.
fraudulent proof of the amount of loss.
Here, the facts existing at and after the fire are conclusive
ISSUE
evidence that there were only 16 cases of goods in thebodega at
the time of the fire, and the majority of this court are of the opinion
What is the amount and value of the amount of goods which Go Lu
that plaintiff's claim is not only fraudulent, but that he knew it was
had in the bodega at the time of the fire?
fraudulent at the time it was made, and that, for such reason, he is
not entitled to recover anything.
Page 67 of 154
INSURANCE G01 CASE DIGESTS

108. FINMAN’ S GENERAL ASSURANCE V. COURT OF APPEALS, 361 13. The insured shall give immediate written notice to the
SCRA 214 (2001) Company of any loss, protect the property from further
damage, forthwith separate the damaged and undamaged
FINMAN'S GENERAL ASSURANCE vs. COURT OF APPEALS and personal property, put it in the best possible order, furnish a
USUPHIL complete inventory of the destroyed, damaged, and
undamaged property, showing in detail quantities, costs, actual
Topic: Notice and Proof of Loss cash value and the amount of loss claimed; AND WITHIN SIXTY
DAYS AFTER THE LOSS, UNLESS SUCH TIME IS EXTENDED IN
FACTS: WRITING BY THE COMPANY, THE INSURED SHALL RENDER TO
1. Usuphil obtained a fire insurance policy from Finman's THE COMPANY A PROOF OF LOSS, signed and sworn to by the
Assurance covering certain properties, e.g., office, furniture, insured, stating the knowledge and belief of the insured as to
fixtures, shop machinery and other trade equipment, for the following: the time and origin of the loss, the interest of the
damages and loss arising from fire. insured and of all others in the property, the actual cash value
of each item thereof and the amount of loss thereto, all
2. Usuphil filed a claim for the loss of the insured properties due encumbrances thereon, all other contracts of insurance,
to fire. Finman's appointed Adjuster H.H. Bayne to undertake whether valid or not, covering any of said property, any changes
the valuation and adjustment of the loss. H.H. Bayne then in the title, use, occupation, location, possession or exposures
required Usuphil to file a formal claim and submit proof of loss. of said property since the issuing of this policy by whom and for
what purpose any buildings herein described and the several
3.Usuphil submitted its Sworn Statement of Loss and Formal parts thereof were occupied at the time of loss and whether or
Claim signed by its Manager. It likewise submitted Proof of Loss not it then stood on leased ground, and shall furnish a copy of
signed by its Accounting Manager and countersigned by H.H. all the descriptions and schedules in all policies, and if required
Bayne’s Adjuster. verified plans and specifications of any building, fixtures, or
machinery destroyed or damaged. The insured, as often as may
4. Despite repeated demands by private respondent, Finman's be reasonably required, shall exhibit to any person designated
refused to pay the insurance claim. Hence, Usuphil filed a by the company all that remains of any property herein
complaint against Finman's for the unpaid insurance claim. described, and submit to examination under oath by any person
named by the Company, and subscribe the same; and, as often
5. In its Answer, Finman's maintained that the claim could not as may be reasonably required, shall produce for examination
be allowed because it failed to comply with Policy Condition No. all books of account, bills, invoices, and other vouchers or
13 regarding the submission of certain documents to prove the certified copies thereof if originals be lost, at such reasonable
loss. time and place as may be designated by the Company or its
representative and shall permit extracts and copies thereof to
Policy Condition No. 13 reads: be made.

Page 68 of 154
INSURANCE G01 CASE DIGESTS

that at a meeting between Usuphil and Finman's, the latter


No claim under this policy shall be payable unless the terms of summoned the said manager to reconcile the claims. One who
this condition have been complied with. clothes another with apparent authority as his agent and holds
him to the public as such, cannot later be allowed to deny the
6. RTC ruled in favor of Usuphil. CA affirmed. Hence this present authority of such person to act as his agent when such third
petition. Finman's alleged that complate diregard of the person entered into the contract in good faith and in an honest
required documents communicated by Bayne, through a letter belief that he is such agent.
justified their failure to grant Usuphil's claim.

ISSUE:
WON disallowance of Usuphil’s claim is justified by its failure to 109. PACIFIC TIMBER V. COURT OF APPEALS, 112 SCRA 199 (1982)
submit the required documents in accordance with Policy
Condition No. 13 Topic: Notice and Proof of Loss

HELD: NO Pacific Timber Export Corp vs Court of Appeals

RATIO DECIDENDI: Facts:

According to the facts of this case, after the occurrence of the  March 19, 1963 – Pacific Timber Export Corp secured
fire, Usuphil immediately notified Finman's thereof. Thereafter, temporary insurance from Workmen’s Insurance Company
it submitted the following documents: (1) Sworn Statement of for its exportation of 1,250,000 board feet of Philippine
Loss and Formal Claim and; (2) Proof of Loss. lauan and Apitong logs to be shipped from Diapitan Bay,
Quezon Province to Okinawa and Tokyo, Japan
The submission of these documents constitutes substantial o Cover Note No. 1010 was issued insuring the said
compliance with the above provision. Indeed, as regards the cargo subject to the terms and conditions of the 2
submission of documents to prove loss, substantial, not strict as marine cargo policies issued on April 2, 1963
urged by petitioner, compliance with the requirements will  March 29, 1963 – after the issuance of the cover note but
always be deemed sufficient. before the issuance of the two marine policies, some of the
logs intended to be exported were lost during loading
In any case, Finman itself acknowledged its liability when operations in the Diapitan Bay.
through its Finance Manager, it signed the document indicating  In a letter dated April 4, 1963, Pacific Timber informed the
that the amount due private Usuphil is P842,683.40. insurer about the loss of approximately 32 pieces of logs
 On July 17, 1963, Insurer requested their adjuster to inspect
The contention that such manager was without authority to the loss and assess the damage. The latter reported that the
bind the corporation is without merit. The evidence indicate loss of 30 pieces of logs is not covered by the Policies.

Page 69 of 154
INSURANCE G01 CASE DIGESTS

 The insurer denied the claim on the ground that the therefore no cause for petitioner to lose what is due it as if there
investigation revealed that the entire shipment of logs had been payment of premium.
covered by 2 policies were received in good order at their
point of destination. Loss cannot be considered covered The defense of DELAY cannot be sustained.
under the Cover Note as it became null and void by virtue of
the issuance of the marine policies. The law requires this ground of delay to be promptly and specifically
asserted when a claim on the insurance agreement is made.

Issue: Whether or not the Insurance Company was correct in Instead of invoking the ground of delay in objecting to petitioner’s
denying the claim based on the Cover Note issued without claim of recovery on the Note, it took steps clearly indicative that
consideration? his particular ground for objection to the claim was never in its
mind.
Held: NO.
The nature of the round places the insurer on duty to inquire when
The Cover Note was not without consideration.
the loss took place, so that it could determine whether delay would
The fact that no separate premium was paid on the Cover Note be a valid ground.
before the loss insured against occurred, does not militate against
From April 1963 to July 1963, enough time was available for Insurer
the validity of petitioner’s contention for no such premium could
to determine if petitioner was guilty of delay in communicating the
have been paid since by the nature of the Cover Note,, it did not
loss to Insurer
contain, as all Cover Notes do not contain particulars of the
shipment that would serve as basis for the computation of Sec 84 of the Insurance Act provides:
premiums.
Sec. 84 – Delay in the presentation to an insurer of notice or
If the Note is to be treated as a separate policy instead of proof of loss is waived if caused by any act of his or if he omits to
integrating it to the regular policies subsequently issued, the take objection promptly and specifically upon that ground.
purpose and function of the Cove Note would be set at naught or
rendered meaningless. Waiver can successfully be raised against private respondent

It was not necessary to ask petitioner to pay premium on the Note,


for the loss insured against having already occurred, the more
practical procedure is simply to deduct the premium form the
amount due the petitioner on the Note. The non payment is

Page 70 of 154
INSURANCE G01 CASE DIGESTS

DOUBLE INSURANCE On September 20, 1926, Ulpiano Santa Ana took out another
insurance policy on the house in question for P6,000 in the
"Filipinas, Compania de Seguros, which issued the one-year policy
Exhibit E, upon receiving from said plaintiff the amount of P195 as
110. STA. ANA V. COMMERCIAL UNION ASSURANCE , 55 PHIL. 329 premium thereon.
(1930)
Issue: W/N the notice to the other insurance companies of the fact
Facts
that ulpiano has procured other insurances are enough to not annul
the insurance in her favour.
On the 1st of October, 1925, the plaintiff Ulpiano Santa Ana
took out a three-thousand-peso fire insurance policy on the house Held: no.
in the Phoenix Assurance Company (Exhibit C), and six-thousand-
peso policy in the Guardian Assurance Company, Limited (Exhibit D),
It should be noted that clause three of the "Filipinas" policy drawn
for a period of one year from that date until 4 o'clock in the
up in Spanish, and the english policies issued by the four other
afternoon of October 1, 1926, paying the respective premiums of
companies, provided that any outstanding insurance upon the
P97.50 and P196 to said companies through their duly authorized
whole or a portion of the objects thereby assured must be declared
Philippine agent, Kerr & Company. (Exhibit C and D)
by the insured in writing and he must cause the company to add or
insert it in the policy, without which such policy shall be null and
On November 19, 1925, the plaintiff Ulpiano Santa Ana
avoid, and the insured will not be entitled to indemnity in case of
mortgaged said house to the plaintiff Rafael Garcia for P5,000, for a
loss.lawphil.net
period of two years, the contract being drawn up as a retro sale
(Exhibit A) for the sum of P5,000, and the policies issued by the
Ulpiano Santa Ana maintains that he gave the required notice
Phoenix Assurance Company and the Guardian Assurance Company,
to all the insurance companies; To Kerr & Company through their
Limited, were endorsed to the mortgagee, Rafael Garcia (Exhibits C,
sub-agent, Mariano Morelos; to the Pacific Commercial Company
D, and E). On December 16, 1925, the plaintiff Urpiano Santa Ana
through their employee, Guillermo de Leon; and to the "Filipinas,
reinsured said house with the defendant companies, the Globe and
Compania de Seguros" through their agent, Juan Grey; telling them
Rutgers Fire Insurance Company of New York, and the Commercial
he had paid for other insurance on the same property. But he has
Union Assurance Company, Limited of London, through their
been contradicted in this by all the persons mentioned, and this
common agent duly authorized to represent them in the Philippine
deprives his allegations of probative force, especially considering
Islands, the Pacific Commercial Company, for the amount of P3,000
that such advises or notices, so basic and essential to the existence
each, paying the 90-peso premium due upon each policy, which was
and validity of the policies, must be given in writing as required in
to be effective for one year from the aforementioned date until 4
the noted attached to the four policies above mentioned, and must
o'clock in the afternoon of December 16, 1926 (Exhibits B and B-1).
be given in writing as required in the note attached to the four
policies above mentioned, and must be endorsed upon each of
them, so that in case of necessity, as in the instant one, when a loss
Page 71 of 154
INSURANCE G01 CASE DIGESTS

occurs, the insured may clearly show that he has fulfilled this 111. UNION MANUFACTURING V. PHI’ GUARANTY, 47 SCRA 271
indispensable requisite, since all companies, to which people apply (1972)
for insurance upon property already assured, have an interest in
knowing what other policies issued by other companies the insured
already holds, for the purpose of knowing just what interest the UNION MANUFACTURING CO., INC. VS. PHILIPPINE GUARANTY
applicant has in the preservation of the property, and the care and CO., INC.
precaution to be taken for the prevention of loss. 47 SCRA 271 (G.R. NO. L-27932)
OCTOBER 30, 1972
Without deciding whether notice of other insurance upon the same DOUBLE INSURANCE
property must be given in writing, or whether a verbal notice is
sufficient to render an insurance valid which requires such notice, Petitioner: Republic Bank
Respondent: Philippine Guaranty Co.. Inc.
whether oral or written , we hold that in the absolute absence of
J. Fernando:
such notice when it is one of the conditions specified in the fire
insurance policy, the policy is null and void. FACTS:

On January 12, 1962, the Union Manufacturing Co., Inc. obtained


certain loans from the Republic Bank in the total sum of ₱
415,000.00. To secure the payment thereof, UMC executed real and
chattel mortgage on certain properties.

The Republic Bank procured from the defendant Philippine


Guaranty Co., Inc. an insurance coverage on loss against fire for ₱
500,000.00 over the properties of the UMC, as described in
defendant’s cover note dated September 25, 1962, with the
annotation that loss or damage, if any, under said cover note is
payable to Republic Bank as its interest may appear, subject
however to the printed conditions of said defendant’s Fire
Insurance Policy Form.

On September 6, 1964, a fire occurred in the premises of UMC and


on October 6, 1964, UMC filed its fire claim with the PGC Inc., thru
its adjuster, H.H. Bayne Adjustment Co., which was denied by said
defendant in its letter dated November 26, 1964 on the following
ground: “Policy Condition No. 3 and/or the ‘Other Insurance Clause’

Page 72 of 154
INSURANCE G01 CASE DIGESTS

of the policy was violated because you did not give notice to us of
the other insurance which you had taken from New India for ₱ The insurance contract may be rather onerous, but that in itself
80,000.00. Sincere Insurance for ₱ 25,000.00 and Manila Insurance does not justify the abrogation of its express terms, terms which the
for ₱ 200,000.00 with the result that these insurances of which we insured accepted or adhered to and which is the law between the
became aware of only after the fire, were not endorsed on our contracting parties.
policy.
**lifted digest
ISSUE: Whether Republic Bank can recover.

HELD: Without deciding- whether notice of other insurance upon


the same property must be given in writing, or whether a verbal 112. PIONEER INSURANCE V. YAP, 61 SCRA 426 (1974)
notice is sufficient to render an insurance valid which requires such INSURANCE: Double Insurance
notice, whether oral or written, we hold that in the absolute
absence of such notice when it is one of the conditions specified in PIONEER INSURANCE and SURETY CORPORATION vs OLIVA YAP
the fire insurance policy, the policy is null and void. (Santa Ana vs. (December 19, 1974)
Commercial Union Ass. Co., 55 Phil. 128).
FACTS:
If the insured has violated or failed to perform the conditions of the
contract, and such a violation or want of performance has not been  Oliva Yap was the owner of a store in a 2-storey building in
waived by the insurer, then the insured cannot recover. Courts are Manila where she sells bags and footwear. Chua Soon Poon,
not permitted to make contracts for the parties. The functions and the son-in-law was in charge of the store.
duty of the courts consist simply in enforcing and carrying out the  Yap procured a fire insurance policy from Pioneer Insurance
contracts actually made. and Surety Corp. (25,000 face value) on her stocks and
office furniture. One of the conditions in the policy is that
While it is true, as a general rule, that contracts of insurance are she shall notify the insurer of any other insurance already
construed most favorably to the insured, yet contracts of insurance, effected or subsequently procured, and state or endorse it
like other contracts, are to be construed according to the sense and in the policy. Otherwise the Policy shall be forfeited.
meaning of the terms which the parties themselves have used. If  There was another insurance from Great American
such terms are clear and unambiguous they must be taken and Insurance Company which was noted on Pioneer’s policy
understood in their plain, ordinary and popular sense. and the parties endorsed Great American’s policy.
 Yap took out another fire insurance policy from Federal
The annotation then, must be deemed to be a warranty that the Insurance Company but without notice to and consent of
property was not insured by any other policy. Violation thereof Pioneer.
entitles the insurer to rescind. xxx The materiality of non disclosure
of other insurance policies is not open to doubt.

Page 73 of 154
INSURANCE G01 CASE DIGESTS

 A fire broke out in the building and the store was burned. company to make operative the clause avoiding the
Yap filed an insurance claim but Pioneer refused saying contract, wherever the specified conditions should occur. Its
there’s a violation of terms and conditions. obligations ceased, unless, being informed of the fact, it
 CFI and CA: In favour of Oliva Yap consented to the additional insurance. The obvious purpose
of the aforesaid requirement in the policy is to prevent
over-insurance and thus avert the perpetration of fraud.
ISSUE: WON Pioneer should be absolved from liability on account of The public, as well as the insurer, is interested in preventing
Yap’s violation of co-insurance clause. the situation in which a fire would be profitable to the
insured. According to Justice Story: "The insured has no
Yap argues that Federal merely substituted Great American. right to complain, for he assents to comply with all the
stipulation on his side, in order to entitle himself to the
HELD: YES. Yap violated it. benefit of the contract, which, upon reason or principle, he
has no right to ask the court to dispense with the
 The insurance policy issued by Great American Insurance performance of his own part of the agreement, and yet to
Company duly noted on Pioneer’s Policy co-insurance, bind the other party to obligations, which, but for those
ceased, by agreement of the parties to be recognized by stipulation would not have been entered into."
them as a co-insurance policy. The Court of Appeals says
that Great American policy was substituted by the Federal
Insurance policy for the same amount, and because it was a
mere case of substitution, there was no necessity for its
endorsement on Pioneer policy. There is no evidence to
establish and prove such a substitution. If anything was
substituted for Great American policy, it could only be the
Northwest Insurance policy for the same amount of
P20,000.00. The endorsement shows the clear intention of
the parties to recognize on the date the endorsement was
made (August 29, 1962), the existence of only one co-
insurance, and that is the Northwest Insurance policy, which
according to the stipulation of the parties during the
hearing, was issued on August 20, 1962 and endorsed only
on August 20, 1962. The finding that Great American policy
was substituted by the Federal Insurance policy is
unsubstantiated.
 By the plain terms of the policy, other insurance without the
consent of Pioneer would ipso facto avoid the contract. It
required no affirmative act of election on the part of the
Page 74 of 154
INSURANCE G01 CASE DIGESTS

113. GEAGONIA V. COURT OF APPEALS, 241 SCRA 152 (1995) Petition was denied. Upon reconsideration, he attached a document
Geagonia vs CA “Annex A” admitting that at the time he obtained the policy from
GR 114427, 241 SCRA 152; February 6, 1995 the respondent, the policies by Philippine First Insurance were
already existing. He claims though that nothing in the respondent’s
Topic: Double Insurance: Payment of claim where there is over- policy required him to disclose other subsisting policies and that the
insurance by double insurance respondent’s agent failed to inform him of this, if there was any.
Respondent denied the allegations and claimed he is not entitled to
Facts: the proceeds for violating the condition.
Petitioner owned Norman’s Mart in Agusan del Sur. In The insurance commission reversed and stated the
December 1989, he filed insurance policy (fire insurance) from petitioner had no knowledge that its creditor Cebu Tesing obtained
respondent for Php100,000, valid for one year, covering “Stock-in- the policy from Philippine First Insurance over the goods, and even
trade consisting principally of dry goods such as RTW's for men and benefitted from it upon the fire. Respondent’s motion for
women wear and other usual to assured's business." Under the reconsideration was denied, thus it appealed to the CA. The CA
policy, he noted Mercantile Insurance as co-insurer for Php50,000. reversed the insurance commission and found the petitioner knew
The policy contained the condition (Condition 3) that the insured of the existence of the third policy because it was in his name and
shall notify the company of any insurance/s already affected or he even paid the premiums.
subsequently effected covering the same properties. The stipulation
further states that unless notice be given and the particulars of such Issue:
insurance or insurances be stated therein or endorsed in this policy 1. Did the petitioner actually have knowledge of the third
pursuant to Section 50 of the Insurance Code, all benefits under this policy’s existence thus violating condition 3 of the policy
policy shall be deemed forfeited, provided however, that this with the respondent?
condition shall not apply when the total insurance or insurances in 2. If he had, may he still recover the proceeds?
force at the time of the loss or damage is not more than
P200,000.00. His goods totaled a valued of Php392,130. The market Held:
burned in May 1990 and the petitioner’s stocks were completely 1. YES
destroyed. He filed to claim the insurance proceeds from the 2. NO
respondent, but was denied because the same goods were also
covered by a third insurer (Philippines First Insurance) not disclosed Rationale:
with the respondent. The policies with the third insurer indicate We agree with the Court of Appeals that the petitioner knew of the
petitioner as insured and show that there was even a mortgage prior policies issued by the PFIC. His letter of 18 January 1991 to the
attached over the properties with a loss payable clause to the private respondent conclusively proves this knowledge. His
mortgagee. testimony to the contrary before the Insurance Commissioner and
Petitioner filed with the Insurance Commission for recovery which the latter relied upon cannot prevail over a written admission
of the Php100,000 and attorney’s fees. He also claims that the value madeante litem motam. It was, indeed, incredible that he did not
of the goods actually valued at Php1,000,000 at time of loss. know about the prior policies since these policies were not new or

Page 75 of 154
INSURANCE G01 CASE DIGESTS

original. Policy No. GA-28144 was a renewal of Policy No. F-24758, most strictly against those for whose benefits they are inserted, and
while Policy No. GA-28146 had been renewed twice, the previous most favorably toward those against whom they are intended to
policy being F-24792. operate. The reason for this is that, except for riders which may
Condition 3 of the private respondent's Policy No. F-14622 later be inserted, the insured sees the contract already in its final
is a condition which is not proscribed by law. Its incorporation in the form and has had no voice in the selection or arrangement of the
policy is allowed by Section 75 of the Insurance Code which words employed therein. On the other hand, the language of the
provides that "[a] policy may declare that a violation of specified contract was carefully chosen and deliberated upon by experts and
provisions thereof shall avoid it, otherwise the breach of an legal advisers who had acted exclusively in the interest of the
immaterial provision does not avoid the policy." Such a condition is insurers and the technical language employed therein is rarely
a provision which invariably appears in fire insurance policies and is understood by ordinary laymen.
intended to prevent an increase in the moral hazard. It is commonly With these principles in mind, we are of the opinion that
known as the additional or "other insurance" clause and has been Condition 3 of the subject policy is not totally free from ambiguity
upheld as valid and as a warranty that no other insurance exists. Its and must, perforce, be meticulously analyzed. Such analysis leads us
violation would thus avoid the policy. However, in order to to conclude that (a) the prohibition applies only to double
constitute a violation, the other insurance must be upon same insurance, and (b) the nullity of the policy shall only be to the extent
subject matter, the same interest therein, and the same risk. exceeding P200,000.00 of the total policies obtained.
It must be underscored that unlike the "other insurance" The first conclusion is supported by the portion of the
clauses involved in General Insurance and Surety Corp. vs. Ng Hua or condition referring to other insurance "covering any of the property
in Pioneer Insurance & Surety Corp. vs. Yap, Condition 3 in the or properties consisting of stocks in trade, goods in process and/or
private respondent's policy No. F-14622 does not absolutely declare inventories only hereby insured," and the portion regarding the
void any violation thereof. It expressly provides that the condition insured's declaration on the subheading CO-INSURANCE that the co-
"shall not apply when the total insurance or insurances in force at insurer is Mercantile Insurance Co., Inc. in the sum of P50,000.00. A
the time of the loss or damage is not more than P200,000.00." double insurance exists where the same person is insured by
It is a cardinal rule on insurance that a policy or insurance several insurers separately in respect of the same subject and
contract is to be interpreted liberally in favor of the insured and interest. As earlier stated, the insurable interests of a mortgagor
strictly against the company, the reason being, undoubtedly, to and a mortgagee on the mortgaged property are distinct and
afford the greatest protection which the insured was endeavoring separate. Since the two policies of the PFIC do not cover the same
to secure when he applied for insurance. It is also a cardinal interest as that covered by the policy of the private respondent, no
principle of law that forfeitures are not favored and that any double insurance exists. The non-disclosure then of the former
construction which would result in the forfeiture of the policy policies was not fatal to the petitioner's right to recover on the
benefits for the person claiming, will be avoided, if it is possible to private respondent's policy.
construe the policy in a manner which would permit recovery, as, Furthermore, by stating within Condition 3 itself that such
for example, by finding a waiver for such forfeiture. Stated condition shall not apply if the total insurance in force at the time of
differently, provisions, conditions or exceptions in policies which loss does not exceed P200,000.00, the private respondent was
tend to work a forfeiture of insurance policies should be construed amenable to assume a co-insurer's liability up to a loss not

Page 76 of 154
INSURANCE G01 CASE DIGESTS

exceeding P200,000.00. What it had in mind was to discourage 4. In Jan. 1964, a fire broke out destroying the subject
over-insurance. Indeed, the rationale behind the incorporation of property.
"other insurance" clause in fire policies is to prevent over-insurance 5. Pacific Banking demanded payment from Oriental.
and thus avert the perpetration of fraud. When a property owner
a. Pacific did not present any proof of loss.
obtains insurance policies from two or more insurers in a total
amount that exceeds the property's value, the insured may have an 6. Oriental did not accede to the demand because it was
inducement to destroy the property for the purpose of collecting waiting for the insurance adjuster report.
the insurance. The public as well as the insurer is interested in 7. The insurance adjust notified Pacific Banking that
preventing a situation in which a fire would be profitable to the Paramount had not filed any claim with Oriental yet, nor
insured. submitted proof of loss which is a clear violation of Policy
Condition No.11, and for which reason, determination of
the liability of Oriental could not be had.
114. PACIFIC BANKING CORP. V. COURT OF APPEALS, 168 SCRA 1
(1968) a. Pacific Banking answered the insurance adjuster
saying that it can get a proof of loss by verifying
Paras, J. from the records of the Bureau of Customs the
entries of merchandise taken into the customs
Facts bonded warehouse razed by fire.
1. An open fire policy was issued to Paramount Shirt by 8. But still, Oriental refused to pay.
insurer, Oriental Assurance. 9. Pacific Banking filed an action for sum of money against
a. Oriental bound itself to indemnify the insured for Oriental – P61,000.
any loss or damage, not exceeding P61,000.00, 10. Oriental raised the following defenses:
caused by fire to its property consisting of stocks, a. lack of formal claim by insured over the loss and
materials and supplies usual to a shirt factory from b. premature filing of the suit as neither plaintiff nor
Oct. 21, 1965 to Oct. 21, 1964 insured had submitted any proof of loss on the basis
2. Paramount Shirt owed money from Pacific Banking in the of which defendant would determine its liability and
amount of P800,000. As security, the goods insured was the amount thereof
held by Pacific Banking in trust. 11. During the trial, Pacific presented in evidence a document
3. Said policy was duly endorsed to petitioner as mortgagee/ revealing undeclared co-insurances with 6 other insurance
trustor of the properties insured, with the knowledge and companies taken by Paramount covering the same goods
consent of Oriental to the effect that "loss if any under this destroyed.
policy is payable to the Pacific Banking Corporation". a. It will be noted that the defense of fraud and/or
violation of Condition No. 3 in the Policy, in the
Page 77 of 154
INSURANCE G01 CASE DIGESTS

form of non-declaration of co-insurances which was deception. Otherwise stated, had the insurer known that
not pleaded in the answer was also not pleaded in there were many co-insurances, it could have hesitated or
the Motion to Dismiss. plainly desisted from entering into such contract. Hence,
12. RTC ruled in favor of Pacific Banking. Oriental should pay. the insured was guilty of clear fraud (Rollo, p. 25).
13. CA reversed the decision.  Petitioner's contention that the allegation of fraud is but a
14. Hence, this petition. mere inference or suspicion is untenable. In fact, concrete
evidence of fraud or false declaration by the insured was
furnished by the petitioner itself when the facts alleged in
Issues
the policy under clauses "Co-Insurances Declared" and
1. Whether or not the non-declaration of co-insurances "Other Insurance Clause" are materially different from the
violates the policy condition which required the insured to actual number of co-insurances taken over the subject
reveal other insurances already effected. property.
a. Yes.
2. Whether or not failure of the insured to file the required  On the question of Pacific Banking’s right to the proceeds
proof of loss prior to court action. as the mortgagee, the SC held that:
a. Yes. o It is but fair and just that where the insured who is
primarily entitled to receive the proceeds of the
policy has by its fraud and/or misrepresentation,
Ruling forfeited said right, with more reason Pacific which
is merely claiming as indorsee of said insured,
First Issue cannot be entitled to such proceeds.
 As the insurance policy against fire expressly required that o It will be noted that the fact of fraud was tried by
notice should be given by the insured of other insurance express or at least implied consent of the parties.
upon the same property, the total absence of such notice Pacific did not only object to the introduction of
nullifies the policy. evidence but on the contrary, presented the very
 It is not disputed that the insured failed to reveal before the evidence that proved its existence.
loss three other insurances. As found by the Court of Second Issue
Appeals, by reason of said unrevealed insurances, the
insured had been guilty of a false declaration; a clear  Generally, the cause of action on the policy accrues when
misrepresentation and a vital one because where the the loss occurs, But when the policy provides that no action
insured had been asked to reveal but did not, that was shall be brought unless the claim is first presented
Page 78 of 154
INSURANCE G01 CASE DIGESTS

extrajudicially in the manner provided in the policy, the together with the preliminary submittal of relevant
cause of action will accrue from the time the insurer finally documents had not been complied with, it follows that
rejects the claim for payment (Eagle Star Insurance v. Chia private respondent could not be deemed to have finally
Yu, 55 Phil 701 [1955]). rejected petitioner's claim and therefore the latter's cause
 In the case at bar, policy condition No. 11 specifically of action had not yet arisen.
provides that the insured shall on the happening of any loss
or damage give notice to the company and shall within SC affirms CA. Petition dismissed.
fifteen (15) days after such loss or damage deliver to the
private respondent (a) a claim in writing giving particular
account as to the articles or goods destroyed and the
amount of the loss or damage and (b) particulars of all other
insurances, if any. Likewise, insured was required "at his
own expense to produce, procure and give to the company
all such further particulars, plans, specifications, books,
vouchers, invoices, duplicates or copies thereof, documents,
proofs and information with respect to the claim". (Record
on Appeal, pp. 18-20).
 The evidence adduced shows that twenty-four (24) days
after the fire, petitioner merely wrote letters to private
respondent to serve as a notice of loss, thereafter, the
former did not furnish the latter whatever pertinent
documents were necessary to prove and estimate its loss.
Instead, petitioner shifted upon private respondent the
burden of fishing out the necessary information to ascertain
the particular account of the articles destroyed by fire as
well as the amount of loss. It is noteworthy that private
respondent and its adjuster notified petitioner that insured
had not yet filed a written claim nor submitted the
supporting documents in compliance with the requirements
set forth in the policy. Despite the notice, the latter
remained unheedful. Since the required claim by insured,
Page 79 of 154
INSURANCE G01 CASE DIGESTS

REINSURANCE The funds were supposed to be their contributions to a new


corporation proposed by Lim to expand his airline business.
They executed two (2) separate indemnity agreements
115. PIONEER INSURANCE V. COURT OF APPEALS, 175 SCRA 668 (Exhibits D-1 and D-2) in favor of Pioneer, one signed by
(1989) Maglana and the other jointly signed by Lim for SAL,
PIONEER INSURANCE & SURETY CORP V CA, BORMAHECO, Bormaheco and the Cervanteses.
MAGLANA AND LIM G.R. No. 84197  indemnity agreements stipulated that the indemnitors
principally agree and bind themselves jointly and severally
LIM V CA, PIONEER INSURANCE AND SURETY CORP, BORMAHECO, to indemnify and hold and save harmless Pioneer from and
CERVANTES AND MAGLANA against any/all damages, losses, costs, damages, taxes,
penalties, charges and expenses of whatever kind and
FACTS: nature which Pioneer may incur in consequence of having
 2 consolidated petitions become surety
 1965 – Jacob S. Lim was the owner-operator of Southern Air  June 10, 1965 - Lim doing business under the name and
Lines (SAL) a single proprietorship style of SAL executed in favor of Pioneer as deed of chattel
 May 17, 1985 at Tokyo, Japan, Japan Domestic Airlines (JDA) mortgage as security for the latter's suretyship in favor of
and Lim entered into and executed a sales contract for the the former. It was stipulated therein that Lim transfer and
sale and purchase of and purchase of two (2) DC-3A Type convey to the surety the two aircrafts. The deed (Exhibit D)
aircrafts and one (1) set of necessary spare parts for the was duly registered with the Office of the Register of Deeds
total agreed price of US $109,000.00 to be paid in of the City of Manila and with the Civil Aeronautics
installments. One DC-3 Aircraft with Registry No. PIC-718, Administration pursuant to the Chattel Mortgage Law and
arrived in Manila on June 7,1965 while the other aircraft, the Civil Aeronautics Law (Republic Act No. 776),
arrived in Manila on July 18,1965. respectively.
 On May 22, 1965, Pioneer Insurance and Surety Corporation  Lim defaulted on his subsequent installment payments
(Pioneer, petitioner in G.R. No. 84197) as surety executed prompting JDA to request payments from the surety.
and issued its Surety Bond No. 6639 (Exhibit C) in favor of Pioneer paid a total sum of P298,626.12.
JDA, in behalf of its principal, Lim, for the balance price of  Pioneer then filed a petition for the extrajudicial foreclosure
the aircrafts and spare parts. of the said chattel mortgage before the Sheriff of Davao
 It appears that Border Machinery and Heavy Equipment City. The Cervanteses and Maglana, however, filed a third
Company, Inc. (Bormaheco), Francisco and Modesto party claim alleging that they are co-owners of the aircrafts,
Cervantes (Cervanteses) and Constancio Maglana  On July 19, 1966, Pioneer filed an action for judicial
(respondents in both petitions) contributed some funds foreclosure with an application for a writ of preliminary
used in the purchase of the above aircrafts and spare parts.
Page 80 of 154
INSURANCE G01 CASE DIGESTS

attachment against Lim and respondents, the Cervanteses, In general a reinsurer, on payment of a loss
Bormaheco and Maglana. acquires the same rights by subrogation as are
acquired in similar cases where the original insurer
ISSUES/HELD/RATIO: pays a loss (Universal Ins. Co. v. Old Time Molasses
Co. C.C.A. La., 46 F 2nd 925).
G.R. No. 84197-relevant discussion
The rules of practice in actions on original
1. Has Pioneer a cause of action against defendants with respect to
insurance policies are in general applicable to
so much of its obligations to JDA as has been paid with reinsurance actions or contracts of reinsurance. (Delaware, Ins.
money? NO Co. v. Pennsylvania Fire Ins. Co., 55 S.E. 330,126 GA.
2. If the answer to the preceding question is in the negative, has 380, 7 Ann. Con. 1134).
Pioneer still any claim against defendants, considering the amount it
has realized from the sale of the mortgaged properties? NO Hence the applicable law is Article 2207 of the new Civil Code, to
wit:
The total amount paid by Pioneer to JDA is P299,666.29. Since
Pioneer has collected P295,000.00 from the reinsurers, the Art. 2207. If the plaintiffs property has been
uninsured portion of what it paid to JDA is the difference between insured, and he has received indemnity from the
the two amounts, or P3,666.28. This is the amount for which insurance company for the injury or loss arising out
Pioneer may sue defendants, assuming that the indemnity of the wrong or breach of contract complained of,
agreement is still valid and effective. But since the amount realized the insurance company shall be subrogated to the
from the sale of the mortgaged chattels are P35,000.00 for one of rights of the insured against the wrongdoer or the
the airplanes and P2,050.00 for a spare engine, or a total of person who has violated the contract. If the amount
P37,050.00, Pioneer is still overpaid by P33,383.72. Therefore, paid by the insurance company does not fully cover
Pioneer has no more claim against defendants. the injury or loss, the aggrieved party shall be
entitled to recover the deficiency from the person
The payment to the petitioner made by the reinsurers was not causing the loss or injury.
disputed in the appellate court. Considering this admitted payment,
the only issue that cropped up was the effect of payment made by Interpreting the aforesaid provision, we ruled in the case of Phil. Air
the reinsurers to the petitioner. Therefore, the petitioner's Lines, Inc. v. Heald Lumber Co. (101 Phil. 1031 [1957]) which we
argument that the respondents had no interest in the reinsurance subsequently applied in Manila Mahogany Manufacturing
contract as this is strictly between the petitioner as insured and the Corporation v. Court of Appeals (154 SCRA 650 [1987]):
reinsuring company pursuant to Section 91 (should be Section 98) of
the Insurance Code has no basis. Note that if a property is insured and the owner
receives the indemnity from the insurer, it is
provided in said article that the insurer is deemed

Page 81 of 154
INSURANCE G01 CASE DIGESTS

subrogated to the rights of the insured against the remedy of foreclosure of the chattel mortgage both by
wrongdoer and if the amount paid by the insurer extrajudicial foreclosure and the instant suit.
does not fully cover the loss, then the aggrieved
party is the one entitled to recover the deficiency. Also, Pioneer's liability as surety to JDA had already prescribed
Evidently, under this legal provision, the real party in when Pioneer paid the same. Consequently, Pioneer has no more
interest with regard to the portion of the indemnity cause of action to recover from these defendants, as supposed
paid is the insurer and not the insured. (Emphasis indemnitors, what it has paid to JDA. By virtue of an express
supplied). stipulation in the surety bond, the failure of JDA to present its claim
to Pioneer within ten days from default of Lim or SAL on every
It is clear from the records that Pioneer sued in its own name and installment, released Pioneer from liability from the claim.
not as an attorney-in-fact of the reinsurer. Moreover, the Therefore, Pioneer is not entitled to exact reimbursement from
indemnity agreement ceased to be valid and effective after the these defendants thru the indemnity. (Art 1318)
execution of the chattel mortgage. This is judicial admission and
aside from the chattel mortgage there is no other security for the Petition in G.R. No. 84197 is not meritorious.
claim sought to be enforced by this action, which necessarily means
that the indemnity agreement had ceased to have any force and G.R. No. 84157
effect at the time this action was instituted. Sec 2, Rule 129, Revised 1. What legal rules govern the relationship among co-investors
Rules of Court. whose agreement was to do business through the corporate vehicle
but who failed to incorporate the entity in which they had chosen to
The indemnity agreement was ipso jure extinguished upon the
foreclosure of the chattel mortgage. These defendants, as invest?
indemnitors, would be entitled to be subrogated to the right of 2. How are the losses to be treated in situations where their
Pioneer should they make payments to the latter. Articles 2067 and contributions to the intended 'corporation' were invested not
2080 of the New Civil Code of the Philippines. through the corporate form?

Other bases/issues: Principles governing: While it has been held that as between
themselves the rights of the stockholders in a defectively
Independently of the preceding proposition Pioneer's election of
the remedy of foreclosure precludes any further action to recover incorporated association should be governed by the supposed
any unpaid balance of the price. SAL or Lim, having failed to pay the charter and the laws of the state relating thereto and not by the
second to the eight and last installments to JDA and Pioneer as rules governing partners, it is ordinarily held that persons who
surety having made of the payments to JDA, the alternative attempt, but fail, to form a corporation and who carry on business
remedies open to Pioneer were as provided in Article 1484 of the under the corporate name occupy the position of partners inter.
New Civil Code, known as the Recto Law (=p). Pioneer exercised the Thus, where persons associate themselves together under articles
to purchase property to carry on a business, and their organization
Page 82 of 154
INSURANCE G01 CASE DIGESTS

is so defective as to come short of creating a corporation within the 116. GIBSON V. REVILLA, 92 SCRA 219 (1979)
statute, they become in legal effect partners inter se, and their
rights as members of the company to the property acquired by the GIBSON v. REVILLA and LEPANTO CONSOLIDATED MINING
COMPANY
company will be recognized.
FACTS:
Petitioner denied having received any amount from respondents
Bormaheco, the Cervanteses and Maglana. The trial court and the Lepanto Consolidated Mining Company filed a complaint against
appellate court, however, found through Exhibit 58, that the Malayan Insurance Company, Inc. The civil suit thus instituted
petitioner received the amount of P151,000.00 representing the by Lepanto against Malayan was founded on the fact that
participation of Bormaheco and Atty. Constancio B. Maglana in the Malayan issued a Marine Open Policy covering all shipments of
ownership of the subject airplanes and spare parts. The record copper, gold, and silver concentrates in bulk from Poro, San
shows that defendant Maglana gave P75,000.00 to petitioner Jacob Fernando, La Union to Tacoma, Washington or to other places
Lim thru the Cervanteses. in the United States. Thereafter, Malayan obtained reinsurance
abroad through Sedgwick, Collins & Co., Limited, a London
It is therefore clear that the petitioner never had the intention to insurance brokerage. The Memorandum of Insurance issued by
form a corporation with the respondents despite his Sedgwick to Malayan listed three groups of underwriters or
representations to them. This gives credence to the cross-claims of reinsurers – Lloyds 62.808%, Companies (I.L.U.) 34.705%, Other
the respondents to the effect that they were induced and lured by companies 2.487%. At the top of the list of underwriting
the petitioner to make contributions to a proposed corporation members of Lloyds is Syndicate No. 448, assuming 2.48% of the
which was never formed because the petitioner reneged on their risk assumed by the reinsurer, which syndicate number
agreement. petitioner Ivor Robert Dayton Gibson claims to be himself.
Petitioner then filed a motion to intervene as defendant, which
Applying therefore the principles of law earlier cited to the facts of motion was denied by the lower court.
the case, necessarily, no de facto partnership was created among
the parties which would entitle the petitioner to a reimbursement ISSUE: WHETHER OR NOT THE LOWER COURT COMMITTED,
of the supposed losses of the proposed corporation. The record REVERSIBLE ERROR IN REFUSING THE INTERVENTION OF THE
shows that the petitioner was acting on his own and not in behalf of PETITIONER IN THE SUIT BETWEEN LEPANTO AND MALAYAN
his other would-be incorporators in transacting the sale of the COMPANIES.
airplanes and spare parts.
HELD:
DISPOSITIVE: petitions are DISMISSED. Decision of CA AFFIRMED. No. The respondent Judge committed no error of law in denying
petitioner’s Motion to Intervene and neither has he abused his
discretion in his denial of petitioner’s Motion for Intervention.
We agree with the holding of the respondent court that since

Page 83 of 154
INSURANCE G01 CASE DIGESTS

movant Ivor Robert Dayton Gibson appears to be only one of loss strictly within the terms of the original policy has taken
several re-insurers of the risks and liabilities assumed by place. “This clause does not enable the original underwriter to
Malayan Insurance Company, Inc., it is highly probable that recover from his reinsurer to an extent beyond the subscription
other re-insurers may likewise intervene. If petitioner is allowed of the latter. “Wherefore, in view of the foregoing, the petition
to intervene, We hold that there is good and sufficient basis for is hereby dismissed. No costs.”
the Court a quo to declare that the trial between Lepanto and
Malayan would be definitely disrupted and would certainly
unduly delay the proceedings between the parties especially at 117. ARTEX DEVELOPMENT V. WELLINGTON INSURANCE , 51 SCRA
the stage where Lepanto had already rested its case and that 352 (1973)
the issue would also be compounded as more parties and more
matters will have to be litigated. In other words, the Court’s
discretion is justified and reasonable. ARTEX DEVELOPMENT CO INC v. WELLINGTON INSURANCE CO INC

We also hold that respondent Judge committed no reversible 51 SCRA 352


error in further sustaining the fourth ground of Lepanto’s
Opposition to the Motion to Intervene that the rights, if any, of TEEHANKEE; June 27, 1973
petitioner are not prejudiced by the present suit and will be
fully protected in a separate action against him and his co- FACTS
insurers by Malayan. Petitioner’s contention that he has to pay
- Wellington Insurance Co. Inc. insured for P24,346,509.00 the
once Malayan is finally adjudged to pay Lepanto because of the
very nature of a contract of reinsurance and considering that buildings, stocks and machinery of plaintiff Artex Development Co.
the re-insurer is obliged to pay as may be paid thereon Inc. against loss or damage by fire or lighting upon payment of the
(referring to the original policies), although this is subject to plaintiff of the corresponding premiums; that said properties were
other stipulations and conditions of the reinsurance contract, is insured for an additional sum of P883,034.00; that defendant
without merit. insured plaintiff against business interruption (use and occupancy)
for P5,200,000.00; Wellington entered into a contract of
The general rule in the law of reinsurance is that there-insurer is
reinsurance with Alexander and Alexander, Inc. of New York. USA.
entitled to avail itself of every defense which the re-insured
(which is Malayan)might urge in an action by the person
- The buildings, stocks and machineries of plaintiffs spinning
originally insured (which is Lepanto). As to the effect of the
department were burned.
clause “to pay as may be paid thereon” contained in petitioner’s
re-insurance contract, Arnould, on the Law of Marine Insurance
- Notice of the loss and damage was given the defendant; that as
and Average, 13thEd., Vol. 1, Section 327, p. 315, states the
rule, this: “It has been decided that this clause does not per report of the adjusters, the total property loss of the plaintiff
preclude the reinsurer from insisting upon proper proof that a was the sum of P10,106,554.40 and the total business interruption
loss was P3,000,000.00;
Page 84 of 154
INSURANCE G01 CASE DIGESTS

- That defendant has paid to the plaintiff the sum of P6,481,870.07 benefit or favor to the insured, the insured, not being privy to the
of the property loss suffered by plaintiff and P1,864,134.08 on its reinsurance contract, has no cause of action against the reinsurer. It
business interruption loss, leaving a balance of P3,624,683.43 and is expressly provided in Section 91 the Insurance Act 1 that "(T)he
P1,748,460.00, respectively. original insured has no interest in a contract of insurance."

- The counsel for Artex filed a Manifestation saying that in view of


the Deeds of Discharge and Collateral Agreement, the only
remaining liability subject of litigation shall be the proportion of the
loss reinsured with or through Alexander and Alexander, Inc. of New
York, USA, namely, P397,813.00.

- The document recited further that Artex acknowledges receipt of


the sum of P3.6M paid by the insurer in full and final settlement of
all or any claims of Artex against its insurer. It discharges its insurer
from all actions, proceedings, claims, demands, costs and expenses
in respect thereof.

- With regard the balance unpaid, Wellington contends that Artex


should have been directed against the reinsurers to cover the
liability and not against Wellington.

ISSUE

WON the insured (Artex) has a cause of action against the reinsurer

HELD

NO

- Unless there is a specific grant in, or assignment of, the


reinsurance contract in favor of the insured or a manifest intention
of the contracting parties to the reinsurance contract to grant such

Page 85 of 154
INSURANCE G01 CASE DIGESTS

Respondent ignored the petitioner demand for payment of ₱


150,000.00 for the loss of the shipment plus ₱ 100,000.00 as
MARINE INSURANCE unrealized profits.
Respondent Pioneer denied the claim of petitioner for the full
118. ROQUE V. INTERMEDIATE APPELLATE COURT, 139 SCRA 596 amount of ₱ 100,000.00 on the ground that its liability depended
(1985) upon the total loss of vessel only.

ROQUE V. INTERMEDIATE APPELLATE COURT, 139 SCRA 596 (1985) The trial court decided in favor of the plaintiff (petitioner).
The appellate court modified the trial courts decision and absolved
Petitioner: Isabela Roque, doing busines under the name and style Pioneer from liability after finding that there was a breach of
of Isabela Roque Timber Enterprises and Ong Chiong implied warranty of seaworthiness on the part of the petitioners
Respondent: Intermediate Appelate Court and Pioneer Insurance and that the loss of the insured cargo was caused by the “perils of
And Surety Corporation the ship” and not by the “perils of sea.” It ruled that the loss is not
covered by the marine insurance policy.

FACTS: ISSUE:
Manila Bay Lighterage Corporation (Manila Bay) a common carrier, Whether or not the implied warranty of seaworthiness in marine
entered into a contract with petitioners whereby the former would insurance attaches to the shipper who is not the shipowner.
load and cary on board its barge Marble 10 about 422.18 cubic
meters of logs from Malampaya Sound, Palawan to North Harbor HELD:
Manila. The petitioners insured the logs against loss for ₱ Section 113 of the Insurance Code provides:
100,000.00 with respondent Pioneer Insurance and Surety In every marine insurance upon a ship or freight, or freightage, or
Corporation (Pioneer). upon any thing which is the subject of marine insurance, a warranty
is implied that the ship is seaworthy.
The petitioner loaded on the barge, 811 pieces of logs at Section 99 of the same Code also provides in part.
Malampaya Sound, Palawan for carriage and delivery to North Marine insurance includes:
Harbor, Port of Manila, but the shipment never reached its (1) Insurance against loss of or damage to:
destination because Marble 10 sank with the 811 pieces of logs (a) Vessels, craft, aircraft, vehicles, goods, freights, cargoes,
somewhere off Cabuli Point in Palawan on its way to Manila. As merchandise, …
alleged by the petitioners in their complaint and as found by both From the above-quoted provisions, there can be no mistaking the
the trial and appellate courts, the barge where the logs were loaded fact that the term “cargo” can be the subject of marine insurance
was not seaworthy such that it developed a leak. The appellate and that once it is so made, the implied warranty of seaworthiness
court further found that one of the hatches was left open causing immediately attaches to whoever is insuring the cargo whether he
water to enter the barge and because the barge was not provided be the shipowner or not.
with the necessary cover or tarpauline, the ordinary splash of
seawaves brought more water inside the barge.
Page 86 of 154
INSURANCE G01 CASE DIGESTS

Moreover, the fact that the unseaworthiness of the ship was On the contention of the petitioners that the trial court found that
unknown to the insured is immaterial in ordinary marine insurance the loss was occasioned by the perils of the sea characterized by the
and may not be used by him as a defense in order to recover on the “storm and waves” which buffeted the vessel, the records show that
marine insurance policy. the court ruled otherwise. It stated: “x x x The other affirmative
defense of defendant Lighterage, That the supposed loss of the logs
Since the law provides for an implied warranty of seaworthiness in was occasioned by force majeure was not supported by the
every contract of ordinary marine insurance, it becomes the evidence. At the time Mable 10 sank, there was no typhoon but
obligation of a cargo owner to look for a reliable common carrier ordinary strong wind and waves, a condition which is natural and
which keeps its vessels in seaworthy condition. The shipper of cargo normal in the open sea. The evidence shows that the sinking of
may have no control over the vessel but he has full control in the Mable 10 was due to improper loading of the logs on one side so
choice of the common carrier that will transport his goods. Or the that the barge was tilting on one side and for that it did not navigate
cargo owner may enter into a contract of insurance which on even keel; that it was no longer seaworthy that was why it
specifically provides that the insurer answers not only for the perils developed leak; that the personnel of the tugboat and the barge
of the sea but also provides for coverage of perils of the ship. committed a mistake when it turned loose the barge from the
tugboat east of Cabuli point where it was buffeted by storm and
There is no doubt that the term ‘perils of the sea’ extends only to waves, while the tugboat proceeded to west of Cabuli point where
losses caused by sea damage, or by the violence of the elements, it was protected by the mountain side from the storm and waves
and does not embrace all losses happening at sea. They insure coming from the east direction. x x x”
against losses from extraordinary occurrences only, such as stress of
weather, winds and waves, lightning, tempests, rocks and the like. lt must be considered to be settled, furthermore, that a loss which,
These are understood to be the ‘perils of the sea’ referred in the in the ordinary course of events, results from the natural and
policy, and not those ordinary perils which every vessel must inevitable action of the sea, from the ordinary wear and tear of the
encounter. ‘Perils of the sea’ has been said to include only such ship, or from the negligent failure of the ship’s owner to provide the
losses as are of extraordinary nature, or arise from some vessel with proper equipment to convey the cargo under ordinary
overwhelming power, which cannot be guarded against by the conditions, is not a peril of the sea. Such a loss is rather due to what
ordinary exertion of human skill and prudence. Damage done to a has been aptly called the ‘peril of the ship.’ ‘The insurer undertakes
vessel by perils of the sea includes every species of damages done to insure against perils of the sea and similar perils, not against
to a vessel at sea, as distinguished from the ordinary wear and tear perils of the ship. As was well said by Lord Herschell in Wilson, Sons
of the voyage, and distinct from injuries suffered by the vessel in & Co. v. Owners of Cargo per the Xantho ([1887], 12 A. C., 503, 509),
consequence of her not being seaworthy at the outset of her there must, in order to make the insurer liable, be ’some casualty,
voyage (as in this case). It is also the general rule that everything something which could not be foreseen as one of the necessary
which happens thru the inherent vice of the thing, or by the act of incidents of the adventure. The purpose of the policy is to secure an
the owners, master or shipper, shall not be reputed a peril, if not indemnity against accidents which may happen, not against events
otherwise borne in the policy. which must happen.

Page 87 of 154
INSURANCE G01 CASE DIGESTS

Barratry as defined in American Insurance Law is “any willful 119. GO TIACO V. UNION INSURANCE OF CANADA, 40 PHIL. 40
misconduct on the part of master or crew in pursuance of some (1919)
unlawful or fraudulent purpose without the consent of the owners,
and to the prejudice of the owner’s interest,” (Sec. 171, U.S. GO TIACO v. UNION INSURANCE
Insurance Law, quoted in Vance, Handbook on Law of Insurance,
1951, p. 929.) Barratry necessarily requires a willful and intentional FACTS
act in its commission. No honest error of judgment or mere • Union Insurance Society of Canton, Ltd., issued a marine
negligence, unless criminally gross, can be barratry. (See Vance on insurance policy upon a cargo of rice belonging to the Go Tiaoco
Law of Insurance, p. 929 and cases cited therein.) Brothers, which was transported in the early days of May, 1915, on
the steamship Hondagua from the port of Saigon to Cebu.
In the case at bar, there is no finding that the loss was occasioned
by the willful or fraudulent acts of the vessel’s crew. There was only • On discharging the rice from one of the compartments in
simple negligence or lack of skill. Hence, the second assignment of the after hold, upon arrival at Cebu, it was discovered that 1473
error must likewise be dismissed. sacks had been damaged by sea water. The loss was P3,875.25. -
The trial court found that the inflow of the sea water during the
voyage was due to a defect in one of the drain pipes of the ship and
concluded that the loss was not covered by the policy of insurance.

• The trial court made the ff findings: The drain pipe which
served as a discharge from the water closet passed down through
the compartment where the rice in question was stowed and
thence out to sea through the wall of the compartment, which was
a part of the wall of the ship. The joint or elbow where the pipe
changed its direction was of cast iron; and in course of time it had
become corroded and abraded until a longitudinal opening had
appeared in the pipe about one inch in length. This hole had been in
existence before the voyage was begun, and an attempt had been
made to repair it by filling with cement and bolting over it a strip of
iron. The effect of loading the boat was to submerge the vent, or
orifice, of the pipe until it was about 18 inches or 2 feet below the
level of the sea. As a consequence the sea water rose in the pipe.
Navigation under these conditions resulted in the washing out of
the cement-filling from the action of the sea water, thus permitting
the continued flow of the salt water into the compartment of rice.

Page 88 of 154
INSURANCE G01 CASE DIGESTS

• The court found in effect that the opening above described words become most important. X x x" (Thames and Mersey Marine
had resulted in course of time from ordinary wear and tear and not Insurance Co. vs. Hamilton, Fraser & Co.)
from the straining of the ship in rough weather on that voyage. The
court also found that the repairs that had been made on the pipe - a loss which, in the ordinary course of events, results from the
were slovenly and defective and that, by reason of the condition of natural and inevitable action of the sea, from the ordinary wear and
this pipe, the ship was not properly equipped to receive the rice at tear of the ship, or from the negligent failure of the ship's owner to
the time the voyage was begun. For this reason the court held that provide the vessel with proper equipment to convey the cargo
the ship was unseaworthy. under ordinary conditions, is not a peril of the sea. Such a loss is
rather due to what has been aptly called the "peril of the ship." The
• The policy purports to insure the cargo from the following insurer undertakes to insure against perils of the sea and similar
among other risks: "Perils . . . of the seas, men, of war, fire, perils, not against perils of the ship. There must, in order to make
enemies, pirates, rovers, thieves, .jettisons, . . . barratry of the the insurer liable, be "some casualty, something which could not be
master and mariners, and of all other perils, losses, and misfortunes foreseen as one of the necessary incidents of the adventure. The
that have or shall come to the hurt, detriment, or damage of the purpose of the policy is to secure an indemnity against accidents
said goods and merchandise or any part thereof." which may happen, not against events which must happen."
(Wilson, Sons & Co. vs. Owners of Cargo per the Xantho)
ISSUE
WON Union Insurance is liable for the loss of the Go Tiaco Brothers - In the present case the entrance of the sea water into the ship's
hold through the defective pipe already described was not due to
HELD any accident which happened during the voyage, but to the failure
of the ship's owner properly to repair a defect of the existence of
NO - the words "all other perils, losses, and misfortunes" are to be which he was apprised. The loss was therefore more analogous to
interpreted as covering risks which are of like kind (ejusdem that which directly results from simple unseaworthiness than to
generis) with the particular risks which are enumerated in the that which results from perils of the sea.
preceding part of the same clause of the contract. ''According to the
ordinary rules of construction, these words must be interpreted - there is no room to doubt the liability of the shipowner for such a
with reference to the words which immediately precede them. They loss as occurred in this case. By parity of reasoning the insurer is not
were no doubt inserted in order to prevent disputes founded on liable; for, generally speaking, the shipowner excepts the perils of
nice distinctions. X x x For example, if the expression 'perils of the the sea from his engagement under the bill of lading, while this is
seas' is given its widest sense the general words have little or no the very peril against which the insurer intends to give protection.
effect as applied to that case. If on the other hand that expression is As applied to the present case it results that the owners of the
to receive a limited construction, as apparently it did in Cullen vs. damaged rice must look to the shipowner for redress and not to the
Butler (5 M. & S., 461), and loss by perils of the seas is to be insurer. The same conclusion must be reached if the question be
confined to loss ex marine tempestatis discrimine, the general discussed with reference to the seaworthiness of the ship. It is
universally accepted that in every contract of insurance upon

Page 89 of 154
INSURANCE G01 CASE DIGESTS

anything which is the subject of marine insurance, a warranty is  Petitioner filed a claim for said loss against respondent
implied that the ship shall be seaworthy at the time of the inception insurance company.
of the voyage. This rule is accepted in our own Insurance Law (Act  Respondent insurance company rejected the claim alleging
No. 2427, sec. 106). It is also well settled that a ship which is that assuming that spillage took place while the goods were
seaworthy for the purpose of insurance upon the ship may yet be in transit, petitioner and his agent failed to avert or
unseaworthy for the purpose of insurance upon the cargo (Act No. minimize the loss by failing to recover spillage from the sea
2427, sec. 106). van, thus violating the terms of the insurance policy sued
upon; and that assuming that the spillage did not occur
Disposition Decision of trial court is affirmed while the cargo was in transit, the said 400 bags were
loaded in bad order, and that in any case, the van did not
carry any evidence of spillage.
120. CHOA TIEK SENG V. COURT OF APPEALS, 183 SCRA 223 (1990)  Hence, petitioner filed the complaint in the RTC of Manila
against respondent insurance company seeking payment. In
Choa Tiek Seng v Court of Appeals, 183 SCRA 223 (1990) its answer, respondent insurance company denied all the
material allegations of the complaint and raised several
Facts: special defenses as well as a compulsory counterclaim.
Respondent insurance company filed a third-party
 Petitioner imported some lactose crystals from Holland. The complaint against respondents Ben Lines and broker.
importation involved fifteen (15) metric tons packed in 600 Respondent broker filed its answer to the third-party
6-ply paper bags with polythelene inner bags, each bag at complaint denying liability and arguing, among others, that
25 kilos net. The goods were loaded at the port at the petitioner has no valid cause of action against it.
Rotterdam in sea vans on board the vessel "MS Benalder' as Similarly, Ben Lines filed its answer denying any liability and
the mother vessel, and thereafter aboard the feeder vessel a special defense arguing that respondent insurance
"Wesser Broker V-25" of respondent Ben Lines Container, company was not the proper party in interest and has no
Ltd. (Ben Lines for short). The goods were insured by the connection whatsoever with Ben Lines Containers, Ltd. and
respondent Filipino Merchants' Insurance Co., Inc. that the third-party complaint has prescribed under the
(insurance company for short) against all risks under the applicable provisions of the Carriage of Goods by Sea Act.
terms of the insurance cargo policy. Upon arrival at the  Respondent Ben Lines filed a motion for preliminary hearing
port of Manila, the cargo was discharged into the custody of on the affirmative defense of prescription. The trial court
the arrastre operator respondent E. Razon, Inc. (broker for deferred resolution of the aforesaid motion after trial on
short), prior to the delivery to petitioner through his broker. the ground that the defense of prescription did not appear
Of the 600 bags delivered to petitioner, 403 were in bad to be indubitable.
order. The surveys showed that the bad order bags suffered  After the pre-trial conference and trial on the merits, on
spillage and loss. March 31, 1986, the court a quo rendered a judgment

Page 90 of 154
INSURANCE G01 CASE DIGESTS

dismissing the complaint, the counterclaim and the third- every specie of damage done to the ship or goods at sea by the
party complaint with costs against the petitioner. violent action of the winds or waves. They do not embrace all loses
 Hence, the appeal to the Court of Appeals by petitioner happening on the sea. A peril whose only connection with the sea is
which, in due course, as aforestated, affirmed the judgment that it arises aboard ship is not necessarily a peril of the sea; the
of the trial court. peril must be of the sea and not merely one accruing on the sea.
 A motion for reconsideration of said judgment was denied
by the appellate court in a resolution. Moreover, the cargo in question was insured in an "against all risk
 Petitioner now filed this petition for review on certiorari in policy." Insurance "against all risk" has a technical meaning in
this Court. marine insurance. Under an "all risk" marine policy, there must be a
general rule be a fortuitous event in order to impose liability on the
ISSUES: insurer; losses occasioned by ordinary circumstances or wear and
tear are not covered, thus, while an "all risk" marine policy purports
WON court erred in holding that an "all risks" coverage covers only to cover losses from casualties at sea, it does not cover losses
losses occasioned by or resulting from "extra and fortuitous events" occasioned by the ordinary circumstances of a voyage, but only
despite the clear and unequivocal definition of the term made and those resulting from extra and fortuitous events.
contained in the policy sued upon.
It has been held that damage to a cargo by high seas and other
Held: YES. The decision appealed from is hereby REVERSED AND SET weather is not covered by an "all risk" marine policy, since it is not
ASIDE. Respondent Filipinas Merchants Insurance Company, Inc. is fortuitous, particularly where the bad weather occurs at a place
liable. where it could be expected at the time in question.

There is no question that the 403 bags in damaged condition The Court disagrees.
delivered and received by petitioner.
In Gloren Inc. vs. Filipinas Cia. de Seguros, it was held that an all
Nevertheless, on the assumption that the cargo suffered damages, risk insurance policy insures against all causes of conceivable loss
the appellate court ruled: Even assuming that the cargo indeed or damage, except as otherwise excluded in the policy or due to
sustained damage, still the appellant cannot hold the appellee fraud or intentional misconduct on the part of the insured. It
insurance company liable on the insurance policy. In the case at bar, covers all losses during the voyage whether arising from a marine
appellant failed to prove that the alleged damage was due to risks peril or not, including pilferage losses during the war.
connected with navigation. A distinction should be made between
"perils of the sea" which render the insurer liable on account of the In the present case, the "all risks" clause of the policy sued upon
loss and/or damage brought about thereof and "perils of the ship" reads as follows:
which do not render the insurer liable for any loss or damage. Perils
of the sea or perils of navigation embrace all kinds of marine 5. This insurance is against all risks of loss or
casualties, such as shipwreck, foundering, stranding, collision and damage to the subject matter insured but shall in

Page 91 of 154
INSURANCE G01 CASE DIGESTS

no case be deemed to extend to cover loss, 121. FILIPINO MERCHANTS INS. V. COURT OF APPEALS, 179 SCRA
damage, or expense proximately caused by delay or 638 (1989)
inherent vice or nature of the subject matter
insured. Claims recoverable hereunder shall be Marine Insurance
payable irrespective of percentage.
Filipino Merchants Insurance Co., Inc v Court of Appeals
The terms of the policy are so clear and require no interpretation.
The insurance policy covers all loss or damage to the cargo except FACTS:
those caused by delay or inherent vice or nature of the cargo
insured. It is the duty of the respondent insurance company to 1. The consignee of the shipment of fishmeal loaded on board the
establish that said loss or damage falls within the exceptions vessel SS Bougainville and unloaded at the Port of Manila on or
provided for by law, otherwise it is liable therefor. about December 11, 1976 and seeks to recover from the defendant
insurance company the amount of P51,568.62 representing
An "all risks" provision of a marine policy creates a special type of damages to the said shipment.
insurance which extends coverage to risks not usually
contemplated and avoids putting upon the insured the burden of
establishing that the loss was due to peril falling within the policy's
coverage. The insurer can avoid coverage upon demonstrating that 2. The defendant brought a party against third party
a specific provision expressly excludes the loss from coverage. defendants Compagnie Maritime Des Chargues Reunis and/or E.
Razon Inc seeking judgment against the third defendants.
In this case, the damage caused to the cargo has not been
attributed to any of the exceptions provided for nor is there any 3. Evidence shows that the plaintiff insured said shipment with the
pretension to this effect. Thus, the liability of respondent defendant insurance company under the said cargo Policy No. M-
insurance company is clear.
2678 for the sum of P267,653.59 for the 600 metric tons of fishmeal
I new gunny bags of 90 kilos each from Thailand to Manila
against all risks under warehouse to warehouse terms. What was
delivered was only 59.940 metric tons not 600 tons at $395.42 a
ton.

4. The fishmeal in 666 new gunny bags were unloaded from the ship
on December 11, 1976 at Manila unto the arrastre contractor E.
Razon, Inc. and defendant's surveyor ascertained and certified
that in such discharge 105 bags were in bad order condition as

Page 92 of 154
INSURANCE G01 CASE DIGESTS

jointly surveyed by the ship's agent and the arrastre contractor. inconsistent with the broad protective purpose of "all risks"
The condition of the bad order was proven in the survey report. insurance.

5. The cargo was also surveyed by the arrastre contractor before the In the present case, there being no showing that the loss was
delivery and a total of 227 bags were in bad order condition. caused by any of the excepted perils, the insurer is liable under
the policy.
6. The Filipino Merchants Insurance Company refused to pay
the claim when the consignee made a formal claim which There is no evidence presented to show that the condition of the
amounted to P51,568.62. gunny bags in which the fishmeal was packed was such that they
could not hold their contents in the course of the necessary transit,
7. RTC: In favor of the consignee. Defendant was ordered to pay
much less any evidence that the bags of cargo had burst as the
P51,568.62. result of the weakness of the bags themselves. Had there been such
8. CA: Affirmed the decision of the RTC a showing that spillage would have been a certainty, there may
have been good reason to plead that there was no risk covered by
ISSUE: the policy.

Whether or not the insurer is liable under the "all risks policy''? OTHER TOPIC: INSURABLE INTEREST

HELD: Anent the issue of insurable interest, we uphold the ruling of


the respondent court that private respondent, as consignee of the
RATIONALE:
goods in transit under an invoice containing the terms under "C & F
A marine insurance policy providing that the insurance was to Manila," has insurable interest in said goods.
be "against all risks" must be construed as creating a special
insurance and extending to other risks than are usually
contemplated, and covers all losses except such as arise from the
fraud of the insured. The burden of the insured, therefore, is to
prove merely that the goods he transported have been
lost, destroyed or deteriorated. Thereafter, the burden is
shifted to the insurer to prove that the loss was due to
excepted perils. To impose on the insured the burden of
proving the precise cause of the loss or damage would be

Page 93 of 154
INSURANCE G01 CASE DIGESTS

2. Petitioner states in its complainants that Philipp Brothers


"was the charterer of the vessel MV 'Liliana Dimitrova'
122. PUROMINES V. COURT OF APPEALS, 220 SCRA 281 (1993)] which transported the shipment from Yuzhny USSR to
Manila."
Puromines vs Court of Appeals 3. Petitioner further alleged that the caking and hardening,
wetting and melting, and contamination by rust and dirt of
Facts: the damaged portions of the shipment were due to the
1. Puromines, Inc. (Puromines for brevity) and Makati Agro improper ventilation and inadequate storage facilities of the
Trading, Inc. (not a party in this case) entered into a vessel; that the wetting of the cargo was attributable to the
contract with private respondents Philipp Brothers Oceanic, failure of the crew to close the hatches before and when it
Inc. for the sale of prilled Urea in bulk. rained while the shipment was being unloaded in the Port
2. On or about May 22, 1988, the vessel M/V "Liliana of Manila
Dimitrova" loaded on board at Yuzhny, USSR a shipment of 4. Moreover, in its Opposition to the Motion to Dismiss,
15,500 metric tons prilled Urea in bulk complete and in petitioner said that "[t]he cause of action of the complaint
good order and condition for transport to Iloilo and Manila, arose from breach of contract of carriage by the vessel that
to be delivered to petitioner. was chartered by defendant Philipp Brothers
3. 3 Bill of Ladings were issued. The shipment covered by Bill 5. American jurisprudence defines charter party as a contract
of Lading no. 2 was discharged in Iloilo safely, while those by which an entire ship or some principal part thereof is let
covered by Bill of Ladings Nos. 1 and 3 arrived in Manila in by the owner to another person for a specified time or use.
bad condition. 6. Charter or charter parties are of two kinds. Charter of
4. Damages were valued at P683, 056. 29 including additional demise or bareboat and contracts of affreightment.
discharging expenses. 7. Under the demise or bareboat charter of the vessel, the
5. Petitioner filed a complaint for breach of contract of charterer will generally be considered as owner for the
carriage against Maritime Factors as ship-agent and owner voyage or service stipulated. The charterer mans the vessel
of the vessel, and against Philipp Brothers as charterer of with his own people and becomes, in effect, the owner pro
the vessel. hac vice, subject to liability to others for damages caused by
6. Philipp Brothers filed a motion to dismiss on the ground negligence.
that the complaint stated no cause of action against them. 8. To create a demise the owner of a vessel must completely
and exclusively relinquish possession, anything short of such
Issue: a complete transfer is a contract of affreightment (time or
1. Whether or not petitioner has a cause of action against voyage charter party) or not a charter party at all.
Philipp Brothers 9. A contract of affreightment is in which the owner of the
vessel leases part or all of its space to haul goods for others.
Held: It is a contract for a special service to be rendered by the
1. NO RULING. There is a valid arbitration clause. Court owner of the vessel and under such contract the general
ordered for arbitration as to the issue.
Page 94 of 154
INSURANCE G01 CASE DIGESTS

owner retains the possession, command and navigation of 123. CALTEX (PHILS.) INC. V. SULPICIO LINES, 315 SCRA 709 (1999)
the ship, the charterer or freighter merely having use of the
space in the vessel in return for his payment of the charter Caltex vs. Sulpicio Lines (1999)
hire.
10. If the charter is a contract of affreightment, which leaves Facts:
the general owner in possession of the ship as owner for the 1. Dec. 19, 1987; 8:00 pm: A motor tanker MT Vector (owned
voyage, the rights, responsibilities of ownership rest on the and operated by Vector Shipping Corporation) carried 8,800
owner and the charterer is usually free from liability to third barrels of petroleum products of Caltex by virtue of a
persons in respect of the ship. charter contract. From Bataan, it headed to Masbate.
11. If possession is transferred to the charterer by virtue of a 2. Dec. 20, 1987 6:30 am: MV Doña Paz passenger and cargo
demise, the charterer, and not the owner, is liable as carrier vessel owned and operated by Sulpicio Lines, left the port of
on the contract of affreightment made by himself or by the Tacloban headed for Manila with 1,493
master with third persons, and is answerable for loss, passengers indicated in the Coast Guard Clearance.
damage or non-delivery of goods received for 3. Dec. 20, 1987: MT Vector collided with MV Doña Paz in the
transportation. open sea between Marinduque and Oriental Mindoro,
12. An owner who retains possession of the ship, though the killing almost all the passengers and crew members of both
hold is the property of the charterer, remains liable as ships. Only 24 survived.
carrier and must answer for any breach of duty as to the 4. MV Doña Paz carried an estimated 4,000 passengers most
care, loading or unloading of the cargo. were not in the passenger manifest.
13. Assuming that in the present case, the charter party is a 5. The Board of Marine found that the Vector Shipping
demise or bareboat charter, then Philipp Brothers is liable Corporation was at fault and responsible for the collision
to Puromines, Inc., subject to the terms and conditions of with MV Doña Paz.
the sales contract. On the other hand, if the contract 6. Teresita Cañezal and Sotera Cañezal, Sebastian Cañezal’s
between respondent and the owner of the vessel MV wife and mother filed a complaint for “Damages Arising
"Liliana Dimitrova" was merely that of affreightment, then it from Breach of Contract of Carriage” against Sulpicio Lines
cannot be held liable for the damages caused by the breach for the death of Sebastian Cañezal and his 11-year old
of contract of carriage, the evidence of which is the bills of daughter Corazon Cañezal.
lading 7. Sulpicio Lines filed a 3rd party complaint against Vector
Shipping Corporation and Caltex. It alleged that Caltex
chartered MT Vector with gross and evident bad faith
knowing fully well that MT Vector was improperly manned,
ill-equipped, unseaworthy and a hazard to safe navigation.
8. The RTC dismissed the 3rd party complaint against Caltex.
The CA modified the ruling by including Caltex as one of
those liable for damages.

Page 95 of 154
INSURANCE G01 CASE DIGESTS

Issue: W/N Caltex is liable for damages for the collision between
Vector and Doña Paz
MT Vector is a common carrier. Charter parties fall into three main
Held: NO. Caltex and Vector entered into a contract of categories: (1) Demise or bareboat, (2) time charter, (3) voyage
affreightment, also known as a voyage charter. A charter party is a charter. Does a charter party agreement turn the common carrier
contract by which an entire ship, or some principal part thereof, is into a private one? We need to answer this question in order to
shed light on the responsibilities of the parties. In this case, the
let by the owner to another person for a specified time or use; a
charter party agreement did not convert the common carrier into a
contract of affreightment is one by which the owner of a ship or private carrier. The parties entered into a voyage charter, which
other vessel lets the whole or part of her to a merchant or other retains the character of the vessel as a common carrier.
person for the conveyance of goods, on a particular voyage, in
consideration of the payment of freight.

A contract of affreightment may be either time charter, wherein the Is Caltex liable for damages under the Civil Code? We rule that it is
leased vessel is leased to the charterer for a fixed period of time, not. Sulpicio argues that Caltex negligently shipped its highly
or voyage charter, wherein the ship is leased for a single voyage. In combustible fuel cargo aboard an unseaworthy vessel such as the
both cases, the charter-party provides for the hire of the vessel MT Vector. The charterer of a vessel has no obligation before
transporting its cargo to ensure that the vessel it chartered
only, either for a determinate period of time or for a single or
complied with all legal requirements. The duty rests upon the
consecutive voyage, the ship owner to supply the ship’s store, pay common carrier simply for being engaged in “public service.”
for the wages of the master of the crew, and defray the expenses
for the maintenance of the ship.

Under a demise or bareboat charter on the other hand, the The relationship between the parties in this case is governed by
charterer mans the vessel with his own people and becomes, in special laws. Because of the implied warranty of seaworthiness,
effect, the owner for the voyage or service stipulated, subject to shippers of goods, when transacting with common carriers, are not
liability for damages caused by negligence. expected to inquire into the vessel’s seaworthiness, genuineness of
its licenses and compliance with all maritime laws. To demand
If the charter is a contract of affreightment, which leaves the more from shippers and hold them liable in case of failure exhibits
general owner in possession of the ship as owner for the voyage, nothing but the futility of our maritime laws insofar as the
the rights and the responsibilities of ownership rest on the protection of the public in general is concerned. By the same token,
we cannot expect passengers to inquire every time they board a
owner. The charterer is free from liability to third persons in respect
common carrier, whether the carrier possesses the necessary
of the ship. Caltex is the charterer. The charterer has no liability for papers or that all the carrier’s employees are qualified. Such a
damages under Philippine Maritime laws. practice would be an absurdity in a business where time is always of

Page 96 of 154
INSURANCE G01 CASE DIGESTS

the essence. Considering the nature of transportation business,


passengers and shippers alike customarily presume that common
carriers possess all the legal requisites in its operation. 124. SAN MIGUEL V. HEIRS OF INGUITO, 384 SCRA 87 (2002)

San Miguel vs Heirs of Inguito


GR No. 141716 July 4, 2002
Thus, the nature of the obligation of Caltex demands ordinary
diligence like any other shipper in shipping his cargoes. A cursory Topic: Marine Insurance - Insurable Interest
reading of the records convinces us that Caltex had reasons to
believe that MT Vector could legally transport cargo that time of the Facts:
year.  San Miguel (SMC) entered into a Time Charter Party
Agreement with Julius Ouano, doing business under the
name and style J. Ouano Marine Services. SMC chartered
MV Dona Roberta, owned by Ouano, for a period of 2 years,
Caltex and Vector Shipping Corporation had been doing business for the purpose of transporting SMC's beverage products
since 1985, or for about two years before the tragic incident from Mandaue City plant to various points in Visayas and
occurred in 1987. Past services rendered showed no reason for Mindanao.
Caltex to observe a higher degree of diligence.  The agreement stated that:
o Ouano warrants ownership, title and interest over
the vessel
o that the vessel is put at SMC's disposal; SMC hires
Clearly, as a mere voyage charterer, Caltex had the right to presume the use and service of the vessel
that the ship was seaworthy as even the Philippine Coast Guard o that the owner warrants that the vessel is
itself was convinced of its seaworthiness. All things considered, we seaworthy and in proper, useful and operational
find no legal basis to hold petitioner liable for damages. condition; and that SMC should immediately notify
Ouano in case it finds any defect in the vessel
o that there is no employer-employee relationship
between Ouano (and/or the crew of the vessel) and
SMC; Ouano held SMC free from all claims and
liabilities arising out of the acts of the crew and the
condition of the vessel
o that Ouano undertook to pay all compensation of
all the vessel's crew
o that Ouano shall indemnify SMC for damages and
losses arising from the incompetence and/or
negligence of, and/or the failure to observe the
Page 97 of 154
INSURANCE G01 CASE DIGESTS

required extra-ordinary diligence by the crew; o 11:40pm - power resumed, Moreno made a series
Ouano would automatically be liable for of calls to the vessel but he failed to get in touch
shortlanded shipment, where the value shall be with anyone
withheld from Ouano's collectibles, and wrong  November 13, 1990:
levels, where the value shall likewise be withheld, o 1:15am - Inguito called Moreno over the radio and
but SMC shall reimburse Ouano if SMC's laboratory requested that he contact Rico Ouano, son of the
shall make a determination that the bottles were owner, because they needed a helicopter to rescue
never opened after it left the plant them; vessel was 20 miles west of Sulauan Point.
 SMC issued sailing orders to the master of the vessel, Inguiti requested for a helicopter to rescue them;
Captain Inguito, stating where to sail to, when it is expected the Chief Engineer informed Rico that they can no
to depart and arrive, to maintain communications, and to longer stop the water from coming into the vessel
observe weather condition (exercise utmost precautionary because the crew members were feeling dizzy from
measures) the petroleum fumes
 Inguito obtained the necessary sailing clearance from the o 2:30am - the vessel sank; only 5 of the 25 officers
Philippine Coast Guard. The loading was completed on and crew on board survived
schedule, but the vessel did not leave in accordance with  Ouano, in lieu of the Inguito (who died in the sea tragedy),
the orders. filed a Marine Protest
 November 12, 1990:  the heirs of Inguito filed a complaint for tort against SMC
o 4am - a typhoon was spotted near Samar moving and Ouano with the RTC of Lapu-Lapu City
towards the general direction of Eastern Visayas  Ouano filed a cross-claim: the proximate cause of the loss of
o 6am - the vessel left Mandaue the vessel and of its officers and crew was the fault and
o 7am - while still abeam Cawit Island off Cebu, SMC negligence of SMC, which had complete control and
Radio Operator Moreno contacted Inguito and disposal of the vessel as charterer and which issued a sailing
advised him to take shelter; Inguito replied that order despite being forewarned of the typhoon; he prayed
they would proceed since the typhoon was far away for indemnification of the cost of the vessel and unrealized
from them and that the winds were in their favor rentals and earning
o 4pm - Moreno reiterated to Inguito the advice and  SMC: it was Ouano who had control, supervision and
pointed out that it would be difficult to take shelter responsibilities over the navigation of the vessel; Ouano
after passing Balicasag Island because they were never initiated contact with the vessel despite knowledge of
approaching open sea; Inguito refused to heed the typhoon; the proximate cause was Ouano's breach of
o 8pm - vessel was 38 miles southeast of Balicasag his obligation to provide SMC with a seaworthy vessel duly
o 10pm - vessel was 25 miles approaching Sulauan manned by competent crew members; demanded the value
Point; moments later power went out in Moreno's of the cargo lost in sea
office  RTC: the proximate cause was attributable to SMC; liable for
the loss of earnings of those who died in the sea tragedy,

Page 98 of 154
INSURANCE G01 CASE DIGESTS

moral and exemplary damages for heirs of each deceased  If the charter is a contract of affreightment, which leaves
crew member; liable for the value of the total loss of the the general owner in possession of the ship as owner for the
vessel and unrealized rental earnings voyage, the rights and the responsibilities of ownership rest
 SMC and Ouano appealed to the CA. CA - modified the on the owner. The charterer is free from liability to third
decision: SMC and Ouano are jointly liable to the heirs of persons in respect of the ship
the deceased; the claims of SMC and Ouano against each  the charter between SMC and Ouano was a contract of
other are dismissed affreighment, as evidenced by the the provisions of their
agreement
Issue:  Ouano was the employer, and had command and control
Who should be held liable for the loss over the vessel and the crew
note: i don't know how this involves insurable interest >.< it only  SMC should free from liability for any loss or damage
talks about how the owner shall indemnify the charterer sustained during the voyage, unless it can be shown that
the loss was due to its fault or negligence, which the
Ruling: evidence does not show (attempts of SMC's radio operator
Ouano, the owner of the vessel, is liable to the heirs of the to advise Inguito to take shelter and efforts to keep in
deceased, and to SMC for the loss of their goods contact)
 a charter party is a contract by virtue of which the owner or  Absent any showing of fault or negligence from SMC, Ouano
the agent of a vessel binds himself to transport had the clear duty to ensure the safe carriage and arrival of
merchandise or persons for a fixed price. It has also been goods transported on board its vessels. Ouano also
defined as a contract by virtue of which the owner or the expressly warranted that the vessel was seaworthy.
agent of the vessel leases for a certain price the whole or a  seaworthiness: the sufficiency of the vessel in materials,
portion of the vessel for the transportation of goods or construction, equipment, officers, men, and outfit for the
persons from one port to another trade or service in which it is employed; includes the fitness
 2 types of charter parties: (1) bareboat or demise: the of a ship for its fuel and provisions supply, the quality of its
charterer mans the vessel with his own people; (2) contract officers and crew, and its adaptability for the time of voyage
of affreightment: the owner of the vessel leases part or all proposed
of its space to haul goods for others. It is a contract for  The CA found that the proximate cause of the sinking was
special service to be rendered by the owner of the vessel. the negligence of Inguito
Under such contract the ship owner retains the possession,  Ouano is vicariously liable for the negligent acts of his
command and navigation of the ship, the charterer or employee, Captain Inguito. Under Articles 2176 and 2180 of
freighter merely having use of the space in the vessel in the Civil Code, owners and managers are responsible for
return for his payment of the charter hire. damages caused by the negligence of a servant or an
 2 types of contract of affreightment: time charter and employee, the master or employer is presumed to be
voyage charter negligent either in the selection or in the supervision of that
employee. This presumption may be overcome only by

Page 99 of 154
INSURANCE G01 CASE DIGESTS

satisfactorily showing that the employer exercised the care 125. CALTEX (PHILS.) INC. V. SULPICIO LINES, 315 SCRA 709 (1999)
and the diligence of a good father of a family in the
selection and the supervision of its employee. Caltex vs Sulpicio Lines
 Ouano failed to present proof that he exercised the due
diligence in the selection and supervision of the captain. Facts:
Thus, he is vicariously liable for the loss of lives and
property occasioned by the lack of care and negligence of MV Vector and Caltex entered into a charter contract (contract of
his employee affreightment). On December 20, 1987, motor tanker MV Vector,
carrying petroleum products of Caltex, collided in the open sea with
**as for insurable interest, Section 100, 103, 105 and 106 provide passenger ship MV Doña Paz, causing the death of all but 25 of the
for who has an insurable interest in this case
latter’s passengers. Among those who died were Sebastian Canezal
Section 100. The owner of a ship has in all cases an insurable
interest in it, even when it has been chartered by one who and his daughter Corazon Canezal. On March 22, 1988, the board of
covenants to pay him its value in case of loss; Provided, That in this marine inquiry found that Vector Shipping Corporation was at fault.
case the insurer shall be liable for only that part of the loss which On February 13, 1989, Teresita Cañezal and Sotera E. Cañezal,
the insured cannot recover from the charterer Sebastian Cañezal’s wife and mother respectively, filed with the
Regional Trial Court of Manila a complaint for damages arising from
Sec. 103. The owner of a ship has an insurable interest in expected
breach of contract of carriage against Sulpicio Lines. Sulpicio filed a
freightage which according to the ordinary and probable course of
third-party complaint against Vector and Caltex. The trial court
things he would have earned but for the intervention of a peril
insured against or other peril incident to the voyage. (freightage: dismissed the complaint against Caltex, but the Court of Appeals
the benefit which is to accrue to the owner from the use of the included the same in the liability. Hence, Caltex filed this petition.
vessel in the voyage contemplated, ir the benefit derived from the
employment of the ship)

Sec. 105. One who has an interest in the thing from which profits Issue:
are expected to proceed has an insurable interest in the profits.
Whether or not Caltex, as the charterer of a sea vessel, is liable for
Sec. 106. The charterer of a ship has an insurable interest in it, to damages resulting from a collision between the chartered vessel
the extent that he is liable to be damnified by its loss.
and a passenger ship?

Ruling:

Page 100 of 154


INSURANCE G01 CASE DIGESTS

The Supreme Court ruled in the negative. The Supreme Court ruled being engaged in "public service." The relationship between the
that MT Vector is a common carrier. The charter party agreement parties in this case is governed by special laws. Because of the
did not convert the common carrier into a private carrier. The implied warranty of seaworthiness, shippers of goods, when
parties entered into a voyage charter, which retains the character of transacting with common carriers, are not expected to inquire into
the vessel as a common carrier. It is imperative that a public carrier the vessel’s seaworthiness, genuineness of its licenses and
shall remain as such, notwithstanding the charter of the whole or compliance with all maritime laws. To demand more from shippers
portion of a vessel by one or more persons, provided the charter is and hold them liable in case of failure exhibits nothing but the
limited to the ship only, as in the case of a time-charter or voyage futility of our maritime laws insofar as the protection of the public
charter. It is only when the charter includes both the vessel and its in general is concerned. Such a practice would be an absurdity in a
crew, as in a bareboat or demise that a common carrier becomes business where time is always of the essence. Considering the
private, at least insofar as the particular voyage covering the nature of transportation business, passengers and shippers alike
charter-party is concerned. Indubitably, a ship-owner in a time or customarily presume that common carriers possess all the legal
voyage charter retains possession and control of the ship, although requisites in its operation.
her holds may, for the moment, be the property of the charterer. A
common carrier is a person or corporation whose regular business is
to carry passengers or property for all persons who may choose to
employ and to remunerate him. 16 MT Vector fits the definition of a
common carrier under Article 1732 of the Civil Code.

The public must of necessity rely on the care and skill of common
carriers in the vigilance over the goods and safety of the passengers,
especially because with the modern development of science and
invention, transportation has become more rapid, more
complicated and somehow more hazardous. For these reasons, a
passenger or a shipper of goods is under no obligation to conduct
an inspection of the ship and its crew, the carrier being obliged by
law to impliedly warrant its seaworthiness.

The charterer of a vessel has no obligation before transporting its


cargo to ensure that the vessel it chartered complied with all legal
requirements. The duty rests upon the common carrier simply for

Page 101 of 154


INSURANCE G01 CASE DIGESTS

126. SAN MIGUEL V. HEIRS OF INGUITO, 384 SCRA 87 (2002) SMC Radio Operator Moreno contacted Captain Inguito 3 times
through the radio and advised him to take shelter but Capt. Inquito
G.R. No. 141716. July 4, 2002] did not heed the advice and continued to sail.
SAN MIGUEL CORPORATION, petitioner, vs. HEIRS OF SABINIANO
INGUITO, and JULIUS OUANO, respondents. At 1:15 a.m., November 13, 1990, Captain Inguito called
Moreno over the radio and requested him to contact Rico Ouano,
FACTS: son of Julius Ouano, because they needed a helicopter to rescue
them.
San Miguel Corporation entered into a Time Charter Party
Agreement with Julius Ouano, owner of Ouano Marine Services. At 2:30 a.m. of November 13, 1990, the M/V Doña Roberta
Under the terms of the agreement, SMC will charter the M/V Doña sank. Out of the 25 officers and crew on board the vessel, only five
Roberta owned by Julius Ouano for a period of two years, to survived.
transport SMC’s beverage products from its Mandaue City plant to
various points in Visayas and Mindanao. Shipowner Julius Ouano, in lieu of the captain who perished in
the sea tragedy, filed a Marine Protest.
Pertinent portions of the Time Charter Party Agreement state:
The heirs of the deceased captain and crew, as well as the
1. OWNER [i.e., Ouano] warrants his ownership, title and interest survivors filed a complaint for tort against San Miguel Corporation
over the vessel DOÑA ROBERTA; and Julius Ouano.
4. OWNER warrants that the vessel is seaworthy;
9. There shall be no employer-employee relations between the Julius Ouano alleged that the proximate cause of the loss of the
OWNER and/or its vessel’s crew on one hand and the CHARTERER vessel and its officers and crew was the fault and negligence of SMC
on the other; which issued the sailing order for its departure despite being
10. The OWNER shall undertake to pay all compensation of all the forewarned of the impending typhoon.
vessel’s crew;
11. The OWNER shall be responsible to and shall indemnify the SMC argued that the proximate cause of the sinking was
CHARTERER for damages and losses arising from the incompetence Ouano’s breach of his obligation to provide SMC with a seaworthy
and/or negligence of its crew. vessel duly manned by competent crew members.

During the term of the charter, SMC issued sailing orders to MV RTC --- proximate cause of the loss of the M/V Doña Roberta was
Doña Roberta Captain Inguito to sail on November 12, 1990. attributable to SMC.

Meanwhile, at 4:00 a.m. of November 12, 1990, typhoon Both SMC and Ouano appealed to the Court of Appeals. SMC
Ruping was spotted. argued that as mere charterer, it did not have control of the vessel
and that the proximate cause of the loss of the vessel and its cargo

Page 102 of 154


INSURANCE G01 CASE DIGESTS

was the negligence of the ship captain. For his part, Ouano message. Neither Ouano nor his son was available during the entire
complained of the reduced damages awarded to him by the trial time that the vessel set out and encountered foul weather.
court. Considering that the charter was a contract of affreightment, the
shipowner had the clear duty to ensure the safe carriage and arrival
CA --- SMC and Ouano solidarily liable to plaintiffs heirs and of goods transported on board its vessels. More specifically, Ouano
survivors, except to the heirs of Capt. Sabiniano Inguito expressly warranted in the Time Charter Party that his vessel was
seaworthy.

ISSUE: WHETHER THE WARRANT BY THE SHIPOWNER OF THE


SEAWORTHINESS OF THE M/V DOÑA ROBERTA WAS VIOLATED. WHAT IT MEANS FOR A VESSEL TO BE SEAWORTHY

For a vessel to be seaworthy, it must be adequately equipped


for the voyage and manned with a sufficient number of competent
HELD: YES. officers and crew.[32]

CHARTER PARTY AGREEMENT MAKES SHIPOWNER LIABLE TO SMC DEFINITION OF SEAWORTHINESS

Under the foregoing definitions, as well as the clear terms of Seaworthiness is defined as the sufficiency of the vessel in
the Charter Party Agreement between the parties, the charterer, materials, construction, equipment, officers, men, and outfit, for
SMC, should be free from liability for any loss or damage sustained the trade or service in which it is employed.[33] It includes the fitness
during the voyage,[31] unless it be shown that the same was due to of a ship for a particular voyage with reference to its physical and
its fault or negligence. mechanical condition, the extent of its fuel and provisions supply,
the quality of its officers and crew, and its adaptability for the time
The evidence does not show that SMC or its employees were of voyage proposed.[34]
amiss in their duties.

PROXIMATE CAUSE OF THE SINKING WAS NEGLIGENCE OF CAPTAIN


SHIPOWNER WAS NEGLIGENT INGUITO

In contrast to the care exercised by Moreno, Rico Ouano tried It appears that the proximate cause of the sinking of the vessel
to communicate with the captain only after receiving the S.O.S. was the gross failure of the captain of the vessel to observe due
Page 103 of 154
INSURANCE G01 CASE DIGESTS

care and to heed SMC’s advices to take shelter. He was fully previous occasion, SMC issued a sailing order to the captain of the
apprised of typhoon “Ruping” and its strength. Due diligence M/V Doña Roberta, but the vessel cancelled its voyage due to
dictates that at any time before the vessel was in distress, he should typhoon.[39] Likewise, it appears from the records that SMC issued
have taken shelter in order to safeguard the vessel and its crew. the sailing order on November 11, 1990, before typhoon “Ruping”
was first spotted at 4:00 a.m. of November 12, 1990.[40]
It is very clear that Captain Sabiniano Inguito had sufficient
time within which to secure his men and the vessel. But he waited
until the vessel was already in distress. In fact, there was an incident
when a sailing order was issued by SMC to Inguito but he decided ONLY SHIPOWNER OUANO IS LIABLE FOR THE LOSSES
not to proceed with the voyage because of a tropical storm.[35]
Consequently, Ouano should answer for the loss of lives and
damages suffered by the heirs of the officers and crew members
who perished on board the M/V Doña Roberta, except Captain
OUANO VICARIOUSLY LIABLE UNDER ARTS. 2176 AND 2180 OF THE Sabiniano Inguito. The award of damages granted by the Court of
CIVIL CODE Appeals is affirmed only against Ouano, who should also indemnify
SMC for the cost of the lost cargo, in the total amount of
Under Articles 2176 and 2180 of the Civil Code, owners and P10,278,542.40.[41]
managers are responsible for damages caused by the negligence of
a servant or an employee, the master or employer is presumed to
be negligent either in the selection or in the supervision of that
employee. This presumption may be overcome only by satisfactorily
showing that the employer exercised the care and the diligence of a
good father of a family in the selection and the supervision of its (NOTE: The Time Charter Agreement between SMC and Ouano in
employee.[38] this case was a contract of of affreightment which means that the
shipowner or the captain of the vessel had command and control of
Ouano miserably failed to overcome the presumption of his the vessel at all times. This is in contrast to a bareboat or demise
negligence. charter wherein it is the charterer who mans the vessel with his
own people and therefore had control of the vessel.)
SMC CANNOT BE LIABLE FOR THE LOSSES

However, we cannot sustain the appellate court’s finding that


SMC was likewise liable for the losses. The contention that it was
the issuance of the sailing order by SMC which was the proximate
cause of the sinking is untenable. The fact that there was an
approaching typhoon is of no moment. It appears that on one

Page 104 of 154


INSURANCE G01 CASE DIGESTS

 The shipment was insured with petitioner Philippine


American General Insurance Co., Inc. (PHILAMGEN for
127. PHILIPPINE AMERICAN GENERAL INSURANCE V. CA, 273 SCRA brevity), under Marine Open Policy No. 100367-PAG.
262 (1997)

G.R. No. 116940  "MV Asilda" left the port of Zamboanga in fine weather at
eight o'clock in the evening of the same day. At around
June 11, 1997 eight forty-five the following morning, 7 July 1983, the
vessel sank in the waters of Zamboanga del Norte bringing
THE PHILIPPINE AMERICAN GENERAL INSURANCE COMPANY, INC., down her entire cargo with her including the subject 7,500
petitioner, cases of 1-liter Coca-Cola softdrink bottles.
vs.
COURT OF APPEALS and FELMAN SHIPPING LINES, respondents.
 On 15 July 1983 the consignee Coca-Cola Bottlers
Philippines, Inc., Cebu plant, filed a claim with respondent
Nature:
FELMAN for recovery of damages it sustained as a result of
This case deals with the liability, if any, of a shipowner for loss of the loss of its softdrink bottles that sank with "MV Asilda."
Respondent denied the claim thus prompting the consignee
cargo due to its failure to observe the extraordinary diligence
to file an insurance claim with PHILAMGEN which paid its
required by Art. 1733 of the Civil Code as well as the right of the claim of P755,250.00.
insurer to be subrogated to the rights of the insured upon payment
of the insurance claim.
 Claiming its right of subrogation PHILAMGEN sought
recourse against respondent FELMAN which disclaimed any
liability for the loss. Consequently, on 29 November 1983
Facts: PHILAMGEN sued the shipowner for sum of money and
damages.
 On 6 July 1983 Coca-Cola Bottlers Philippines, Inc., loaded
on board "MV Asilda," a vessel owned and operated by
respondent Felman Shipping Lines (FELMAN for brevity), Issue:
7,500 cases of 1-liter Coca-Cola softdrink bottles to be
transported from Zamboanga City to Cebu City for Whether or not the limited liability under Art. 587 of the Code of
consignee Coca-Cola Bottlers Philippines, Inc., Cebu. Commerce should apply.

Page 105 of 154


INSURANCE G01 CASE DIGESTS

Held: from 29 November 1983, the date of judicial demand,


pursuant to Arts. 2212 and 2213 of the Civil Code.

 No, Art. 587 of the Code of Commerce is not applicable to 128. DELSAN TRANSPORT V. CA, 369 SCRA 24 (2001)
the case at bar. Simply put, the ship agent is liable for the
negligent acts of the captain in the care of goods loaded on Delsan Transport vs. Court of Appeals
the vessel. This liability however can be limited through
abandonment of the vessel, its equipment and freightage as G.R. No. 127897; November 15, 2001
provided in Art. 587. Nonetheless, there are exceptional
circumstances wherein the ship agent could still be held Facts:
answerable despite the abandonment, as where the loss or
 Caltex Philippines entered into a contract of affreightment with
injury was due to the fault of the shipowner and the
the petitioner, Delsan Transport Lines, Inc. for a period of one
captain.
year whereby the petitioner agreed to transport Caltex industrial
fuel oil from Batangas refinery to different parts of the country.
 The international rule is to the effect that the right of
abandonment of vessels, as a legal limitation of a
shipowner's liability, does not apply to cases where the
 Under the contract, petitioner took on board its vessel, M/T
injury or average was occasioned by the shipowner's own
Maysun industrial oil of Caltex to be delivered to the latter’s Oil
fault.
Terminal in Zamboanga City. The shipment was insured with the
private respondent, American Home Assurance Corporation
 It must be stressed at this point that Art. 587 speaks only of
situations where the fault or negligence is committed solely
 On August 14, 1986, MT Maysun set sail for Zamboanga City but
by the captain. Where the shipowner is likewise to be
unfortunately the vessel in the early morning of August 16, 1986
blamed, Art. 587 will not apply, and such situation will be
near Panay Gulf. The shipment was insured with the private
covered by the provisions of the Civil Code on common
respondent, American Home Assurance Corporation.
carrier.
Subsequently, private respondent paid Caltex the sum of
Php.5,096,635.57.
 WHEREFORE, the petition is GRANTED. Respondent
FELMAN SHIPPING LINES is ordered to pay petitioner
 Exercising its right of subrogation under Art. 2207, NCC, the
PHILIPPINE AMERICAN GENERAL INSURANCE CO., INC.,
private respondent demanded from the petitioner the same
Seven Hundred Fifty-five Thousand Two Hundred and Fifty
amount paid to Caltex. Due to its failure to collect from the
Pesos (P755,250.00) plus legal interest thereon counted
petitioner, private respondent filed a complaint with the RTC of

Page 106 of 154


INSURANCE G01 CASE DIGESTS

Makati City but the trial court dismissed the complaint, finding foreclose recourse against the petitioner for any liability under
the vessel to be seaworthy and that the incident was due to a its contractual obligation as a common carrier.
force majeure, thus exempting the petitioner from liability.

 The fact of payment grants the private respondent subrogatory


 However, the decision of the trial court was reversed by the right which enables it to exercise legal remedies that would
Court of Appeals, giving credence to the report of PAGASA that otherwise be available to Caltex as owner of the lost cargo
the weather was normal and that it was impossible for the vessel against the petitioner common carrier.
to sink.

 In order to escape liability for the loss of its cargo of industrial


 In the absence of any explanation as to what may have caused fuel oil belonging to Caltex, petitioner attributes the sinking of
the sinking of the vessel coupled with the finding that the same MT Maysun to fortuitous even or force majeure. But the tale of
was improperly manned, the appellate court ruled that the strong winds and big waves by Captain and the Chief Mate of the
petitioner is liable on its obligation as common carrier to herein said vessel however, was effectively rebutted and belied by the
private respondent insurance company as subrogee of Caltex. weather report from PAGASA. Thus, as the appellate court
correctly ruled, petitioner’s vessel, MT Maysun, sank with its
entire cargo for the reason that it was not seaworthy. There was
Issue: Whether or not the payment made by American Home no squall or bad weather or extremely poor sea condition in the
Assurance to Caltex for the insured value of the lost cargo vicinity when the said vessel sank.
amounted to an admission that the vessel was seaworthy, thus
precluding any action of recovery against Delsan Transport?
 Neither may petitioner escape liability by presenting in
evidence certificates that tend to show that at the time of dry-
docking and inspection by the Philippine Coast Guard, the
Held: NO. Instant Petition is DENIED. vessel MT Maysun, was fit for voyage. These pieces of evidence
do not necessarily take into account the actual condition of the
Ratio: vessel at the time of the commencement of the voyage. As
correctly observed by the Court of Appeals:
 The payment made by the private respondent for the insured
value of the lost cargo operates as waiver of its (private At the time of dry-docking and inspection, the ship may
respondent) right to enforce the term of the implied warranty have appeared fit. The certificates issued, however, do not
against Caltex under the marine insurance policy. However, the negate the presumption of unseaworthiness triggered by
same cannot be validly interpreted as an automatic admission of an unexplained sinking. Of certificates issued in this
the vessel’s seaworthiness by the private respondent as to regard, authorities are likewise clear as to their probative
value, (thus):
Page 107 of 154
INSURANCE G01 CASE DIGESTS

failure to rebut the presumption of fault or negligence as


Seaworthiness relates to a vessel’s actual common carrier occasioned by the unexplained sinking of
condition. Neither the granting of its vessel, MT Maysun, while in transit.
classification or the issuance of certificates
established seaworthiness. (2-A Benedict on
Admiralty, 7-3, Sec. 62).
129. ORIENTAL ASSURANCE V. COURT OF APPEALS, 200 SCRA 459
And also: (1991)

Authorities are clear that diligence in Oriental Assurance vs. Court of Appeals
securing certificates of seaworthiness does
not satisfy the vessel owner’s obligation. G.R. No. 94052 August 9, 1991
Also securing the approval of the shipper of
the cargo, or his surveyor, of the condition of FACTS:
the vessel or her stowage does not establish
due diligence if the vessel was in fact
unseaworthy, for the cargo owner has no
obligation in relation to seaworthiness. 1. Panama Sawmill shipped 1208 pieces of apitog logs to Manila and
(Ibid.) insured the logs with Oriental for the value of Php 1 million. Two
barges were loaded with 610 and 598 logs. At sea, typhoons
ravaged one of the barges, resulting in the loss of 497 of 598 of the
 Additionally, the exoneration of MT Maysun’s officers and crew logs.
by the Board of Marine Inquiry merely concerns their respective
administrative liabilities. It does not in any way operate to 2. The Insurance contract provided for indemnity under the
absolve the petitioner common carrier from its civil liabilities. It
following conditions:
does not in any way operate to absolve the petitioner common
carrier from its civil liability arising from its failure to observe Warranted that this Insurance is against TOTAL LOSS ONLY.
extraordinary diligence in the vigilance over the goods it was
Subject to the following clauses:
transporting and for the negligent acts or omissions of its
employees, the determination of which properly belongs to the — Civil Code Article 1250 Waiver clause
courts.
— Typhoon warranty clause
In the case at bar, petitioner is liable for the insured value of — Omnibus clause.
the lost cargo of industrial fuel oil belonging to Caltex for its

Page 108 of 154


INSURANCE G01 CASE DIGESTS

3. Oriental didn’t give an indemnity because there wasn’t total loss Also, the insurer's liability was for "total loss only" as stipulated. A
of the shipment. total loss may be either actual or constructive. An actual total loss
under Sec 130 of the Insurance Code is caused by:
4. The sawmill filed a civil case against Oriental and the court
ordered it to pay 410,000 as value for the missing logs. The CA (a) A total destruction of the thing insured;
affirmed the lower court judgment but reduced the legal interest.
Hence this appeal by Oriental. (b) The irretrievable loss of the thing by sinking, or by being broken
up;

(c) Any damage to the thing which renders it valueless to the owner
ISSUE: for the purpose for which he held it; or

WON Oriental Assurance can be held liable under its marine (d) Any other event which effectively deprives the owner of the
insurance policy based on the theory of a divisible contract of possession, at the port of destination, of the thing insured.
insurance and, consequently, a constructive total loss? NO.

HELD:
A constructive total loss, gives to a person insured a right to
Perla v CA- The terms of the contract constitute the measure of the abandon and it means:
insurer liability and compliance therewith is a condition precedent
SECTION 139. A person insured by a contract of marine insurance
to the insured's right to recovery from the insurer.
may abandon the thing insured, or any particular portion thereof
“Whether a contract is entire or severable is a question of intention separately valued by the policy, or otherwise separately insured,
to be determined by the language employed by the parties. The and recover for a total loss thereof, when the cause of the loss is a
policy in question shows that the subject matter insured was the peril injured against,
entire shipment of 2,000 cubic meters of apitong logs. The fact that
the logs were loaded on two different barges did not make the (a) If more than three-fourths thereof in value is actually lost, or
contract several and divisible as to the items insured. The logs on would have to be expended to recover it from the peril;
the two barges were not separately valued or separately insured. (b) If it is injured to such an extent as to reduce its value more than
Only one premium was paid for the entire shipment, making for three-fourths
only one cause or consideration. The insurance contract must,
therefore, be considered indivisible.”

Page 109 of 154


INSURANCE G01 CASE DIGESTS

The appellate court considered the cargo in one barge as separate 130. PHILIPPINE HOME ASSURANCE V. COURT OF APPEALS, 257
from the other and ruled that 497 of 598 was more than ¾ of the SCRA 468 (1996)
amount lost, showing a constructive total loss.
G.R. No. 106999, June 26, 1996

FACTS:
The SC, however, said that although the logs were placed in two
barges, they were not separately valued by the policy, nor  Eastern Shipping Lines, Inc. (ESLI) loaded on board SS
Eastern Explorer in Japan shipment of carriage to Mania and
separately insured. Of the entirety of 1,208, pieces of logs, only 497
Cebu consigned to the ff: (1) William Lines, Inc. - engine
pieces thereof were lost or 41.45% of the entire shipment. Since the parts; (2) Orca's Company - ammonium chloride; (3) Pan
cost of those 497 pieces does not exceed 75% of the value of all Oriental Match Company - bags of Glue; (4) Ding Velayo -
1,208 pieces of logs, the shipment can not be said to have sustained garments.
a constructive total loss under Section 139(a) of the Insurance Code.  While the vessel was off Okinawa, a small flame was
detected on the acetylene cylinder located in the
accommodation area near the engine room on the main
deck level. The acetylene cylinder suddenly exploded
causing death and severe injuries to the crew and instantly
setting fire to the whole vessel, which forced the crew to
abandon the ship.
 The cargoes which were saved were loaded to another
vessel for delivery to their original ports of destination. ESLI
charged the consignees several amounts corresponding to
additional freight and salvage charges.
 The charges were all paid by Philippine Home Assurance
Corporation (PHAC) under protest for and in behalf of the
consignees. Thereafter, as subrogee of the consignees, filed
a complaint against ELSI to recover the sum paid under
protest on the ground that the same were actually damage
directly brought by the fault and negligence of ESLI.
 RTC & CA: Dismissed the complaint and ruled in favor of
ESLI.

ISSUE:

Page 110 of 154


INSURANCE G01 CASE DIGESTS

(1) Who among the carrier, consignee or insurer of the goods is  Consequently, respondent ESLI's claim for the contribution
liable for the additional charges or expenses incurred? Carrier. from the consignees at the time of the occurrence of the
average turns to naught.
(2)Whether or not the respondent court committed an error in The cargo consignees cannot be made liable to respondent carrier
concluding that the expenses incurred in saving the cargo are thus respondent carrier must refund petitioner the amount it paid
considered general average? Yes. under protest for additional freight and salvage charges in behalf of
the consignees.

RATIO:

(1)

 Fire may not be considered a natural disaster or calamity


since it almost always arise from some act of men. In this
case, it is not disputed that a small flame was detected on
the acetylene cylinder and that by reason thereof the same
exploded despite efforts to extinguish the fire. Moreover,
the acetylene cylinder, obviously full loaded, was stored in
the accommodation area near the engine room and not in a
storage area considerably far and safe distance from the
engine room. Therefore, there is a strong evidence that the
acetylene cylinder caught fire because of the fault and
negligence of respondent ESLI, its captain and its crew.

(2)

 As a rule, general or gross averages include all damages and


expenses which are deliberately caused in order to save the
vessel, its cargo, or both at the same time, from a real and
known risk. While this case may technically fall within the
purview of the said provision, the formalities prescribed
under Art. 813 and 814 of the Code of Commerce in order
to incur the expenses and cause the damage corresponding
to gross average were not complied with.
Page 111 of 154
INSURANCE G01 CASE DIGESTS

FIRE INSURANCE Phil Home paid under protest, and as subrogee of the
consignee, filed a complaint to recover the sum paid under protest
131. PHILIPPINE HOME ASSURANCE V. COURT OF APPEALS, 257 SCRA
on the ground that the damages were the fault, negligence of ESLI
468 (1996)
FIRE INSURANCE
ISSUE
Phil Home Assurance v. CA
WON the burning of the ship is a natural disaster or
FACTS calamity which would absolve ESLI from liability? NO

ESLI (respondent) loaded to a ship certain goods like engine


parts, mamonium chloride, and glue 300. . HELD

While the vessel was off Okinawa, Japan, a small flame was In our jurisprudence, fire may not be considered a natural
detected on the acetylene cylinder located in the accommodation disaster or calamity since it almost always arises from some act of
area near the engine room on the main deck level. As the crew was man or by human means.
trying to extinguish the fire, the acetylene cylinder suddenly
exploded sending a flash of flame throughout the accommodation It cannot be an act of God unless caused by lightning or a
area, thus causing death and severe injuries to the crew and natural disaster or casualty not attributable to human agency.
instantly setting fire to the whole superstructure of the vessel. The
incident forced the master and the crew to abandon the ship. In the case at bar, it is not disputed that a small flame was
detected on the acetylene cylinder and that by reason thereof, the
After the fire was extinguished, the cargoes which were same exploded despite efforts to extinguish the fire. Neither is
saved were loaded to another vessel for delivery to their original there any doubt that the acetylene cylinder, obviously fully loaded,
ports of destination. was stored in the accommodation area near the engine room and
not in a storage area considerably far, and in a safe distance, from
ESLI charged the consignees additional charges or expenses the engine room. Moreover, there was no showing, and none was
incurred by the owner of the ship in the salvage operations and in alleged by the parties, that the fire was caused by a natural disaster
the transshipment of the goods via a different carrier. or calamity not attributable to human agency. On the contrary,
there is strong evidence indicating that the acetylene cylinder
caught fire because of the fault and negligence of respondent ESLI,
its captain and its crew.
Page 112 of 154
INSURANCE G01 CASE DIGESTS

132. BACHRACH V. BRITISH AMERICAN ASSURANCE , 17 PHIL. 35


the fact that the acetylene cylinder was checked, tested and (1910)
examined and subsequently certified as having complied with the
E. M. BACHRACH vs. BRITISH AMERICAN ASSURANCE
safety measures and standards by qualified experts before it was
COMPANY
loaded in the vessel only shows to a great extent that negligence G.R. No. L-5715 December 20, 1910
was present in the handling of the acetylene cylinder after it was
loaded and while it was on board the ship. Indeed, had the JOHNSON, J.:
respondent and its agents not been negligent in storing the
acetylene cylinder near the engine room, then the same would not FACTS:
have leaked and exploded during the voyage 1. E. M. Bachrach insured against loss or damage by fire,
goods, belonging to its general furniture store, such as iron
and brass bedsteads, toilet tables, chairs, ice boxes,
There is no merit in the finding of the trial court to which bureaus, washstands, mirrors, and sea-grass furniture from
respondent court erroneously agreed that the fire was not the British American Assurance Company through it's agent.
fault or negligence of respondent but a natural disaster or
calamity. The records are simply wanting in this regard. 2. In the policy, Co insurance was allowed which should be
declared in the event of loss or claim. Also, such policy
provided Permission granted for the use of gasoline not to
exceed 10 gallons for the "Calalac" automobile, but only
WHEREFORE, the judgment appealed from is hereby REVERSED and
whilst contained in the reservoir of the car.
SET ASIDE. Respondent Eastern Shipping Lines, Inc. is ORDERED to
return to petitioner Philippine Home Assurance Corporation the 3. Fire broke out. British American Assurance Company
amount it paid under protest in behalf of the consignees herein. denied the claim of Bachrach alleging the ff:

a. That the plaintiff maintained a paint and varnish shop in


the said building where the goods which were insured were
stored.

b. That the plaintiff transferred his interest in and to the


property covered by the policy to H. W. Peabody & Co. to
secure certain indebtedness due and owing to said
company, and also that the plaintiff had transferred his
interest in certain of the goods covered by the said policy to
one Macke, to secure certain obligations assumed by the

Page 113 of 154


INSURANCE G01 CASE DIGESTS

said Macke for and on behalf of the insured. That the by the defendant to the plaintiff, it will be noted that there
sanction of the said defendant had not been obtained by is no provision in said policy prohibiting the plaintiff from
the plaintiff, as required by the said policy. placing a mortgage upon the property insured. It is claimed
that the execution of a chattel mortgage on the insured
c. That the plaintiff, immediately preceding the outbreak of property violated what is known as the "alienation clause,"
the alleged fire, willfully placed a gasoline can containing 10 which is now found in most policies, and which is expressed
gallons of gasoline in the upper story of said building in in the policies involved in cases 6496 and 6497 by a
close proximity to a portion of said goods, wares, and purchase imposing forfeiture if the interest in the property
merchandise, which can was so placed by the plaintiff as to pass from the insured. (Cases 6496 and 6497, in which are
permit the gasoline to run on the floor of said second story, involved other action against other insurance companies for
and after so placing said gasoline, he, the plaintiff, placed in the same loss as in the present action.)
close proximity to said escaping gasoline a lighted lamp
containing alcohol, thereby greatly increasing the risk of *alienation clause - forfeiture if the interest in the property
fire. pass from the insured

4. The lower court ruled in favor of Bachrach making British - there is no alienation within the meaning of the insurance
American Assurance liable. law until the mortgage acquires a right to take possession
by default under the terms of the mortgage. No such right is
ISSUE: claimed to have accrued in the case at bar, and the
WoN British American Assurance liable of such claim. alienation clause is therefore inapplicable.

HELD:YES. - This clause has been the subject of a vast number of


judicial decisions (13 Am. & Eng. Encyc. of Law, 2d ed., pp.
RATIO DECIDENDI: 239 et seq.), and it is held by the great weight of authority
that the interest in property insured does not pass by the
- Keeping of inflammable oils on the premises, though mere execution of a chattel mortgage and that while a
prohibited by the policy, does not void it if such keeping is chattel mortgage is a conditional sale, there is no alienation
incidental to the business. It may be added also that there within the meaning of the insurance law until the mortgage
was no provision in the policy prohibiting the keeping of acquires a right to take possession by default under the
paints and varnishes upon the premises where the insured terms of the mortgage. No such right is claimed to have
property was stored. If the company intended to rely upon accrued in the case at bar, and the alienation clause is
a condition of that character, it ought to have been plainly therefore inapplicable.
expressed in the policy.

- Furthermore, upon reading the policy of insurance issued

Page 114 of 154


INSURANCE G01 CASE DIGESTS

o No explanation was offered which would account


for the remarkable conduct of Tan Chuco’s manager
133. TAN CHUCO V. YORKSHIRE F IRE AND LIFE INSURANCE , 14 PHIL. in preparing an inventory two months after his
346 (1909) employer had left for China and then instead of
forwarding such inventory to his principal by mail,
Tan Chuco vs Yorkshire Fire and Life Insurance Company entrusted it for transmission to a friend who had
not even left for China when the fire took place.
Facts: o Indication that Tan Chuco had been experiencing
adverse business conditions before the fire
 Tan Chuco files a claim under an open fire insurance policy
for the alleged loss by fire of certain stock of goods insured
by Yorkshire. Issue: Whether or not Tan Chuco may claim under the fire
 CFI: Evidence did not sustain Yorkshire’s allegation that Tan insurance policy?
Chuco or his agents had intentionally and fraudulently set
the building on fire
o But was of the opinion that the Tan Chuco failed to Held: NO.
establish the value of the goods he alleges were
destroyed by the fire. We think that the action of the trial court in rejecting the proof
o He submitted fabricated written evidence and false offered by Tan Chuco as to the amount of the loss must be
testimony in support of his claim that the insured
sustained.
goods actually destroyed were worth more than the
total amount of the insurance thereon.
o CFI was of the opinion that the submitted inventory
The contract of fire insurance being a contract of indemnity, Tan
was not genuine and was fraudulently prepared.
o Tan Chuco’s representatives and employees who Chuco is only entitled to recover the amount of actual loss sustained
were in the building when the fire took place, not by him. There being no express valuation in the policy, the
only made no effort to extinguish the fire, or to save judgment was properly entered against him for lack of satisfactory
the goods from destruction, but also failed to save proof of the amount of loss.
any of the books or papers connected with the
business of which he was in charge of—those could Rule: In the absence of express valuation in a fire insurance policy,
have corroborated with the data in the alleged the insured is only entitled to recover the amount of actual loss
inventory sustained and the burden is upon him to establish such amount.
o The inventory submitted was dated January 1, not
of custom to Tan Chuco who were of Chinese
decent.

Page 115 of 154


INSURANCE G01 CASE DIGESTS

134. MALAYAN INSURANCE V. CRUZ-ARNALDO, 154 SCRA 672  Under Section 416 of the Insurance Code, the period for appeal
(1987)
is thirty days from notice of the decision of the Insurance
Commission. The petitioner filed its motion for reconsideration
G.R. No. L-67835 October 12, 1987
on April 25, 1981, or fifteen days such notice, and the
Lessons Applicable: Authority to Receive Payment/Effect of
reglementary period began to run again after June 13, 1981,
Payment (Insurance)
date of its receipt of notice of the denial of the said motion for
Laws Applicable: Article 64, Article 65, Section 77, Section 306 of the reconsideration. As the herein petition was filed on July 2, 1981,
Insurance Code or nineteen days later, there is no question that it is tardy by
four days.
 Insurance Commission: favored Pinca
FACTS:
 MICO appealed
ISSUE: W/N MICO should be liable because its agent Adora was
 June 7, 1981: Malayan insurance co., inc. (MICO) issued
authorized to receive it
to Coronacion Pinca, Fire Insurance Policy forher
property effective July 22, 1981, until July 22, 1982
HELD: YES. petition is DENIED
 October 15,1981: MICO allegedly cancelled the policy for non-
payment, of the premium and sent the corresponding notice to  SEC. 77. An insurer is entitled to payment of the premium as
Pinca soon as the thing is exposed to the peril insured against.
 December 24, 1981: payment of the premium for Pinca was Notwithstanding any agreement to the contrary, no policy or
received by Domingo Adora, agent of MICO contract of insurance issued by an insurance company is valid
 January 15, 1982: Adora remitted this payment to and binding unless and until the premium thereof has been
MICO,together with other payments paid, except in the case of a life or an industrial life
 January 18, 1982: Pinca's property was completely burned policy whenever the grace period provision applies.
 February 5, 1982: Pinca's payment was returned by MICO to  SEC. 306. xxx xxx xxx
Adora on the ground that her policy had been cancelled earlier
but Adora refused to accept it and instead demanded for
Any insurance company which delivers to an insurance agant
payment
or insurance broker a policy or contract of insurance shall be
demmed to have authorized such agent or broker to receive on its
Page 116 of 154
INSURANCE G01 CASE DIGESTS

behalf payment of any premium which is due on such policy or As for the method of cancellation, Section 65 provides as follows:
contract of insurance at the time of its issuance or delivery or which
 SEC. 65. All notices of cancellation mentioned in the preceding
becomes due thereon.
section shall be in writing, mailed or delivered to the named
 Payment to an agent having authority to receive or collect insured at the address shown in the policy, and shall state (a)
payment is equivalent to payment to the principal himself; such which of the grounds set forth in section sixty-four is relied
payment is complete when the money delivered is into the upon and (b) that, upon written request of the named insured,
agent's hands and is a discharge of the indebtedness owing to the insurer will furnish the facts on which the cancellation is
the principal. based.
 SEC. 64. No policy of insurance other than life shall be  A valid cancellation must, therefore, require concurrence of the
cancelled by the insurer except upon prior notice thereof to the following conditions:
insured, and no notice of cancellation shall be effective unless it (1) There must be prior notice of cancellation to the insured;
is based on the occurrence, after the effective date of the
(2) The notice must be based on the occurrence, after the
policy, of one or more of the following:
effective date of the policy, of one or more of the grounds
mentioned;
(a) non-payment of premium;
(3) The notice must be (a) in writing, (b) mailed, or delivered to
(b) conviction of a crime arising out of acts increasing the hazard the named insured, (c) at the address shown in the policy;
insured against;
(4) It must state (a) which of the grounds mentioned in Section 64
(c) discovery of fraud or material misrepresentation; is relied upon and (b) that upon written request of the insured, the
insurer will furnish the facts on which the cancellation is based.
(d) discovery of willful, or reckless acts or commissions increasing
the hazard insured against;  All MICO's offers to show that the cancellation was
communicated to the insured is its employee's testimony that
(e) physical changes in the property insured which result in the
the said cancellation was sent "by mail through our mailing
property becoming uninsurable;or
section." without more
(f) a determination by the Commissioner that the continuation of  It stands to reason that if Pinca had really received the said
the policy would violate or would place the insurer in violation of notice, she would not have made payment on the original policy
this Code. on December 24, 1981. Instead, she would have asked for a
Page 117 of 154
INSURANCE G01 CASE DIGESTS

new insurance, effective on that date and until one year later,
and so taken advantage of the extended period.
 Incidentally, Adora had not been informed of the cancellation
either and saw no reason not to accept the said payment
 Although Pinca's payment was remitted to MICO's by its agent
on January 15, 1982, MICO sought to return it to Adora only on
February 5, 1982, after it presumably had learned of the
occurrence of the loss insured against on January 18, 1982
make the motives of MICO highly suspicious

Page 118 of 154


INSURANCE G01 CASE DIGESTS

CASUALTY INSURANCE
2. W/N the heirs can sue the insurer and insured jointly? - YES

135. GUINGON V. DEL MONTE, 20 SCRA 1043 (1967)


Guingon v. Del Monte RATIO: Affirmed in toto.
20 SCRA 1043 (1967)
Casualty Insurance 1. YES

FACTS:  policy: the insurer agreed to indemnify the insured "against all
sums . . . which the Insured shall become legally liable to pay in
1. Julio Aguilar owner and operator of several jeepneys respect of: a. death of or bodily injury to any person . . . ." -
insured them with Capital Insurance & Surety Co., Inc. indemnity against liability
2. February 20, 1961: Along the intersection of Juan Luna and  TEST: Where the contract provides for indemnity
Moro streets, City of Manila, the jeepneys operated by against liability to third persons, then third persons to whom
Aguilar driven by Iluminado del Monte and Gervacio the insured is liable, CAN sue the insurer. Where the contract is
Guingon bumped and Guingon died some days after for indemnity against actual loss or payment, then third persons
3. Iluminado del Monte was charged with homicide thru CANNOT proceed against the insurer, the contract being solely
reckless imprudence and was penalized 4 months to reimburse the insured for liability actually discharged by him
imprisonment thru payment to third persons, said third persons' recourse
4. The heirs of Gervacio Guingon filed an action for damages being thus limited to the insured alone.
praying that P82,771.80 be paid to them jointly and
severally by the driver del Monte, owner and operator 2. YES
Aguilar, and the Capital Insurance & Surety Co., Inc.
5. CFI: Iluminado del Monte and Julio Aguilar jointly and  policy: expressly disallows suing the insurer as a co-defendant
severally to pay plaintiffs the sum of P8,572.95 as damages of the insured in a suit to determine the latter's liability
for the death of their father, plus P1,000.00 for attorney's  no action close: suit and final judgment be first obtained against
fees plus costs the insured; that only "thereafter" can the person injured
6. Capital Insurance and Surety Co., Inc. is hereby sentenced recover on the policy
to pay P5,000 plus P500 as attorney's fees and costs to be  Sec. 5 of Rule 2 on "Joinder of causes of action" and Sec. 6 of
applied in partial satisfaction of the judgment rendered Rule 3 on "Permissive joinder of parties" cannot be
against Iluminado del Monte and Julio Aguilar in this case superseded, at least with respect to third persons not a party to
the contract, as herein, by a "no action" clause in the contract
ISSUE: of insurance.

1. W/N there a stipulation pour autriu to enable that will enable


the heirs to sue against Capital Insurance and Surety Co., Inc.? - YES
Page 119 of 154
INSURANCE G01 CASE DIGESTS

136. DE LA CRUZ V. CAPITAL INSURANCE , 17 SCRA 559 (1966) HELD: YES

INSURANCE: CASUALTY INSURANCE  The terms "accident" and "accidental", as used in insurance
contracts, have not acquired any technical meaning, and are
SIMON DE LA CRUZ vs CAPITAL INSURANCE and SURETY Co., Inc. construed by the courts in their ordinary and common
acceptation. Thus, the terms have been taken to mean that
FACTS: which happen by chance or fortuitously, without intention
and design, and which is unexpected, unusual, and
 Eduardo de la Cruz, a mucker in Itogon-Suyoc Mines, Inc. unforeseen. An accident is an event that takes place
was the holder of an accident insurance policy underwritten without one's foresight or expectation — an event that
by Capital Insurance for the period of November 13, 1956 - proceeds from an unknown cause, or is an unusual effect of
November 12, 1957. a known cause and, therefore, not expected.
 Itugon sponsored a boxing contest as part of their  Even if we take appellant's theory, the death of the insured
celebration of New Year. Eduardo participated as a non- in the case at bar would still be entitled to indemnification
professional boxer. In the course of his bout with another under the policy. The generally accepted rule is that, death
person, another non-professional, of the same height, or injury does not result from accident or accidental means
weight, and size, Eduardo slipped and was hit by his within the terms of an
opponent on the left part of the back of his head. This accident-policy if it is the natural result of the insured's
caused Eduardo to fall with his head hitting the rope of the voluntary act, unaccompanied by anything unforeseen
ring. He died the following day. The cause of his death was except the death or injury. There is no accident when a
hemmorhage, intracranial, left. deliberate act is performed unless some additional,
 The father, Simon, was named beneficiary and filed a claim unexpected, independent, and unforeseen happening
with the insurance company. Insurer denied. CFI (Specific occurs which produces or brings about the result of injury
Performance) ruled in favour of Simon. or death. In other words, where the death or injury is not
 Insurer contends that while the death of the insured was the natural or probable result of the insured's voluntary
due to head injury, said injury was sustained by his act, or if something unforeseen occurs in the doing of the
voluntary participation in the contest. He cannot be act which produces the injury, the resulting death is within
considered to have met his death by “accidental means.” In the protection of policies insuring against death or injury
other words what is required to be accidental is the means from accident.
that caused the death and not the death itself.  In the present case, while the participation of the insured in
the boxing contest is voluntary, the injury was sustained
when he slid, giving occasion to the infliction by his
ISSUE: WON Eduardo’s death is an accident within the meaning of opponent of the blow that threw him to the ropes of the
the policy despite the fact that he voluntarily participated in the ring. Without this unfortunate incident, that is, the
boxing contest. unintentional slipping of the deceased, perhaps he could

Page 120 of 154


INSURANCE G01 CASE DIGESTS

not have received that blow in the head and would not have
died. The fact that boxing is attended with some risks of 137. PAN MALAYAN V. COURT OF APPEALS, 184 SCRA 54 (1990)
external injuries does not make any injuries received in the PAN MALAYAN INSURANCE vs CA
course of the game not accidental. In boxing as in other GR 81026, 184 SCRA 54; April 3, 1990
equally physically rigorous sports, such as basketball or
baseball, death is not ordinarily anticipated to result. If, Topic: Casualty Insurance: Accident and health insurance
therefore, it ever does, the injury or death can only be
accidental or produced by some unforeseen happening or Facts:
event as what occurred in this case. Pan Malayan insured a Lancer car registered in the name of
 As to the policy provision enumerating sports events which Canlubang Automotive Resources. On May 26, 1985 the car was hit
are excluded from coverage, death or disablement resulting by a pick-up truck due to the “carelessness, recklessness and
from engagement in boxing contests was not declared imprudence” of its driver. The insured car suffered damages, whose
outside of the protection of the insurance contract. Failure repairs Pan Malayan paid for. Thus it was subrogated in the rights of
of the defendant insurance company to include death the owner against the pick-up driver’s employer, the private
resulting from a boxing match or other sports among the respondent Erlinda Fabie. Despite demands, Fabie refused and
prohibitive risks leads inevitably to the conclusion that it did failed to pay.
not intend to limit or exempt itself from liability for such Pan Malayan alleged that the damage to the insured car
death. was covered under the “own damage” coverage of the policy, and
that its driver was authorized at the time of the accident. The
respondents argued that payment under the "own damage" clause
of the insurance policy precluded subrogation under Article 2207 of
the Civil Code, since indemnification there was made on the
assumption that there was no wrongdoer or no third party at fault.
Pan Malayan alleged in its complaint that, pursuant to a
motor vehicle insurance policy, it had indemnified Canlubang for
the damage to the insured car resulting from a traffic accident
allegedly caused by the negligence of the driver of private
respondent, Erlinda Fabie. It thus claims that its cause of action
against private respondents was anchored upon Article 2207 of the
Civil Code.
The RTC dismissed Pan Malayan’s complaint. The CA
affirmed.

Issue:

Page 121 of 154


INSURANCE G01 CASE DIGESTS

1. Was the insurer’s indemnity under the “own damage” 1. It is a basic rule in the interpretation of contracts that the
clause of its policy an admission that its insured/ client terms of a contract are to be construed according to the
caused the damage? sense and meaning of the terms which the parties
2. Given this admission, was the insurer subrogated? thereto have used. In the case of property insurance
policies, the evident intention of the contracting
Held: parties, i.e., the insurer and the assured, determine the
1. NO import of the various terms and provisions embodied in the
2. YES policy. It is only when the terms of the policy are
ambiguous, equivocal or uncertain, such that the parties
Rationale: (Only #1) themselves disagree about the meaning of particular
It must be emphasized that the lower court's ruling that the "own provisions, that the courts will intervene. In such an event,
damage" coverage under the policy impliesdamage to the insured the policy will be construed by the courts liberally in favor
car caused by the assured itself, instead of third parties, proceeds of the assured and strictly against the insurer.
from an incorrect comprehension of the phrase "own damage" as
used by the insurer. When PANMALAY utilized the phrase "own Section III-1 of the insurance policy which refers to the conditions
damage" — a phrase which, incidentally, is not found in the under which the insurer PANMALAY is liable to indemnify the
insurance policy — to define the basis for its settlement of assured CANLUBANG against damage to or loss of the insured
CANLUBANG's claim under the policy, it simply meant that it had vehicle, reads as follows:
assumed to reimburse the costs for repairing the damage to the
insured vehicle. It is in this sense that the so-called "own damage" SECTION III — LOSS OR DAMAGE
coverage under Section III of the insurance policy is differentiated
from Sections I and IV-1 which refer to "Third Party Liability" 1. The Company will, subject to the Limits of
coverage (liabilities arising from the death of, or bodily injuries Liability, indemnify the Insured against loss of or
suffered by, third parties) and from Section IV-2 which refer to damage to the Scheduled Vehicle and its
"Property Damage" coverage (liabilities arising from damage caused accessories and spare parts whilst thereon: —
by the insured vehicle to the properties of third parties).
(a) by accidental collision or
Neither is there merit in the Court of Appeals' ruling that overturning, or collision or
the coverage of insured risks under Section III-1 of the overturning consequent upon
policy does not include to the insured vehicle arising from mechanical breakdown or
collision or overturning due to the negligent acts of the consequent upon wear and tear;
third party. Not only does it stem from an erroneous
interpretation of the provisions of the section, but it also (b) by fire, external explosion, self
violates a fundamental rule on the interpretation of ignition or lightning or burglary,
property insurance contracts. housebreaking or theft;

Page 122 of 154


INSURANCE G01 CASE DIGESTS

(c) by malicious act; 138. SUN INSURANCE V. COURT OF APPEALS, 211 SCRA 554 (1990)

(d) whilst in transit (including the


processes of loading and unloading) SUN INSURANCE OFFICE, LTD., petitioner, vs.
incidental to such transit by road, THE HON. COURT OF APPEALS and NERISSA LIM, respondents.
rail, inland, waterway, lift or
elevator. CRUZ, J.:

It cannot be said that the meaning given by PANMALAY and July 17, 1992
CANLUBANG to the phrase "by accidental collision or overturning"
found in the first paint of sub-paragraph (a) is untenable. Although Facts
the terms "accident" or "accidental" as used in insurance contracts
have not acquired a technical meaning, the Court has on several 1. Sun life issued a personal accident policy to Felix Lim.
occasions defined these terms to mean that which takes place 2. 2 months later, he died. His wife as the beneficiary sought
"without one's foresight or expectation, an event that proceeds payment of the policy but was rejected.
from an unknown cause, or is an unusual effect of a known cause a. However, despite the rejection of the claim, Sun life
and, therefore, not expected". Certainly, it cannot be inferred from
found that there was neither suicide nor an
jurisprudence that these terms, without qualification, exclude
accident.
events resulting in damage or loss due to the fault, recklessness or
negligence of third parties. The concept "accident" is not necessarily 3. The eyewitness to his death, his secretary, testified that he
synonymous with the concept of "no fault". It may be utilized simply was playing with his handgun from which he had previously
to distinguish intentional or malicious acts from negligent or removed his magazine. He pointed it to his temple and
careless acts of man. then, an explosion occurred. Lim fell on the floor dead.
4. RTC ruled in favor of the wife. Sun life should pay the policy
Moreover, a perusal of the provisions of the insurance policy reveals
plus damages.
that damage to, or loss of, the insured vehicle due to negligent or
careless acts of third parties is not listed under the general and 5. CA affirmed.
specific exceptions to the coverage of insured risks which are
enumerated in detail in the insurance policy itself. Issue

Is the death of Lim an accident in which the beneficiary may claim


payment of the policy? YES

(To restate: Does Lim’s death fall under the exceptions in which the
insurer would not be liable? NO)

Page 123 of 154


INSURANCE G01 CASE DIGESTS

Ruling: agreed upon if the insured is shown to have contributed to


his own accident. Indeed, most accidents are caused by
 The Court believes that Lim’s death was an ACCIDENT. negligence. There are only four exceptions expressly made
o The term, when used in an insurance contract, is to in the contract to relieve the insurer from liability, and none
be construed and considered according to the of these exceptions is applicable in the case at bar. **
ordinary understanding and common usage and
speech of people generally. ** Exceptions
o An accident is an event which happens without any The Company shall not be liable in respect of
1. bodily injury
human agency or, if happening through human a) sustained
agency, an event which, under the circumstances, is i) while the Insured Person is engaging in (or practicing
unusual to and not expected by the person to for or taking part in training peculiar to) any of the
Excluded Activities.
whom it happens. It has also been defined as an
ii) by any person before such person attains the Lower
injury which happens by reason of some violence Age Limit or after the expiry of the Period of Insurance
or CASUALTY to the injured without his design, during which such person attains the Upper Age Limit.
consent, or voluntary co-operation. b) consequent upon
i) the Insured Person committing or attempting to
o As the secretary testified, Lim had removed the
commit suicide or wilfully exposing himself to needless
magazine from the gun and believed it was no peril except in an attempt to save human life.
longer dangerous. He expressly assured her that the ii) war, invasion, act of foreign enemy, hostilities
gun was not loaded. It is submitted that Lim did not (whether war be declared or not) civil war, rebellion,
revolution, insurrection, or military or usurped power.
willfully expose himself to needless peril when he 2. bodily injury or Death Disablement or Medical
pointed the gun to his temple because the fact is Expenses consequent upon or contributed to by the
that he thought it was not unsafe to do so. The act Insured Person
was precisely intended to assure Nalagon that the a) having taken a drug unless the Insured proves that the
drug was taken in accordance with proper medical
gun was indeed harmless. prescription and directions and not for treatment of drug
o Lim did not know that the gun he placed to his head addiction.
was loaded. b) suffering from pre-existing physical or mental defect
or infirmity which had not been declared to and accepted
 Lim was unquestionably negligent and that negligence cost
in writing by the Company.
him his own life. But it should not prevent his widow from 3. Death Disablement or Medical Expenses consequent
recovering from the insurance policy he obtained precisely upon or contributed to by the Insured Person being
against accident. There is nothing in the policy that relieves pregnant or suffering from sickness or disease not
resulting from bodily injury or suffering from bodily
the insurer of the responsibility to pay the indemnity
injury due to a gradually operating cause.
Page 124 of 154
INSURANCE G01 CASE DIGESTS

4. Risks of Murder and Assault. Were the injuries sustained by the deceased “intentionally
afflicted”, thus not covered by the accidental death benefit
 The award of damages is deleted since there is no evidence
clause?
to show that there was any wrongful act done by the
insurer, Sun Life. HELD

SC affirms CA. Yes. Whatever the true intent of the robbers may be, the act itself
of inflicting the injuries was intentional. The exception clause does
139. BIAGTAN V. INSULAR LIFE, 144 SCRA 58 (1972) not speak of the purpose of the one causing the injury, only of the
fact that such injuries have been "intentionally" inflicted, to
Biagtan vs Insular Life (1972) distinguish them from injuries which, although received at the
hands of a third party, are purely accidental. Examples would be
FACTS
when a gun which discharges while being cleaned and kills a
Juan Biagtan was insured with Insular for P5k with a supplementary bystander; or a hunter who shoots at his prey and hits a person
contract “Accidental Death Benefit” clause for another P5k if "the instead; or an athlete in a competitive game involving physical
death of the Insured resulted directly from bodily injury effected effort who collides with an opponent and fatally injures him as a
solely through external and violent means sustained in an accident . result.
. . and independently of all other causes." The clause, however,
In Calanoc vs. CA, where a shot was fired and it turned out
expressly provided that it would not apply where death resulted
afterwards that the watchman was hit in the abdomen, the wound
from an injury "intentionally inflicted by a third party."
causing his death, the Court held that it could not be said that the
One night, robbers entered their house. Juan was stabbed 9 times killing was intentional for there was the possibility that the
with a knife and then died. The robbers were convicted of robbery malefactor had fired the shot to scare the people around for his
with homicide. own protection and not necessarily to kill or hit the victim.

The family now claims for the additional P5k from Insular, under the It is not the same with the case at bar, for while a single shot fired
Accidental Death Benefit clause. Insular refused saying that the from a distance, and by a person who was not even seen aiming at
death resulted from injuries intentionally inflicted by 3rd parties the victim, could indeed have been fired without the intent to kill or
and was therefore not covered. The family sued. injure, nine wounds inflicted with bladed weapons at close range
cannot conceivably be considered as innocent insofar as such intent
CFI ruled in favor of the family. is concerned.

ISSUE
Page 125 of 154
INSURANCE G01 CASE DIGESTS

Insular won and the family cannot claim the additional 5k under the associated words and concepts in the policy plainly exclude the
ADB clause because the injury was intentionally caused by a person, accidental death from the coverage of the policy only where the
i.e. not accidental. injuries are self-inflicted or attended by some proscribed act of the
insured or are incurred in some expressly excluded calamity such as
riot, war or atomic explosion.- The untenability of insurer's claim
Justice Teehankee dissents, saying that Calanoc v. CA is controlling that the insured's death fell within the exception is further
because the insurance company was unable to prove that the killing heightened by the stipulated fact that two other insurance
was intentional. Burden of proof is with the insurance company. companies which likewise covered the insured for much larger sums
Insurance, being contracts of adhesion, must be construed strictly under similar accidental death benefit clauses promptly paid the
against insurance company in cases of ambiguity. The benefits thereof to plaintiffs beneficiaries.
supplementary contract enumerated exceptions. The only exception
which is not susceptible of classification is that provided in 140. CALANOC V. COURT OF APPEALS, 98 PHIL. 79 (1955)
paragraph 5(e), the very exception which is currently the subject of
contention, which would also except injuries "inflicted intentionally Calanoc v CA
by a third party, either with or without provocation on the part of
Facts:
the insured, and whether or not the attack or the defense by the
third party was caused by a violation of the law by the insured." The The circumstances surrounding the death of Melencio Basilio show
ambiguous clause conflicts with all the other four exceptions in the that when he was killed at about seven o’clock in the night of
same paragraph 5 particularly that immediately preceding it in item January 25, 1951, he was on duty as watchman of the Manila Auto
(d) which excepts injuries received where the insured has violated Supply at the corner of Avenida Rizal and Zurbaran; that it turned
the law or provoked the injury, while this clause, construed as the out that Atty. Antonio Ojeda who had his residence at the corner of
insurance company now claims, would seemingly except also all Zurbaran and Oroquieta, a block away from Basilio’s station, had
other injuries, intentionally inflicted by a third party, regardless of come home that night and found that his house was well-lighted,
any violation of law or provocation by the insured, and defeat the but with the windows closed; that getting suspicious that there
very purpose of the policy of giving the insured double indemnity in were culprits in his house, Atty. Ojeda retreated to look for a
case of accidental death by "external and violent means"— in the policeman and finding Basilio in khaki uniform, asked him to
very language of the policy.' It is obvious from the very classification accompany him to the house with the latter refusing on the ground
of the exceptions and applying the rule of noscitus a sociis, that the that he was not a policeman, but suggesting that Atty. Ojeda should
double-indemnity policy covers the insured against accidental ask the traffic policeman on duty at the corner of Rizal Avenue and
death, whether caused by fault, negligence or intent of a third party Zurbaran; that Atty. Ojeda went to the traffic policeman at said
which is unforeseen and unexpected by the insured. All the
Page 126 of 154
INSURANCE G01 CASE DIGESTS

corner and reported the matter, asking the policeman to come


along with him, to which the policeman agreed; that on the way to
the Ojeda residence, the policeman and Atty. Ojeda passed by Ratio:
Basilio and somehow or other invited the latter to come along; that The circumstances of Basilio’s death cannot be taken as purely
as the tree approached the Ojeda residence and stood in front of intentional on the part of Basilio to expose himself to the danger.
the main gate which was covered with galvanized iron, the fence There is no proof that his death was the result of intentional killing
itself being partly concrete and partly adobe stone, a shot was fired; because there is the possibility that the malefactor had fired the
that immediately after the shot, Atty. Ojeda and the policeman shot merely to scare away the people around.
sought cover; that the policeman, at the request of Atty. Ojeda, left
the premises to look for reinforcement; that it turned out While as a general rule "the parties may limit the coverage of the
afterwards that the special watchman Melencio Basilio was hit in policy to certain particular accidents and risks or causes of loss, and
the abdomen, the wound causing his instantaneous death; that the may expressly except other risks or causes of loss therefrom",
shot must have come from inside the yard of Atty. Ojeda, the bullet however, it is to be desired that the terms and phraseology of the
passing through a hole waist-high in the galvanized iron gate; that exception clause be clearly expressed so as to be within the easy
upon inquiry Atty. Ojeda found out that the savings of his children in grasp and understanding of the insured, for if the terms are
the amount of P30 in coins kept in his aparador contained in doubtful or obscure the same must of necessity be interpreted or
stockings were taken away, the aparador having been ransacked; resolved against the one who has caused the obscurity. (Article
that a month thereafter the corresponding investigation conducted 1377, new Civil Code) And so it has been generally held that the
by the police authorities led to the arrest and prosecution of four "terms in an insurance policy, which are ambiguous, equivocal, or
persons in Criminal Case No. 15104 of the Court of First Instance of uncertain . . . are to be construed strictly and most strongly against
Manila for ‘Robbery in an Inhabited House and in Band with the insurer, and liberally in favor of the insured so as to effect the
Murder’. dominant purpose of indemnity or payment to the insured,
especially where a forfeiture is involved", and the reason for this
rule is that he "insured usually has no voice in the selection or
Issue: Whether or not the Philippine American Life Insurance Co. arrangement of the words employed and that the language of the
liable to the petitioner for the amount covered by the supplemental contract is selected with great care and deliberation by experts and
contract? legal advisers employed by, and acting exclusively in the interest of,
the insurance company."

Insurance is, in its nature, complex and difficult for the layman to
Held: Yes. understand. Policies are prepared by experts who know and can

Page 127 of 154


INSURANCE G01 CASE DIGESTS

anticipate the bearings and possible complications of every


contingency. So long as insurance companies insist upon the use of 141. PHIL AM CARE HEALTH SYSTEMS V. COURT OF APPEALS, 379
ambiguous, intricate and technical provisions, which conceal rather SCRA 356 (2002)
than frankly disclose, their own intentions, the courts must, in
PHILAMCARE HEALTH SYSTEMS, INC. V CA (TRINOS)
fairness to those who purchase insurance, construe every ambiguity
in favor of the insured. 379 SCRA 357

An insurer should not be allowed, by the use of obscure phrases and YNARES-SANTIAGO; March 18, 2002
exceptions, to defeat the very purpose for which the policy was
procured.

Applying this to the situation, the ambiguous or obscure terms in NATURE


the insurance policy are to be construed strictly against the insurer
Petition for review of CA decision
and liberally in favor of the insured party. The reason is to ensure
the protection of the insured since these insurance contracts are
usually arranged and employed by experts and legal advisers acting
exclusively in the interest of the insurance company. As long as FACTS
insurance companies insist upon the use of ambiguous, intricate
- Ernani TRINOS, deceased husband of respondent Julita, applied for
and technical provisions, which conceal their own intentions, the
a health care coverage with Philamcare Health Systems, Inc. In the
courts must, in fairness to those who purchase insurance, construe
standard application form, he answered no to the question: “Have
every ambiguity in favor of the insured. you or any of your family members ever consulted or been treated
for high blood pressure, heart trouble, diabetes, cancer, liver
disease, asthma or peptic ulcer? (If Yes, give details).”

- The application was approved for period of one year; upon


termination, it was extended for another 2 years. Amount of
coverage was increased to a maximum sum of P75T per disability.

- During this period, Ernani suffered a HEART ATTACK and was


confined at the Manila Medical Center (MMC) for one month. While
her husband was in the hospital, Julita tried to claim the
hospitalization benefits.
Page 128 of 154
INSURANCE G01 CASE DIGESTS

- Petitioner treated the Health Care Agreement (HCA) as void since (4) It is not an insurance company, governed by Insurance
there was a concealment regarding Ernani’s medical history. Commission, but a Health Maintenance Organization under the
Doctors at the MMC allegedly discovered at the time of his authority of DOH.
confinement, he was hypertensive, diabetic and asthmatic. Julita
then paid the hospitalization expenses herself, amounting to about (5) Trinos concealed a material fact in his application.
P76T. (6) Julita was not the legal wife since at the time of their marriage,
- After her husband died, Julita instituted action for damages the deceased was previously married to another woman who was
against Philamcare and its Pres. After trial, the lower court ruled in still alive.*
her favor and ordered Philamcare to reimburse medical and
hospital coverage amounting to P76T plus interest, until fully paid;
pay moral damages of P10T; pay exemplary damages of P10T; atty’s ISSUES
fees of P20T.
1. WON a health care agreement is an insurance contract (If so,
- CA affirmed the decision of the trial court but deleted all awards “incontestability clause” under the Insurance Code is applicable)
for damages and absolved petitioner Reverente.
2. WON the HCA can be invalidated on the basis of alleged
concealment

Petitioner’s Claims

(1) Agreement grants “living benefits” such as medical check-ups HELD


and hospitalization which a member may immediately enjoy so long
as he is alive upon effectivity of the agreement until its expiration.
YES
(2) Only medical and hospitalization benefits are given under the
agreement without any indemnification, unlike in an insurance
contract where the insured is indemnified for his loss.
Ratio Every person has an insurable interest in the life and health
(3) HCAs are only for a period of one year; therefore, of himself1. The health care agreement was in the nature of non-life
incontestability clause does not apply, as it requires effectivity
period of at least 2 yrs. 1 Sec.10. Every person has an insurable interest in the life and health:

(1) of himself, of his spouse and of his children;

Page 129 of 154


INSURANCE G01 CASE DIGESTS

insurance, which is primarily a contract of indemnity. Once the (e) In consideration of the insurer’s promise, the insured pays a
member incurs hospital, medical or any other expense arising from premium.
sickness, injury or other stipulated contingent, the health care
provider must pay for the same to the extent agreed upon under
the contract. 2. NO

Ratio Where matters of opinion or judgment are called for,


Reasoning answers made in good faith and without intent to deceive will not
avoid a policy even though they are untrue; since in such case the
- A contract of insurance2 is an agreement whereby one undertakes insurer is not justified in relying upon such statement, but is
for a consideration to indemnify another against loss, damage or obligated to make further inquiry.
liability arising from an unknown or contingent event.
Reasoning
- An insurance contract exists where the following elements concur:
- The fraudulent intent on the part of the insured must be
(a) The insured has an insurable interest; established to warrant rescission of the insurance contract. The
right to rescind should be exercised previous to the commencement
(b) The insured is subject to a risk of loss by the happening of the of an action on the contract. No rescission was made. Besides, the
peril; cancellation of health care agreements as in insurance policies
(c) The insurer assumes the risk; requires:

(d) Such assumption of risk is part of a general scheme to distribute (a) Prior notice of cancellation to insured;
actual losses among a large group of persons bearing a similar risk; (b) Notice must be based on the occurrence after effective date of
and the policy of one or more of the grounds mentioned;

(c) Must be in writing, mailed or delivered to the insured at the


address shown in the policy;
(2) of any person on whom he depends wholly or in part for education or support, or in whom he has a pecuniary interest;

(3) of any person under a legal obligation to him for the payment of money, respecting property or service, of which death or illness might (d) Must state the grounds relied upon provided in Section 64 of the
delay or prevent the performance; and
Insurance Code and upon request of insured, to furnish facts on
(4) of any person upon whose life any estate or interest vested in him depends.
which cancellation is based.
2 Section 2 (1) of the Insurance Code

Page 130 of 154


INSURANCE G01 CASE DIGESTS

- These conditions have not been met. When the terms of insurance 142. FORTUNE INSURANCE V. COURT OF APPEALS, 244 SCRA 308
contract contain limitations on liability, courts should construe them (1996)
in such a way as to preclude insurer from non-compliance of
obligation. Being a contract of adhesion, terms of an insurance
FORTUNE VS CA (1995)
contract are to be construed strictly against the party which
prepared it – the insurer. G.R. No. 115278 May 23, 1995
- Also, Philamcare had 12 months from the date of issuance of the
Petitioner: FORTUNE INSURANCE AND SURETY CO., INC. (Fortune)
Agreement within which to contest the membership of the patient
if he had previous ailment of asthma, and six months from the Respondent: PRODUCERS BANK OF THE PHILIPPINES (PBP)
issuance of the agreement if the patient was sick of diabetes or
hypertension. FACTS:

* The health care agreement is in the nature of a contract of > PBP filed against Fortune a complaint for recovery of the sum of
indemnity. Hence, payment should be made to the party who P725,000.00 under the policy issued by Fortune. The money was
incurred the expenses. It is clear that respondent paid all the allegedly lost during a robbery of Producer's armored vehicle while
hospital and medical bills; thus, she is entitled to reimbursement. it was in transit to transfer the money from its Pasay City Branch to
its head office in Makati along Taft Avenue.

>The armored car was driven by Benjamin Magalong escorted by


Disposition Petition DENIED. Security Guard Saturnino Atig.

>Driver Magalong was assigned by PRC Management Systems with


the PBP by virtue of an Agreement and Atiga was assigned by
Unicorn Security Services, Inc. by virtue of a contract of Security
Service.

>After an investigation conducted by the Pasay police authorities,


the driver Magalong and guard Atiga were charged, together with
Edelmer Bantigue, Reynaldo Aquino and John Doe, with violation of
P.D. 532 (Anti-Highway Robbery Law) before the Fiscal of Pasay City.

>Demands were made by PBP but Fortune refused to pay as the loss
is excluded from the coverage of the insurance policy which is
Page 131 of 154
INSURANCE G01 CASE DIGESTS

stipulated under "General Exceptions" Section (b) which reads as HELD:


follows:
YES.
GENERAL EXCEPTIONS
Fortune is exempt from liability under the general exceptions clause
The company shall not be liable under this policy in of the insurance policy.
report of
>It should be noted that the insurance policy entered into by the
xxx xxx xxx parties is a theft or robbery insurance policy which is a form of
casualty insurance (Section 174 of the Insurance Code). Other than
(b) any loss caused by any dishonest, fraudulent or what is mentioned in the provision, the rights and obligations of the
criminal act of the insured or any officer, employee, parties must be determined by the terms of their contract, taking
partner, director, trustee or authorized into consideration its purpose and always in accordance with the
representative of the Insured whether acting alone general principles of insurance law.
or in conjunction with others. . . .
>The purpose of the exception is to guard against liability should the
8. The plaintiff opposes the contention of the theft be committed by one having unrestricted access to the
defendant and contends that Atiga and Magalong property. In such cases, the terms specifying the excluded classes
are not its "officer, employee, . . . trustee or are to be given their meaning as understood in common
authorized representative . . . at the time of the speech. The terms "service" and "employment" are generally
robbery. associated with the idea of selection, control, and compensation.

>RTC & CA: held that there should be recovery. The trial court ruled >A contract of insurance is a contract of adhesion, thus any
that Magalong and Atiga were not employees or representatives of ambiguity therein should be resolved against the insurer, or it
Producers. The wages and salaries of both Magalong and Atiga are should be construed liberally in favor of the insured and strictly
presumably paid by their respective firms, which alone wields the against the insurer. Limitations of liability should be regarded with
power to dismiss them. Neither is the Court prepared to accept the extreme jealousy and must be construed
proposition that driver Magalong and guard Atiga were the in such a way, as to preclude the insurer from non-compliance with
"authorized representatives" of plaintiff. its obligation.

ISSUE: >If the terms of the contract are clear and unambiguous, there is no
room for construction and such terms cannot be enlarged or
W/N the recovery in the policy is precluded under the general diminished by judicial construction.
exceptions clause?

Page 132 of 154


INSURANCE G01 CASE DIGESTS

>An insurance contract is a contract of indemnity. It is settled that companions. In short, for these particular tasks, the three acted as
the terms of the policy constitute the measure of the insurer's agents of Producers. A "representative" is defined as one who
liability. In the absence of statutory prohibition to the contrary, represents or stands in the place of another; one who represents
insurance companies have the same rights as individuals to limit others or another in a special capacity, as an agent, and is
their liability and to impose whatever conditions they deem best interchangeable with "agent."
upon their obligations not inconsistent with public policy.

>It was clear that Fortune’s intention is to exclude and exempt from 143. PERLA COMPANIA SEGUROS V. RAMOLETE , 203 SCRA 487
protection and coverage losses arising from dishonest, fraudulent, (1991)
or criminal acts of persons granted or having unrestricted access to
Producers' money or payroll. When it used then the term PERLA COMPANIA DE SEGUROS, INC., petitioner,
"employee," it must have had in mind any person who qualifies as
such as generally and universally understood, or jurisprudentially vs.
established in the light of the four standards in the determination of
the employer-employee relationship, or as statutorily declared even HON. JOSE R. RAMOLETE, PRIMITIVA Y. PALMES, HONORATO
in a limited sense as in the case of Article 106 of the Labor Code BORBON, SR., OFFICE OF THE PROVINCIAL SHERIFF, PROVINCE OF
which considers the employees under a "labor-only" contract as CEBU, respondents.
employees of the party employing them and not of the party who
supplied them to the employer. FACTS:

>Fortune claims that Producers' contracts with PRC Management  A Cimarron PUJ owned and registered in the name of Nelia
Systems and Unicorn Security Services are "labor-only" contracts. Enriquez, and driven by Cosme Casas, collided with a private
But even granting for the sake of argument that these contracts jeep owned by the late Calixto Palmes (husband of private
were not "labor-only" contracts, and PRC Management Systems and respondent Primitiva Palmes) who was then driving the
Unicorn Security Services were truly independent contractors, we private jeep.
are satisfied that Magalong and Atiga were, in respect of the  The impact of the collision was such that the private jeep
transfer of Producer's money from its Pasay City branch to its head was flung away to a distance of about thirty (30) feet and
office in Makati, its "authorized representatives" who served as then fell on its right side pinning down Calixto Palmes. He
such with its teller Maribeth Alampay. died as a result of cardio-respiratory arrest due to a crushed
chest. It also caused physical injuries on the part of
>Producers entrusted the three with the specific duty to safely Adeudatus Borbon who was then only two (2) years old.
transfer the money to its head office, with Alampay to be  Private respondents Primitiva Palmes (widow of Calixto
responsible for its custody in transit; Magalong to drive the armored Palmes) and Honorato Borbon, Sr. (father of minor
vehicle which would carry the money; and Atiga to provide the Adeudatus Borbon) filed a complaint with the CFI Judge Jose
needed security for the money, the vehicle, and his two other

Page 133 of 154


INSURANCE G01 CASE DIGESTS

R. Ramolete ordered the Borbon claim excluded from the clearly had an interest in the proceeds of the third-party liability
complaint it being distinct and separate from that of Palmes insurance contract. In a third-party liability insurance contract, the
 CFI: in favor of Palmes insurer assumes the obligation of paying the injured third party to
 Judgment became final and executory, writ of execution whom the insured is liable.
issued but returned unsatisfied
 The judgment debtor Nelia Enriquez was summoned. She The insurer becomes liable as soon as the liability of the insured to
declared under oath that the Cimarron PUJ registered in her the injured third person attaches. Prior payment by the insured to
name was covered by a third-party liability insurance policy
the injured third person is not necessary in order that the obligation
issued by petitioner Perla.
 Palmes filed a motion for garnishment. The Judge issued an of the insurer may arise. From the moment that the insured became
Order directing the Provincial Sheriff or his deputy to liable to the third person, the insured acquired an interest in the
garnish the third-party liability insurance policy. insurance contract, which interest may be garnished like any other
 Perla filed a MR alleging that the writ was void on the credit.
ground that it (Perla) was not a party to the case and that
jurisdiction over its person had never been acquired by the
trial court
Thus certiorari and prohibition alleging grave abuse of discretion on
the part of respondent Judge Ramolete in ordering garnishment of
the third-party liability insurance contract issued by petitioner Perla
in favor of the judgment debtor, Nelia Enriquez.

ISSUE: Can Perla be made liable? - YES

RATIO:

In the present case, there can be no doubt, therefore, that the trial
court actually acquired jurisdiction over petitioner Perla when it was
served with the writ of garnishment of the third-party liability
insurance policy it had issued in favor of judgment debtor Nelia
Enriquez. Perla cannot successfully evade liability thereon by such a
contention.

Every interest which the judgment debtor may have in property


may be subjected to execution. judgment debtor Nelia Enriquez

Page 134 of 154


INSURANCE G01 CASE DIGESTS

144. SHAFER V. JUDGE OF RTC, 167 SCRA 386 (1986)  Upon motion, petitioner was granted leave by the former
presiding judge of the trail court to file a third party
complaint against the herein private respondent, Makati
Shafer v Hon Judge of RTC Insurance Company, Inc. Said insurance company,
however, moved to vacate the order granting leave to
Facts: petitioner to file a third party complaint against it and/or
to dismiss the same.
 Petitioner Sherman Shafer obtained a private car policy  The court a quo issued an order dismissing the third party
over his Ford Laser car from Makati Insurance Company, complaint on the ground that it was premature, based on
Inc., for third party liability (TPL).> During the effectivity of the premise that unless the accused (herein petitioner) is
the policy, an information for reckless imprudence resulting found guilty and sentenced to pay the offended party
in damage to property and serious physical injuries was filed (Poblete Sr.) indemnity or damages, the third party
against petitioner. The information reads as follows: “the complaint is without cause of action. The court further
above-named accused, being then the driver and in actual stated that the better procedure is for the accused
physical control of a Ford Laser car did then and there (petitioner) to wait for the outcome of the criminal aspect
wilfully, unlawfully and criminally drive, operate and of the case to determine whether or not the accused, also
manage the said Ford Laser car in a careless, reckless and the third party plaintiff, has a cause of action against the
imprudent manner without exercising reasonable caution, third party defendant for the enforcement of its third party
diligence and due care to avoid accident to persons and liability (TPL) under the insurance contract. Petitioner
damage to property and in disregard of existing traffic rules moved for reconsideration of said order, but the motion
and regulations, causing by such carelessness, recklessness was denied; hence, this petition.
and imprudence the said Ford Laser car to hit and bump a
Volkswagen car… and as a result thereof one Jovencio ISSUE: WON the court a quo erred in dismissing petitioner's third
Poblete, Sr. who was on board of the said Volkswagen car party complaint on the ground that petitioner had no cause of
sustained physical injuries…which injuries causing deformity action yet against the insurance company (third party defendant).
on the face."
 The owner of the damaged Volkswagen car filed a separate HELD: YES
civil action against petitioner for damages, while Jovencio
Poblete, Sr., who was a passenger in the Volkswagen car There is no need on the part of the insured to wait for the decision
when allegedly hit and bumped by the car driven by of the trial court finding him guilty of reckless imprudence. The
petitioner, did not reserve his right to file a separate civil occurrence of the injury to the third party immediately gave rise to
action for damages. Instead, in the course of the trial in the the liability of the insurer under its policy
criminal case, Poblete, Sr. testified on his claim for damages
for the serious physical injuries which he claimed to have Compulsory Motor Vehicle Liability Insurance (third party liability,
sustained as a result of the accident. or TPL) is primarily intended to provide compensation for the

Page 135 of 154


INSURANCE G01 CASE DIGESTS

death or bodily injuries suffered by innocent third parties or A third party complaint is a device allowed by the rules of
passengers as a result of a negligent operation and use of motor procedure by which the defendant can bring into the original suit a
vehicles. The victims and/or their dependents are assured of party against whom he will have a claim for indemnity or
immediate financial assistance, regardless of the financial capacity remuneration as a result of a liability established against him in
of motor vehicle owners. the original suit. Third party complaints are allowed to minimize the
number of lawsuits and avoid the necessity of bringing two (2) or
The liability of the insurance company under the Compulsory Motor more actions involving the same subject matter. They are
Vehicle Liability Insurance is for loss or damage. Where an insurance predicated on the need for expediency and the avoidance of
policy insures directly against liability, the insurer's liability accrues unnecessary lawsuits. If it appears probable that a second action
immediately upon the occurrence of the injury or event upon which will result if the plaintiff prevails, and that this result can be avoided
the liability depends, and does not depend on the recovery of by allowing the third party complaint to remain, then the motion to
judgment by the injured party against the insured. dismiss the third party complaint should be denied.

The injured for whom the contract of insurance is intended can sue Respondent insurance company's contention that the third party
directly the insurer. The general purpose of statutes enabling an complaint involves extraneous matter which will only clutter,
injured person to proceed directly against the insurer is to protect complicate and delay the criminal case is without merit. An offense
injured persons against the insolvency of the insured who causes causes two (2) classes of injuries the first is the social injury
such injury, and to give such injured person a certain beneficial produced by the criminal act which is sought to be repaired thru the
interest in the proceeds of the policy, and statutes are to be liberally imposition of the corresponding penalty, and the second is the
construed so that their intended purpose may be accomplished. It personal injury caused to the victim of the crime, which injury is
has even been held that such a provision creates a contractual sought to be compensated thru indemnity, which is civil in nature.
relation which inures to the benefit of any and every person who
may be negligently injured by the named insured as if such injured In the instant case, the civil aspect of the offense charged, i.e.,
person were specifically named in the policy. serious physical injuries allegedly suffered by Jovencio Poblete, Sr.,
was impliedly instituted with the criminal case. Petitioner may thus
In the event that the injured fails or refuses to include the insurer as raise all defenses available to him insofar as the criminal and civil
party defendant in his claim for indemnity against the insured, the aspects of the case are concerned. The claim of petitioner for
latter is not prevented by law to avail of the procedural rules payment of indemnity to the injured third party, under the
intended to avoid multiplicity of suits. Not even a "no action" clause insurance policy, for the alleged bodily injuries caused to said third
under the policy-which requires that a final judgment be first party, arose from the offense charged in the criminal case, from
obtained against the insured and that only thereafter can the which the injured (Jovencio Poblete, Sr.) has sought to recover civil
person insured recover on the policy can prevail over the Rules of damages. Hence, such claim of petitioner against the insurance
Court provisions aimed at avoiding multiplicity of suits. company cannot be regarded as not related to the criminal action.

Page 136 of 154


INSURANCE G01 CASE DIGESTS

145. MALAYAN INSURANCE V. CA, VALLEJOS ET AL .,, 165 SCRA 136


(1998) 4. PANTRANCO claimed that the jeep of Sio Choy was operated at
an excessive speed and bumped the PANTRANCO bus. It prayed that
it be absolved from any and all liability. Defendant Sio Choy and the
CASUALTY INSURANCE
Insurance Company denied the liability, claiming that the fault in
144. MALAYAN INSURANCE CO., INC.,, vs. the accident was solely imputable to the PANTRANCO.

THE HON. COURT OF APPEALS, MARTIN C. VALLEJOS,


5. SIO CHOY'S CONTENTION

FACTS • alleged that he had actually paid the Matrin Vallejos, the
amount of P5,000.00 for the hospitalization and other expenses
1. Petitioner Malayan issued in favor of the respondent
Sio Choy Private Comprehensive Car Comprehensive covering a • He alleged that Malayan had issued in his favor a private
Willys Jeep. car comprehensive car, wherein the insurance company obligated
itself to indemnify Sio Choy, as insured for the damge to his motor
• The insurance coverage was for own damage not exceeding vehicle as well as the liability to third persin arrising to any vehicular
P600.00 and third party liability in the amount of P20,000 accident

(which the policy is in full force and effect when the vehicular
accident complained of occurred.
2. During the effectivity of the insurance policy, the insured jeep
while being driven by Juan Campollo, an employee of the • He prayed that he be reimbursed by the insurance company for
respondent San Leon Rice Mill collided with passenger bus the amount that he may be ordered to pay
belonging to the respondent PANTRANCO at the national
highway causing damage to the insured vehicle and injuries to • File a third party complaint against San Leon, since the driver
the driver and Respondent Martin Vallejos was its employee

6. RTC ruled in favor of the Martin Vallejos and against Sio


Choy and Malayan Insurance and third party defendant San Leon
3. RESPONDENT VALLEJOS filed an action for damages afaist Sio Roy Rice
Malayan and PANTRANCO before the RTC.

Page 137 of 154


INSURANCE G01 CASE DIGESTS

CA affirmed the decision. They are jointly and severally liable for It must be observed that Respondent Sio Choy is made liable to
the damages of Martin Vallejos. However, San Leon Rice Mill has Vallejos as the owner of the ill- fated Will Jeep pursuant to Article
no obligation to indemnify or reimburse the petitioner 2184 of the Civil Code.
insurance company since it is not a privy to the contract of
◦ Art. 2184. In motor vehicle mishaps, the owner is solidarily
insurance between Sio Choy and the insurance company.
liable with his driver, if the

former, who was in the vehicle, could have, by the use of due
ISSUES: diligence, prevented the misfortune it is disputably presumed that
a driver was negligent, x x x
(1) Whether the trial court, as upheld by the Court of Appeals,
was correct in holding petitioner and respondents Sio Choy and The basis of liability of Respondent San Leon to Vallejos, the former
San Leon Rice Mill, Inc. "solidarily liable" to respondent Vallejos; and being the employer of the driver of the Willys Jeep at the time
of the motor vehicle mishap is Article 2180 of the Civil Code

◦ Art. 2180. The obligation imposed by article 2176 is


(2) Whether petitioner is entitled to be reimbursed by respondent demandable not only for one's own acts or omissions, but also
San Leon Rice Mill, Inc. for whatever amount petitioner has been
for those of persons for whom one is responsible.
adjudged to pay respondent Vallejos on its insurance policy.
◦ Employers shall be liable for the damages caused by their
employees and household helpers acting within the scope of their
RATIONALE: assigned tasks, even though the former are not engaged ill any
business or industry. Sio Choy and San Leon Rice Mill are the
In order to determine the alleged liability of respondent San Leon principal tortfeasors who are primarily liable to Vallejos. The law
Rice Mill to Malayan, it is important to determine first the nature or states that the responsibility of two or more persons who are liable
basis of the liability of Malayan to Respondent Vallejos, as for quasi-delict is solidarily.
compared to that of respondents Sio and San Leon Rice Mill.
ON THE OTHER HAND, the basis of petitioner's liability is its
1. No."ONLY" RESPONDENTS SIO CHOY AND SAN LEON RICE insurance contract with respondent Sio Choy. If petitioner is
ARE SOLIDARILY LIABLE TO VALLEJOS. MALAYAN IS NOT INCLUDED. adjudged to pay respondent Vallejos in the amount of not more
than P20,000.00, this is on account of its being the insurer of
respondent Sio Choy under the third party liability clause included
in the private car comprehensive policy existing between petitioner
Page 138 of 154
INSURANCE G01 CASE DIGESTS

and respondent Sio Choy at the time of the complained vehicular 146. HEIRS OF G.Y. POE V. MALAYAN INSURANCE , 584 SCRA 152
accident (2009)

2. Yes. Malayan has the right to be reimbursed. There is a right of Heirs of GY Poe vs Malayan Insurance
Subrogation. Petitioner, upon paying respondent Vallejos the
amount of riot exceeding P20,000.00, shall become the subrogee Facts:
1. George Y. Poe (George) while waiting for a ride to work in
of the insured, the respondent Sio Choy; as such, it is subrogated to
front of Capital Garments Corporation, Ortigas Avenue
whatever rights the latter has against respondent San Leon Rice Extension, Barangay Dolores, Taytay, Rizal, was run over by
Mill, Inc. Article 1217 of the Civil Code gives to a solidary debtor a ten-wheeler Isuzu hauler truck
who has paid the entire obligation the right to be reimbursed by his 2. The truck was owned by Rhoda Santos (Rhoda), and then
co-debtors for the share which corresponds to each. being driven by Willie Labrador (Willie).
3. To seek redress for George’s untimely death, his heirs and
In accordance with Article 1217, petitioner, upon payment herein petitioners, namely, his widow Emercelinda, and
to respondent Vallejos and thereby becoming the subrogee of their children Flerida and Fernando, filed with the RTC a
solidary debtor Sio Choy, is entitled to reimbursement from Complaint for damages
4. Malayan Insurance was named respondent in this case since
respondent San Leon Rice Mill, Inc.
they are the insurer of Rhoda Santos over the subject truck.
SC hold that only respondents Sio Choy and San Leon Rice Mill, Inc. 5. Under the insurance policy of the said truck Malayan binds
itself, among others, to be liable for damages as well as any
are solidarily liable to the respondent Martin
bodily injury to third persons which may be caused by the
C. Vallejos for the amount of P29,103.00. Vallejos may enforce the operation of the insured vehicle.
6. In their answer, Malayan insurance argued that their
entire obligation on only one of said solidary debtors. If Sio Choy as
liability will only attach if there is a judicial pronouncement
solidary debtor is made to pay for the entire obligation that the insured Rhoda and her driver are liable.
(P29,103.00) and petitioner, as insurer of Sio Choy, is 7. They of course argued that the accident was caused by the
compelled to pay P20,000.00 of said entire obligation, petitioner negligence of George.
would be entitled, as subrogee of Sio Choy as against San 8. RTC Ruled that Rhoda and Malayan are liable solidarily.
Leon Rice Mills, Inc., to be reimbursed by the latter in the amount 9. CA Set aside the RTC decision.
of P14,551.50 (which is 1/2 of P29,103.00 )
Issue:
1. Whether or not Malayan Insurance should be held solidarily
liable with Rhoda.

Held:

Page 139 of 154


INSURANCE G01 CASE DIGESTS

1. Court rundle that it is up to petitioner to hold them either respondent Malayan; petitioners can recover the said
solidary or joint. Nevertheless, Malayan IS LIABLE amounts in full from either of them, thus, making their
2. Respondent MICI does not deny that it is the insurer of the liabilities solidary or joint and several.
truck. Nevertheless, it asserts that its liability is limited, and
it should not be held solidarily liable with Rhoda for all the
damages awarded to petitioners. 147. VDA. DE MAGLANA VS. CONSOLACION, 212 SCRA 268 (1992)
3. It is settled that where the insurance contract provides for
indemnity against liability to third persons, the liability of Vda. De Magdalena vs. Consolacion and AFISCO Insurance Corp
the insurer is direct and such third persons can directly sue (1992)
the insurer.
4. The direct liability of the insurer under indemnity contracts Facts:
against third party liability does not mean, however, that 1. Lope Maglana was an employee of the Bureau of Customs
the insurer can be held solidarily liable with the insured whose work station was at Lasa, Davao City. On December
and/or the other parties found at fault, since they are being 20, 1978, early morning, Lope Maglana was on his way to
held liable under different obligations. his work, driving a motorcycle, he met an accident that
5. The liability of the insured carrier or vehicle owner is based resulted to his death. He died on the spot.
on tort, in accordance with the provisions of the Civil Code 2. The PUJ jeep that bumped the deceased was driven by
6. The liability of the insurer arises from contract, particularly, Pepito Into, operated and owned by defendant Destrajo.
the insurance policy. 3. From the investigation conducted by the traffic investigator,
7. The third-party liability of the insurer is only up to the the PUJ jeep was overtaking another passenger jeep that
extent of the insurance policy and that required by law; and was going towards the city poblacion. While overtaking, the
it cannot be held solidarily liable for anything beyond that PUJ jeep of defendant Destrajo running abreast with the
amount. overtaken jeep, bumped the motorcycle driven by the
8. In the case of Vda de Maglana vs Colsolacion if indeed deceased who was going towards the direction of Lasa,
Malayan have limited liability under the insurance policy Davao City. The point of impact was on the lane of the
petitioners would have had the option either (1) to claim motorcycle and the deceased was thrown from the road
the amount awarded to them from respondent Malayan, up and met his untimely death.
to the extent of the insurance coverage, and the balance 4. Maglana’s heirs filed an action for damages and attorney’s
from Rhoda; or (2) to enforce the entire judgment against fees against Destrajo and AFISCO Insurance Corporation. An
Rhoda, subject to reimbursement from respondent MICI to information for homicide thru reckless imprudence was also
the extent of the insurance coverage. filed against Pepito Into.
9. However, Malayan failed to present the insurance contract 5. The CFI ruled that Destrajo had not exercised sufficient
to prove their limited liability. diligence as the operator of the jeepney, ordering him to
10. As such there is no more difference in the amounts of pay the heirs for damages. AFISCO Insurance Company was
damages which petitioners can recover from Rhoda or ordered to reimburse Destrajo whatever amounts the

Page 140 of 154


INSURANCE G01 CASE DIGESTS

latter shall have paid only up to the extent of its insurance


coverage. The above-quoted provision leads to no other conclusion but that
6. Petitioner heirs filed an MR, contending that AFISCO should AFISCO can be held directly liable by petitioners.
not merely be held secondarily liable because the Insurance
Code provides that the insurer's liability is "direct and As this Court ruled in Shafer vs. Judge, RTC of Olongapo City "where
primary and/or jointly and severally with the operator of an insurance policy insures directly against liability, the insurer's
the vehicle, although only up to the extent of the insurance liability accrues immediately upon the occurrence of the injury or
coverage." Hence, they argued that the P20,000.00 even upon which the liability depends, and does not depend on the
coverage of the insurance policy issued by AFISCO, should recovery of judgment by the injured party against the insured."
have been awarded in their favor.
7. AFISCO argued that since the Insurance Code does not The underlying reason behind the third party liability (TPL) of the
expressly provide for a solidary obligation, the presumption Compulsory Motor Vehicle Liability Insurance is "to protect injured
is that the obligation is joint. persons against the insolvency of the insured who causes such
8. Petitioners reassert their position that the insurance injury, and to give such injured person a certain beneficial interest
company is directly and solidarily liable with the negligent in the proceeds of the policy."
operator up to the extent of its insurance coverage.
Since petitioners had received from AFISCO the sum of P5,000.00
Issue: W/N the insurance company is directly and solidarily liable under the no-fault clause, AFISCO's liability is now limited to
with the negligent operator up to the extent of its insurance P15,000.00.
coverage
However, we cannot agree that AFISCO is likewise solidarily liable
Held: Directly liable with the operator, YES. Solidarily liable with the with Destrajo.
operator, NO.
In Malayan Insurance Co., Inc. v. Court of Appeals, this Court had
The particular provision of the insurance policy on which petitioners the opportunity to resolve the issue as to the nature of the liability
base their claim is as follows: Sec. 1 — LIABILITY TO THE PUBLIC of the insurer and the insured vis-a-vis the third party injured in an
accident. We categorically ruled thus: While it is true that where the
1. The Company will, subject to the Limits of Liability, pay all sums insurance contract provides for indemnity against liability to third
necessary to discharge liability of the insured in respect of persons, such third persons can directly sue the insurer,
(a) death of or bodily injury to any THIRD PARTY however, the direct liability of the insurer under indemnity contracts
.... against third party liability does not mean that the insurer can be
3. In the event of the death of any person entitled to indemnity held solidarily liable with the insured and/or the other parties found
under this Policy, the Company will, in respect of the liability at fault. The liability of the insurer is based on contract; that of the
incurred to such person indemnify his personal representatives in insured is based on tort.
terms of, and subject to the terms and conditions hereof.

Page 141 of 154


INSURANCE G01 CASE DIGESTS

The Court then proceeded to distinguish the extent of the liability 148. GSIS V. COURT OF APPEALS, 308 SCRA 559 (1999)
and manner of enforcing the same in ordinary contracts from that
of insurance contracts. While in solidary obligations, the creditor GSIS vs CA
may enforce the entire obligation against one of the solidary GR No. 101439 June 21, 1999
debtors, in an insurance contract, the insurer undertakes for a
consideration to indemnify the insured against loss, damage or Facts:
liability arising from an unknown or contingent event. To make the  National Food Authority (NFA, formerly National Grains
insurer solidarily liable with the latter’s entire obligation beyond the Authority) owned a Chevrolet truck which was insured
sum limited in the insurance contract would result in an evident against liabilities for death of and injuries to third persons
breach of the concept of solidary obligation. with GSIS (third-person liability).
 The truck, driven by Corbeta, collided with a public utility
Similarly, petitioners herein cannot validly claim that AFISCO, whose vehicle, a Toyota Tamaraw, owned and operated by Victor
liability under the insurance policy is P20,000.00, can be held Uy, under the name of "Victory Line." The tamaraw was a
solidarily liable with Destrajo for the total amount of P53,901.70 in total wreck and all the collision victims were its passengers.
accordance with the decision of the lower court. 5 passengers dies, while 10 sustained bodily injuries.
 3 cases were filed with CFU of Agusan del Norte and Butuan
Since under both the law and the insurance policy, AFISCO's liability City:
is only up to P20,000.00, the second paragraph of the dispositive 1. quasi-delict, damages and attorney's fees by Uy;
portion of the decision in question may have unwittingly sown 2. damages by Taer, an injured passenger, against Uy
confusion among the petitioners and their counsel. What should and insurer Mabuhay Insurance (MIGC), to which
have been clearly stressed as to leave no room for doubt was the Uy filed a cross-claim against MIGC and third-party
liability of AFISCO under the explicit terms of the insurance contract. complaint against Corbeta and NFA;
3. actions by heirs and victims of the deceased for
In fine, we conclude that the liability of AFISCO based on the damages due to quasi-delict against NFA and
insurance contract is direct, but not solidary with that of Destrajo Corbeta, against GSIS as insurer of the truck, against
which is based on Article 2180 of the Civil Code. Uy for breach of contract of carriage, against MIGC
as insurer of the tamaraw.
As such, petitioners have the option either to claim the P15,000  RTC: Corbeta's negligence was the proximate cause of the
from AFISCO and the balance from Destrajo or enforce the entire collision; the truck which crossed over to the other lane was
judgment from Destrajo subject to reimbursement from AFISCO to speeding because after the collision, its left front wheel was
the extent of the insurance coverage. detached and fell into a ravine.
 RTC: first case - Uy was awarded 109,100 for damages; 2nd
case - dismissed against Uy, ordered MIGC, Corbeta and
NFA to pay Taer; 3rd case - damages are also awarded to
the heirs and victims.

Page 142 of 154


INSURANCE G01 CASE DIGESTS

 CA affirmed the ruling of RTC who may be negligently injured by the named insured as if
such injured person were specifically named in the policy.
Issue:  However, although the victim may proceed directly against
1. Whether GSIS should be held solidarily liable with the negligent the insurer for indemnity, the third party liability is only up
insured/owner-operator of the Chevrolet truck for damages to the extent of the insurance policy and those required by
awarded to private respondents which are beyond the limitations of law. While it is true that where the insurance contract
the insurance policy and the Insurance Memorandum Circular No. 5- provides for indemnity against liability to third persons, and
78 such third persons can directly sue the insurer, the direct
2. Whether the private respondents have no cause of action against liability of the insurer under indemnity contracts against
the petitioner, allegedly for failure of the victims to file an insurance third party liability does not mean that the insurer can be
claim within six (6) months from the date of the accident held liable in solidum with the insured and/or the other
parties found at fault. For the liability of the insurer is based
Ruling: on contract; that of the insured carrier or vehicle owner is
1. The liability of GSIS is direct, but not solidary with that of the based on tort. The liability of GSIS based on the insurance
owner and negligent driver, and its liability is only to the extent of contract is direct, but not solidary with that of the NFA. The
the insurance policy and those required by law. latter’s liability is based separately on Article 2180 of the
 Compulsory Motor Vehicle Liability Insurance (third party Civil Code.
liability, or TPL) is primarily intended to provide  As therein provided, the maximum indemnity for death was
compensation for the death or bodily injuries suffered by twelve thousand (P12,000.00) pesos per victim. The
innocent third parties or passengers as a result of a schedules for medical expenses were also provided by said
negligent operation and use of motor vehicles. The victims IMC, specifically in paragraphs (C) to (G). Consequently,
and/or their dependents are assured of immediate financial heirs of the victims who died in the May 9, 1979 vehicular
assistance, regardless of the financial capacity of motor incident, could proceed (1) against GSIS for the indemnity of
vehicle owners. P12,000 for each dead victim, and against NFA and
 The injured for whom the contract of insurance is intended Guillermo Corbeta for any other damages or expenses
can sue directly the insurer. The general purpose of statutes claimed; or (2) against NFA and Corbeta to pay them all
enabling an injured person to proceed directly against the their claims in full.
insurer is to protect injured persons against the insolvency
of the insured who causes such injury, and to give such 2. Private respondents have a cause of action.
injured person a certain beneficial interest in the proceeds  Evidence shows that a notice of loss was sent to GSIS on
of the policy, and statutes are to be liberally construed so September 7, 1979, almost 4 months after the incident
that their intended purpose may be accomplished. It has (May 9, 1979). Likewise, GSIS did not deny receipt of notice
even been held that such a provision creates a contractual of claim during the trial, and it would be too late to state
relation which inures to the benefit of any and every person otherwise.

Page 143 of 154


INSURANCE G01 CASE DIGESTS

 A delay in reporting the loss by the insured and/or by the rendered a favorable response for the respondent. The appellate
beneficiaries must be promptly raised by the insurer in court ruled likewise.
objecting to the claims.
 The insurer failed to object to the presentation of the proof Petitioner filed this petition alleging grave abuse of
of loss before the trial court. The defense of delay should discretion on the part of the appellate court in applying the
have been promptly interposed. Moreover, the petitioner principle of "expresso unius exclusio alterius" in a personal accident
merely waited for the victims or beneficiaries to file their insurance policy, since death resulting from murder and/or assault
complaint. As matters stand now, the defense of laches or are impliedly excluded in said insurance policy considering that the
prescription is deemed waived because of petitioner’s cause of death of the insured was not accidental but rather a
failure to raise it not only before but also during the deliberate and intentional act of the assailant. Therefore, said death
hearing. was committed with deliberate intent which, by the very nature of a
personal accident insurance policy, cannot be indemnified.

149. FINMAN GEN. ASSURANCE V. COURT OF APPEALS, 213 SCRA ISSUE: Whether or not the insurer is liable for the payment of the
493 (1992) insurance premiums?

HELD:
FINMAN GENERAL ASSURANCE CORPORATION vs.THE
HONORABLE COURT OF APPEALS Yes, the insurer is still liable.

FACTS: Contracts of insurance are to be construed liberally in favor


of the insured and strictly against the insurer. Thus ambiguity in the
On October 22, 1986, deceased, Carlie Surposa was insured words of an insurance contract should be interpreted in favor of its
with petitioner Finman General Assurance Corporation with his beneficiary. The terms "accident" and "accidental" as used in
parents, spouses Julia and Carlos Surposa, and brothers Christopher, insurance contracts have not acquired any technical meaning, and
Charles, Chester and Clifton, all surnamed, Surposa, as beneficiaries. are construed by the courts in their ordinary and common
While said insurance policy was in full force and effect, the insured, acceptation. Thus, the terms have been taken to mean that which
Carlie Surposa, died on October 18, 1988 as a result of a stab wound happen by chance or fortuitously, without intention and design, and
inflicted by one of the three (3) unidentified men. Private which is unexpected, unusual, and unforeseen. Where the death or
respondent and the other beneficiaries of said insurance policy filed injury is not the natural or probable result of the insured's voluntary
a written notice of claim with the petitioner insurance company act, or if something unforeseen occurs in the doing of the act which
which denied said claim contending that murder and assault are not produces the injury, the resulting death is within the protection of
within the scope of the coverage of the insurance policy. Private the policies insuring against death or injury from accident. In the
respondent filed a complaint with the Insurance Commission which case at bar, it cannot be pretended that Carlie Surposa died in the
course of an assault or murder as a result of his voluntary act

Page 144 of 154


INSURANCE G01 CASE DIGESTS

considering the very nature of these crimes. Neither can it be said


that where was a capricious desire on the part of the accused to
expose his life to danger considering that he was just going home
after attending a festival.

Furthermore, the personal accident insurance policy


involved herein specifically enumerated only ten (10) circumstances
wherein no liability attaches to petitioner insurance company for
any injury, disability or loss suffered by the insured as a result of any
of the stimulated causes. The principle of " expresso unius exclusio
alterius" — the mention of one thing implies the exclusion of
another thing — is therefore applicable in the instant case since
murder and assault, not having been expressly included in the
enumeration of the circumstances that would negate liability in said
insurance policy cannot be considered by implication to discharge
the petitioner insurance company from liability for, any injury,
disability or loss suffered by the insured. Thus, the failure of the
petitioner insurance company to include death resulting from
murder or assault among the prohibited risks leads inevitably to the
conclusion that it did not intend to limit or exempt itself from
liability for such death.

Page 145 of 154


INSURANCE G01 CASE DIGESTS

SURETYSHIP
(NOTE: Article 2047. By guaranty, a person, called the guarantor,
binds himself to the creditor to fulfill the obligation of the
150. PHILIPPINE PRYCE ASSURANCE V. COURT OF APPEALS, 230 SCRA
principal debtor in case
164 (1994)
the latter should fail to do so. If a person binds
himself solidarily with the principal debtor, the provisions
G.R. No. 107062 February 21, 1994
of Section 4, Chapter 3,
PHILIPPINE PRYCE ASSURANCE CORPORATION, petitioner,
Title I of this Book (Obligations & Contracts) shall be
vs.
observed. In such case, the contract is called a suretyship.)
THE COURT OF APPEALS, (Fourteenth Division) and GEGROCO,
INC., respondents.
--After the issues had been joined, the case was set for pre-trial
conference.
NOCON, J.:
--Pryce failed to appear during the pre-trial that was reset 3 times
because of the non-appearance of Pryce and/or its counsel during
FACTS:
the scheduled pre-trials.
--Petitioner, Interworld Assurance Corporation (now Philippine
--Pryce also failed to pay the docket fees for the Third-Party
Pryce Assurance Corporation), was sued for collection of sum of
Complaint it filed against his principal Sagum.
money by respondent
--Pryce was considered as in default and respondent was allowed to
Gegroco, Inc.
present evidence ex-parte.
--The complaint alleged that Pryce issued two surety bonds in behalf
of its principal Sagum General Merchandise for P500,000.00 and
RTC---ruled in favor of respondent Gregoco.
P1,000,000.00
respectively.
CA---affirmed the RTC ruling.
--Pryce admitted having executed the said bonds, but denied
liability because allegedly
ISSUE: Whether or not there was a contract of suretyship between
1) the checks which were to pay for the premiums bounced
Pryce and its principal Sagum which makes Pryce solidarily liable to
and were dishonored hence there is no contract to speak of
Gregoco
between Pryce and its
supposed principal Sagum; and
HELD: YES.
2) the bonds were merely to guarantee payment of its
We do not find any reversible error in the conclusion reached by the
principal's obligation, thus, there is a benefit of excussion (a right
court a quo.
under Art. 2066 which only
The Insurance Code states that:
a guarantor may invoke against the creditor
Sec. 177. The surety is entitled to payment of the
wherein the guarantor will point out to the creditor all the
premium as soon as the contract of suretyship or
debtor’s properties in the
bond is perfected and delivered to the obligor. No
Philippines sufficient to cover amount of debt).
Page 146 of 154
INSURANCE G01 CASE DIGESTS

contract of suretyship or bonding shall be valid and Q Will you please present to us the
binding unless and until the premium therefor has aforesaid surety bonds?
been paid, except where the obligee has accepted A. Interworld Assurance Corp.
the bond, in which case the bond becomes valid Surety Bond No. 0029 for P500,000
and enforceable irrespective of whether or not the dated July 24, 1987 and Interworld
premium has been paid by the obligor to the Assurance Corp. Surety Bond No.
surety. . . . (emphasis added) 0037 for P1,000.000 dated October
7, 1987. 20
The above provision outrightly negates petitioner's first defense. In
a desperate attempt to escape liability, petitioner further asserts Likewise attached to the record are exhibits consisting of delivery
that the above provision is not applicable because the respondent invoices addressed to Sagum General Merchandise proving that
allegedly had not accepted the surety bond, hence could not have parts were purchased, delivered and received.
delivered the goods to Sagum Enterprises. This statement clearly On the other hand, petitioner's defense that it did not have
intends to muddle the facts as found by the trial court and which authority to issue a Surety Bond when it did is an admission of fraud
are on record. committed against respondent. No person can claim benefit from
In the first place, petitioner, in its answer, admitted to have issued the wrong he himself committed. A representation made is
the bonds subject matter of the original action.19 Secondly, the rendered conclusive upon the person making it and cannot be
testimony of Mr. Leonardo T. Guzman, witness for the respondent, denied or disproved as against the person relying thereon. 22
reveals the following:
Q. What are the conditions and Petition is DISMISSED for lack of merit.
terms of sales you extended to
Sagum General Merchandise? SO ORDERED.
A. First, we required him to submit
to us Surety Bond to guaranty
payment of the spare parts to be
purchased. Then we sell to them on
90 days credit. Also, we required
them to issue post-dated checks.
Q. Did Sagum General merchandise
comply with your surety bond
requirement?
A. Yes. They submitted to us and
which we have accepted two surety
bonds.

Page 147 of 154


INSURANCE G01 CASE DIGESTS

151. AFP GENERAL INSURANCE V. MOLINA, 556 SCRA 630 (2008) On April 6, 1998, the NLRC affirmed with modification the decision
of the Labor Arbiter. The NLRC found the herein private
AFP General Insurance Corporation vs. Molina respondents constructively dismissed and ordered Radon Security
556 SCRA 630 to pay them their separation pay, in lieu of reinstatement with
2008 backwages, as well as their monetary benefits limited to three
years, plus attorney’s fees equivalent to 10% of the entire amount,
Nature: with Radon Security and Ever Emporium, Inc. adjudged jointly and
severally liable.
This is a petition for review on certiorari of the Decision
dated August 20, 2001 of the Court of Appeals in CA-G.R. SP No.
58763, which dismissed herein petitioner’s special civil action for
Radon Security duly moved for reconsideration, but this was
certiorari. Before the appellate court, petitioner AFP General
denied by the NLRC in its Resolution dated June 22, 1998. Radon
Insurance Corporation (AFPGIC) sought to reverse the Resolution
Security then filed a Petition for Certiorari docketed as G.R. No.
dated October 5, 1999 of the National Labor Relations Commission
134891 with this Court, but we dismissed this petition in our
(NLRC) in NLRC NCR CA-011705-96 for having been issued with
Resolution of August 31, 1998.
grave abuse of discretion.

In dismissing the appeal of AFPGIC, the NLRC pointed out


Facts:
that AFPGIC’s theory that the bond cannot anymore be proceeded
against for failure of Radon Security to pay the premium is
The private respondents are the complainants in a case for
untenable, considering that the bond is effective until the finality of
illegal dismissal, docketed as NLRC NCR Case No. 02-00672-90, filed
the decision. The NLRC stressed that a contrary ruling would allow
against Radon Security & Allied Services Agency and/or Raquel Aquias
respondents to simply stop paying the premium to frustrate
and Ever Emporium, Inc. In his Decision dated August 20, 1996, the
satisfaction of the money judgment.
Labor Arbiter ruled that the private respondents were illegally
dismissed and ordered Radon Security to pay them separation pay,
backwages, and other monetary claims. AFPGIC then moved for reconsideration, but the NLRC
denied the motion in its Resolution dated February 29, 2000.
Radon Security appealed the Labor Arbiter’s decision to
public respondent NLRC and posted a supersedeas bond, issued by AFPGIC then filed a special civil action for certiorari,
herein petitioner AFPGIC as surety. docketed as CA-G.R. SP No. 58763, with the Court of Appeals, on the

Page 148 of 154


INSURANCE G01 CASE DIGESTS

ground that the NLRC committed a grave abuse of discretion in 152. REPARATIONS COM. V. UNIVERSAL DEEP SEA FISHING 83 SCRA
affirming the Order dated March 30, 1999 of the Labor Arbiter. 764 (1978)

Reparations Commission vs. Universal Deep-Sea Fishing Corp.


Issue:
Nos. L-21901 and L-21996; June 27, 1
WON THE COURT OF APPEALS SERIOUSLY ERRED IN SUSTAINING
THE PUBLIC RESPONDENT NLRC ALTHOUGH THE LATTER GRAVELY
ABUSED ITS DISCRETION WHEN IT ARBITRARILY IGNORED THE FACT
Universal Deep-Sea Fishing was awarded 6 trawl boats by the
THAT SUBJECT APPEAL BOND WAS ALREADY CANCELLED FOR NON-
Reparations Commission as end-user of reparations goods. The six
PAYMENT OF PREMIUM AND THUS IT COULD NOT BE SUBJECT OF
fishing boats were delivered to Universal two a time. In each
EXECUTION OR GARNISHMENT.
transaction, Universal executed a performance bond with Manila
Surety as the Surety in favor of the Reparations Commission. A
Held:
corresponding indemnity agreement was executed to indemnify the
surety company for any damage, loss, charges, etc., which it may
The filing of a cash or surety bond is a jurisdictional requirement in
sustain or incur as a consequence of having become surety upon the
an appeal involving monetary award, and the bond shall be in effect
performance bond.
until the final disposition of the case. A surety bond, once accepted
by the obligee (the employee to whom money benefits were due),
becomes valid and enforceable, irrespective of whether or not the
premiums thereon have been paid by the obligor (the employer Facts:
liable for payment).
 Universal Deep-Sea Fishing Corporation was awarded six (6)
trawl boats by the Reparations Commission as end-user of
reparations goods. These fishing boats, christened the M/S
UNIFISH 1, M/S UNIFISH 2, M/S UNIFISH 3, M/S UNIFISH 4, M/S
UNIFISH 5, and M/S UNIFISH 6, were delivered to Universal two
at a time, f.o.b. Japanese port.

 M/S UNIFISH 1 and M/S UNIFISH 2, with an aggregate purchase


price of P536,428.44, were delivered to Universal on November
20, 1958 and the Contract of Conditional Purchase and Sale of
Reparations Goods executed by the parties on February 12, 1960
Page 149 of 154
INSURANCE G01 CASE DIGESTS

provided among others, that “the first installment representing  The delivery of the M/S UNIFISH 5 and M/S UNIFISH 6 is covered
10% of the amount or P53,642.84 shall be paid within 24 months by a contract for the Utilization of Reparations Goods executed
from the date of complete delivery thereof. The due date of the by the parties on February 12, 1960. The first installment (10% of
first installment was on May 8, 1961 and the term is for the F.O.B. Cost), P54,500.00, was due October 17, 1961. The term is
payment of purchase price in ten (10) equal yearly installments for the payment of ten (1) equal yearly installments beginning
beginning on May 8, 1962. October 17, 1962. A performance bond in judgment, amount of
P54,500.00 issued by judgment, Manila Surety & Fidelity Co.,
Inc., was submitted, and an indemnity agreement was executed
 To guarantee the faithful compliance with the obligations under by Universal in favor of judgment, surety company.
said contract, a performance bond in the amount of P53,640.00,
with Universal as principal and the Manila Surety & Fidelity Co.,
Inc., as surety, was executed in favor of the Reparations  On August 10, 1962, the Reparations Commission filed an action
Commission. A Corresponding indemnity agreement was against Universal and Manila Surety to recover various amounts
executed to indemnify the surety company for any damage, loss of money due under these contracts. Universal claimed that the
charges, etc., which it may sustain or incur as a consequence of amounts of money sought to be collected are not yet due and
having become a surety upon the performance bond. demandable.

 M/S UNIFISH 3 and M/S UNIFISH 4, with a total purchase price of  Manila Surety also contended that the action is premature, but
P687,777.76 were delivered to Universal on April 20, 1959 and set up a cross-claim against Universal for reimbursement of
the Contract of Conditional Purchase and Sale of Reparation whatever amount of money it may have to pay by the plaintiff by
Goods, dated November 25, 1959, provided that “the first reason of the complaint, including interest, and for the collection
installment representing 10% of the amount or P68,777.77 shall of accumulated and unpaid premiums on the bonds with interest
be paid within 24 months from the date of complete delivery thereon. It also filed a third-party complaint against Pablo S.
thereof. The due date of the first installment was on July 1961 Sarmiento, one of the indemnitors in the indemnity agreements.
and the term is for the payment of ten (10) equal yearly The latter denied personal liability claiming that he signed the
installments beginning July, 1962. agreements in question as his capacity as acting general manage
of Universal.

 A performance bond in the amount of P68,777.77, issued by the


Manila Surety & Fidelity Co., Inc., was also submitted to  The Trial Court rendered judgment ordering Universal to pay the
guarantee the faithful compliance with the obligations set forth Reparations Commission the amount they were seeking to
in the contract, and indemnity agreement was executed in favor recover. Manila Surety & Fidelity was also ordered to pay, jointly
of the surety company in consideration of the said bond. and severally with Universal, P53,643.00, P68,777.77, and
P54.058.00 in favor of the plaintiff.

Page 150 of 154


INSURANCE G01 CASE DIGESTS

 Universal and Pablo Sarmiento were also ordered to pay, jointly 153. ARRANZ V. MANILA FIDELITY, 101 PHIL. 272 (1957)
and severally, Manila Surety, P53,643.00 and P68,777.77 with
12% interest per year from Aug. 10, 1962 until fully paid plus MELECIO ARRANZ vs. MANILA FIDELITY AND SURETY CO., INC
P2,000.00 as attorney’s fees. Universal was likewise ordered to
pay Manila Surety P54,508.00 with 12% interest per year from G.R. No. L-9674 April 29, 1957
August 10, 1962 until fully paid.
FACTS:

1. Manila Fidelity & Surety Co., executed and delivered to the


Manila Ylang Ylang Distillery a surety bond, understood to
Issue: Whether the trial court erred in not awarding in favor of pay jointly and severally with Arranz as principal, the sum of
Manila Surety the premiums on the performance bonds executed by P90,000. The surety bond executed by Arranz and the
Manila Fidelity contains the following stipulation:
Universal in favor of the Reparations Commission.
The surety hereunder waives notice of
default and expressly agrees that it shall not be
necessary for the Manila Ylang Ylang Distillery, Ltd.
Held: YES. Universal is further ordered to pay the Manila Surety to proceed against the Principal upon his default or
P7,251.42 for the premiums and documentary stamps on the to exhaust the property of said Principal, before
proceeding against the surety, the Surety's liability
performance bonds.
under this bond being a primary one and shall be
eligible and demandable immediately upon
occurrence of such default.
Ratio: 2. To secure the surety against loss arising from the surety
bond, plaintiff executed a second mortgaged over the
 The payment of premiums on the bonds to the surety company properties which were transferred by the Manila Ylang
had been expressly undertaken by UNIVERSAL in the indemnity Ylang Distillery to Arranza. When the first installment of
agreements executed by it in favor of judgment, surety P50K became due the surety, Manila Fidelity, did not have
company. The premium is judgment, consideration for furnishing funds to pay the same, and neither did it have funds to pay
judgment, bonds and judgment, obligation to pay judgment, the second installment of P40K which became due.
same subsists for as long as judgment, liability of judgment, 3. Complaint was filed by the Manila Ylang Ylang Distillery and
surety shall exist. a supplemental complaint was later filed to include the
second installment of P40K already due. Manila Fidelity had
no funds with which to pay either the P50K or the P40K due
under the agreement and the only amount it was able to
raise was P20K. And that was paid to Manila Ylang Ylang
Distillery on account.
Page 151 of 154
INSURANCE G01 CASE DIGESTS

4. Manila Fidelity had no money with which to respond for the Now, therefore, if the above abounded Principal
obligation. Arranz was constrained to mortgage some of his shall pay promptly said installments and interest thereon
properties to PNB to secure a loan to pay their obligation. and shall in all respects do and fully observe all and singular
PNB, however, required Manila Fidelity to release the
the covenants, agreements and conditions as provided for
second mortgage it obtained from Arranz. Arranz paid the
property mortgaged from Manila Fidelity except one in in the aforesaid agreement of November 21, 1949, Annexes
order to release the property from the encumbrance. "A" and "B" respectively, to the true intent and meaning
5. Arranz then filed a complaint to recover the premiums it thereof, this obligation shall be null and void, otherwise, it
paid during the period when the property was under shall remain in full force and effect. (p. 16, R.O.A..)
mortgage. He argued that the amounts were never due and
owing to the defendant surety and that he paid it against
his will in order to be able to save the properties from loss
and obtain the credit accommodation from the Philippine As the loan and interest remained unpaid the surety continued to
National Bank. be bound to the creditor-obligee, and as a corollary its right to
6. collect the premium on the bond also continued.
7. The TC ruled against Arranz and dismissed the case. Thus,
this appeal.
ISSUE:
Plaintiff-appellant, therefore, cannot excuse himself from the
WON Arranz is obligated to pay the premium payment of the premium on the bond upon the failure or refusal of
notwithstanding the failure of Manila Fidelity to pay the the surety to pay the loan and the interest. Even if, therefore, the
indebtedness secured by it? YES, he is still obligated to pay. payment of the premium were against his will, still plaintiff-
appellant has no cause of action for the return thereof, because the
HELD:
surety was entitled thereto.
The premium is the consideration for furnishing the bond or
the guaranty. While the liability of the surety to the obligee subsists
the premium is collectible from the principal. Under the terms of
the contract of suretyship the surety's obligation is that the
principal pay the loan and the interest thereon, and that the surety
shall be relieved of his obligation when the loan or obligation
secured is paid.

Page 152 of 154


INSURANCE G01 CASE DIGESTS

On April 30, 1963 or about five (5) days before the expiration of the
154. CAPITAL INSURANCE V. RONQUILLO TRADING, 123 SCRA 526 liability on the bond,
(1983)
P.D. Marchessini and Co., Ltd. and Delgado Shipping Agencies, Inc.,
filed a case in the Court of First Instance of Manila against the
Capital Surety & Insurance Co. Inc. vs. Ronquillo Trading
Philippine Merchants Steamship Co., Inc.,
G.R. No. L-36488
Jose L. Bautista, doing business under the name and style of
July 25, 1983 "Ronquillo Trading", and the herein appellant Capital Insurance &
Surety Co., Inc. for the sum of $14,800.00 or its equivalent in
Facts: Philippine currency, for the loss they allegedly suffered as a direct
consequence of the failure of the defendants to load the stipulated
Capital Surety and Insurance Co., Inc., thru its general agent,
quantity of 406 U.S. surplus army vehicles.
executed and issued a surety bond in the amount of $14,800.00 or
its peso equivalent in behalf of Ronquillo The appellant was made party defendant because of the bond it
posted in behalf of the appellees. Upon the expiration of the 12
Trading and in favor of S.S. Eurygenes, its master, and/or its agents,
months life of the bond, the appellant made a formal demand for
Delgado Shipping Agencies.
the payment of the renewal premiums and cost of documentary
The bond was a guarantee for any additional freight which may be stamps for another year in the amount of P1,827.00.
determined to be due on a cargo of 258 surplus army vehicles
The appellees refused to pay, contending that the liability of the
consigned from Pusan, Korea to the
appellant under the surety bond accrued during the period of
Ronquillo Trading on board the S.S. Eurygenes and booked on said twelve months the said bond was originally in force and before its
vessel by the Philippine Merchants Steamship Company, Inc. expiration and that the defendants-appellees were under no
obligation to renew the surety bond.
In consideration for the issuance by the appellant of the aforesaid
surety bond the The appellant, therefore, filed a complaint to recover the sum of P
l,827.00 against the appellees in the City Court of Manila wherein
appellees executed an indemnity agreement whereby among other said court rendered judgment absolving the appellees from the
things, they jointly and severally promised to pay the appellant the complaint. The appellant appealed the judgment to the Court of
sum of P1,827.00 in advance as premium and documentary stamps
for each period of twelve months while the surety bond was in First Instance of Manila where the decision of the city court was
effect. affirmed and the complaint dismissed. Its motion for
Page 153 of 154
INSURANCE G01 CASE DIGESTS

reconsideration having been denied, appellant filed the instant or accrued liability so as not to let that liability lapse or expire and
appeal. thereby bar enforcement.

It must be noted that in the surety bond it was stipulated that the
"liability of surety on this bond would expire on May 5, 1963 and
Issue: said bond would be cancelled 15 days after its expiration, unless
Wether or not a surety's liability under the bond has accrued, during surety was notified of any existing obligations thereunder."
the period of twelve months the bond was originally in force and Under this stipulation the bond expired on the stated date and the
before its expiration and that herein appellees were under no phrase "unless surety was notified of any existing obligations
obligation to pay the premiums and costs of documentary stamps
thereunder" refers to obligations incurred during the term of the
for the succeeding period it was in effect? bond.
Held: Under the Indemnity Agreement, the appellees "agreed to pay the
Yes. COMPANY the sum of ONE THOUSAND EIGHT HUNDRED ONLY
(P1,800.00) Pesos, Philippine Currency, in advance as premium
Ratio: thereof for every twelve (12) months or fraction thereof, while this
bond or any renewal or substitution thereof was in effect."
The bond was given to secure payment by appellees of such
additional freight as would already be due on the cargo when it Obviously, the duration of the bond was for "every twelve (12)
actually arrived in Manila. The bond was not executed to secure months or fraction thereof, while this bond or any renewal or
obligation or liability which was still to arise after its twelve month substitution was in effect." Since the appellees opted not to renew
life. While it was true that the lower court held that the bond was the contract they cannot be obliged to pay the premiums.
still in effect after its expiry date, the effectivity was not due to a
renewal made by the appellees but because the surety bond More specifically, where a contract of surety is terminated under its
provided that "the liability of the surety will not expire if, as in this terms, the liability of the principal for premiums after such
case, it was notified of an existing obligation thereunder". termination ceases notwithstanding the pendency of a lawsuit to
enforce a liability that accrued during its stipulated lifetime. The
The meaning of the bond's still being in effect was that, the suit on appeal was dismissed for lack of merit. The decision of the court a
the bond instituted by the obligees prior to the expiration of the quo was affirmed.
"liability" thereunder was only for the purpose of enforcing that
liability and amounted to notice to appellant of an already existing

Page 154 of 154

You might also like